127
8/8/2019 22573582 This Work Blends Together Classic Inequality Results With Brand New http://slidepdf.com/reader/full/22573582-this-work-blends-together-classic-inequality-results-with-brand-new 1/127 a 1

22573582 This Work Blends Together Classic Inequality Results With Brand New

Embed Size (px)

Citation preview

Page 1: 22573582 This Work Blends Together Classic Inequality Results With Brand New

8/8/2019 22573582 This Work Blends Together Classic Inequality Results With Brand New

http://slidepdf.com/reader/full/22573582-this-work-blends-together-classic-inequality-results-with-brand-new 1/127

a

1

Page 2: 22573582 This Work Blends Together Classic Inequality Results With Brand New

8/8/2019 22573582 This Work Blends Together Classic Inequality Results With Brand New

http://slidepdf.com/reader/full/22573582-this-work-blends-together-classic-inequality-results-with-brand-new 2/127

2

b

Page 3: 22573582 This Work Blends Together Classic Inequality Results With Brand New

8/8/2019 22573582 This Work Blends Together Classic Inequality Results With Brand New

http://slidepdf.com/reader/full/22573582-this-work-blends-together-classic-inequality-results-with-brand-new 3/127

Preface

This work blends together classic inequality results with brand new problems,

some of which devised only a few days ago. What could be special about it when somany inequality problem books have already been written? We strongly believe that

even if the topic we plunge into is so general and popular our book is very different.

Of course, it is quite easy to say this, so we will give some supporting arguments. This

book contains a large variety of problems involving inequalities, most of them difficult,

questions that became famous in competitions because of their beauty and difficulty.

And, even more importantly, throughout the text we employ our own solutions and

propose a large number of new original problems. There are memorable problems

in this book and memorable solutions as well. This is why this work will clearly

appeal to students who are used to use Cauchy-Schwarz as a verb and want to further

improve their algebraic skills and techniques. They will find here tough problems,new results, and even problems that could lead to research. The student who is not

as keen in this field will also be exposed to a wide variety of moderate and easy

problems, ideas, techniques, and all the ingredients leading to a good preparation

for mathematical contests. Some of the problems we chose to present are known,

but we have included them here with new solutions which show the diversity of ideas

pertaining to inequalities. Anyone will find here a challenge to prove his or her skills. If 

we have not convinced you, then please take a look at the last problems and hopefully

you will agree with us.

Finally, but not in the end, we would like to extend our deepest appreciation

to the proposers of the problems featured in this book and to apologize for not

giving all complete sources, even though we have given our best. Also, we would like

to thank Marian Tetiva, Dung Tran Nam, Constantin Tanasescu, Calin Popa and

Valentin Vornicu for the beautiful problems they have given us and for the precious

comments, to Cristian Baba, George Lascu and Calin Popa, for typesetting and for

the many pertinent observations they have provided.

The authors

3

Page 4: 22573582 This Work Blends Together Classic Inequality Results With Brand New

8/8/2019 22573582 This Work Blends Together Classic Inequality Results With Brand New

http://slidepdf.com/reader/full/22573582-this-work-blends-together-classic-inequality-results-with-brand-new 4/127

Page 5: 22573582 This Work Blends Together Classic Inequality Results With Brand New

8/8/2019 22573582 This Work Blends Together Classic Inequality Results With Brand New

http://slidepdf.com/reader/full/22573582-this-work-blends-together-classic-inequality-results-with-brand-new 5/127

Contents

Preface 3

Chapter 1. Problems 7

Chapter 2. Solutions 25

Glossary 121

Further reading 127

5

Page 6: 22573582 This Work Blends Together Classic Inequality Results With Brand New

8/8/2019 22573582 This Work Blends Together Classic Inequality Results With Brand New

http://slidepdf.com/reader/full/22573582-this-work-blends-together-classic-inequality-results-with-brand-new 6/127

Page 7: 22573582 This Work Blends Together Classic Inequality Results With Brand New

8/8/2019 22573582 This Work Blends Together Classic Inequality Results With Brand New

http://slidepdf.com/reader/full/22573582-this-work-blends-together-classic-inequality-results-with-brand-new 7/127

CHAPTER 1

Problems

7

Page 8: 22573582 This Work Blends Together Classic Inequality Results With Brand New

8/8/2019 22573582 This Work Blends Together Classic Inequality Results With Brand New

http://slidepdf.com/reader/full/22573582-this-work-blends-together-classic-inequality-results-with-brand-new 8/127

8 Problems

1. Prove that the inequality

 a2 + (1 − b)2 +

 b2 + (1 − c)2 +

 c2 + (1 − a)2 ≥ 3

√2

2

holds for arbitrary real numbers a,b,c.

Komal

2. [ Dinu Serbanescu ] If  a,b,c ∈ (0, 1) prove that√

abc +

 (1 − a)(1 − b)(1 − c) < 1.

Junior TST 2002, Romania

3. [ Mircea Lascu ] Let a,b,c be positive real numbers such that abc = 1. Prove

thatb + c√

a+

c + a√b

+a + b√

c≥ √

a +√

b +√

c + 3.

Gazeta Matematica

4. If the equation x4 + ax3 + 2x2 + bx + 1 = 0 has at least one real root, then

a2 + b2 ≥ 8.

Tournament of the Towns, 1993

5. Find the maximum value of the expression x3 + y3 + z3 − 3xyz where x2 + y2 +

z2 = 1 and x,y,z are real numbers.

6. Let a,b,c,x,y,z be positive real numbers such that x + y + z = 1. Prove that

ax + by + cz + 2 

(xy + yz + zx)(ab + bc + ca) ≤ a + b + c.

Ukraine, 2001

7. [ Darij Grinberg ] If  a, b, c are three positive real numbers, then

a

(b + c)2 +b

(c + a)2 +c

(a + b)2≥ 9

4 (a + b + c).

8. [ Hojoo Lee ] Let a,b,c ≥ 0. Prove that a4 + a2b2 + b4 +

 b4 + b2c2 + c4 +

 c4 + c2a2 + a4 ≥

≥ a 

2a2 + bc + b 

2b2 + ca + c 

2c2 + ab.

Gazeta Matematica

9. If  a,b,c are positive real numbers such that abc = 2, then

a3 + b3 + c3 ≥ a√

b + c + b√

c + a + c√

a + b.

Page 9: 22573582 This Work Blends Together Classic Inequality Results With Brand New

8/8/2019 22573582 This Work Blends Together Classic Inequality Results With Brand New

http://slidepdf.com/reader/full/22573582-this-work-blends-together-classic-inequality-results-with-brand-new 9/127

Old and New Inequalities 9

When does equality hold?

JBMO 2002 Shortlist

10. [ Ioan Tomescu ] Let x,y,z > 0. Prove that

xyz

(1 + 3x)(x + 8y)(y + 9z)(z + 6)≤ 1

74.

When do we have equality?

Gazeta Matematica

11. [ Mihai Piticari, Dan Popescu ] Prove that5(a2 + b2 + c2) ≤ 6(a3 + b3 + c3) + 1,

for all a,b,c > 0 with a + b + c = 1.

12. [ Mircea Lascu ] Let x1, x2, . . . , xn ∈ R, n ≥ 2 and a > 0 such that x1 +

x2 + . . . + xn = a and x21 + x2

2 + . . . + x2n ≤ a2

n − 1. Prove that xi ∈

0,

2a

n

, for all

i ∈ {1, 2, . . . , n}.

13. [ Adrian Zahariuc ] Prove that for any a,b,c ∈ (1, 2) the following inequality

holds

b√a4b

√c − c

√a

+ c√b4c

√a − a

√b

+ a√c4a

√b − b

√c

≥ 1.

14. For positive real numbers a,b,c such that abc ≤ 1, prove that

a

b+

b

c+

c

a≥ a + b + c.

15. [ Vasile Cırtoaje, Mircea Lascu ] Let a,b,c,x,y,z be positive real numbers

such that a + x ≥ b + y ≥ c + z and a + b + c = x + y + z. Prove that ay + bx ≥ ac + xz.

16. [ Vasile Cırtoaje, Mircea Lascu ] Let a,b,c be positive real numbers so that

abc = 1. Prove that

1 +3

a + b + c≥ 6

ab + ac + bc.

Junior TST 2003, Romania

17. Let a,b,c be positive real numbers. Prove that

a3

b2+

b3

c2+

c3

a2≥ a2

b+

b2

c+

c2

a.

JBMO 2002 Shortlist

Page 10: 22573582 This Work Blends Together Classic Inequality Results With Brand New

8/8/2019 22573582 This Work Blends Together Classic Inequality Results With Brand New

http://slidepdf.com/reader/full/22573582-this-work-blends-together-classic-inequality-results-with-brand-new 10/127

10 Problems

18. Prove that if  n > 3 and x1, x2, . . . , xn > 0 have product 1, then

1

1 + x1 + x1x2+

1

1 + x2 + x2x3+ · · · +

1

1 + xn + xnx1> 1.

Russia, 2004

19. [ Marian Tetiva ] Let x,y,z be positive real numbers satisfying the condition

x2 + y2 + z2 + 2xyz = 1.

Prove that

a) xyz ≤1

8 ;

b) x + y + z ≤ 3

2;

c) xy + xz + yz ≤ 3

4≤ x2 + y2 + z2;

d) xy + xz + yz ≤ 1

2+ 2xyz .

20. [ Marius Olteanu ] Let x1, x2, x3, x4, x5 ∈ R so that x1 + x2 + x3 +x4 +x5 = 0.

Prove that

| cos x1| + | cos x2| + | cos x3| + | cos x4| + | cos x5| ≥ 1.

Gazeta Matematica

21. [ Florina Carlan, Marian Tetiva ] Prove that if  x, y,z > 0 satisfy the condition

x + y + z = xyz

then xy + xz + yz ≥ 3 +√

x2 + 1 + 

y2 + 1 +√

z2 + 1.

22. [ Laurentiu Panaitopol ] Prove that

1 + x2

1 + y + z2+

1 + y2

1 + z + x2+

1 + z2

1 + x + y2≥ 2,

for any real numbers x, y,z > −1.

JBMO, 2003

23. Let a,b,c > 0 with a + b + c = 1. Show that

a2 + b

b + c+

b2 + c

c + a+

c2 + a

a + b≥ 2.

24. Let a,b,c ≥ 0 such that a4 + b4 + c4 ≤ 2(a2b2 + b2c2 + c2a2). Prove that

a2 + b2 + c2 ≤ 2(ab + bc + ca).

Kvant, 1988

Page 11: 22573582 This Work Blends Together Classic Inequality Results With Brand New

8/8/2019 22573582 This Work Blends Together Classic Inequality Results With Brand New

http://slidepdf.com/reader/full/22573582-this-work-blends-together-classic-inequality-results-with-brand-new 11/127

Old and New Inequalities 11

25. Let n ≥ 2 and x1,...,xn be positive real numbers satisfying

1

x1 + 1998+

1

x2 + 1998+ ... +

1

xn + 1998=

1

1998.

Prove thatn√

x1x2...xnn − 1

≥ 1998.

Vietnam, 1998

26. [ Marian Tetiva ] Consider positive real numbers x,y,z so that

x2

+ y2

+ z2

= xyz.

Prove the following inequalities

a) xyz ≥ 27;

b) xy + xz + yz ≥ 27;

c) x + y + z ≥ 9;

d) xy + xz + yz ≥ 2 (x + y + z) + 9.

27. Let x,y,z be positive real numbers with sum 3. Prove that√

x +√

y +√

z ≥ xy + yz + zx.

Russia, 2002

28. [ D. Olteanu ] Let a,b,c be positive real numbers. Prove that

a + b

b + c· a

2a + b + c+

b + c

c + a· b

2b + c + a+

c + a

a + b· c

2c + a + b≥ 3

4.

Gazeta Matematica

29. For any positive real numbers a,b,c show that the following inequality holds

a

b+

b

c+

c

a≥ c + a

c + b+

a + b

a + c+

b + c

b + a.

India, 2002

30. Let a,b,c be positive real numbers. Prove that

a3

b2 − bc + c2+

b3

c2 − ac + a2+

c3

a2 − ab + b2≥ 3(ab + bc + ca)

a + b + c.

Proposed for the Balkan Mathematical Olympiad

31. [ Adrian Zahariuc ] Consider the pairwise distinct integers x1, x2, . . . , xn,

n ≥ 0. Prove that

x21 + x2

2 + · · · + x2n ≥ x1x2 + x2x3 + · · · + xnx1 + 2n − 3.

Page 12: 22573582 This Work Blends Together Classic Inequality Results With Brand New

8/8/2019 22573582 This Work Blends Together Classic Inequality Results With Brand New

http://slidepdf.com/reader/full/22573582-this-work-blends-together-classic-inequality-results-with-brand-new 12/127

12 Problems

32. [ Murray Klamkin ] Find the maximum value of the expression x21x2 + x2

2x3 +

· · · + x2n−1xn + x2

nx1 when x1, x2, . . . , xn−1, xn ≥ 0 add up to 1 and n > 2.

Crux Mathematicorum

33. Find the maximum value of the constant c such that for any

x1, x2, . . . , xn, · · · > 0 for which xk+1 ≥ x1 + x2 + · · · + xk for any k, the inequality

√x1 +

√x2 + · · · +

√xn ≤ c

√x1 + x2 + · · · + xn

also holds for any n.

IMO Shortlist, 1986

34. Given are positive real numbers a,b,c and x,y,z, for which a + x = b + y =

c + z = 1. Prove that

(abc + xyz) 1

ay+

1

bz+

1

cx

≥ 3.

Russia, 2002

35. [ Viorel Vajaitu, Alexandru Zaharescu ] Let a,b,c be positive real numbers.

Prove thatab

a + b + 2c+

bc

b + c + 2a+

ca

c + a + 2b ≤1

4(a + b + c).

Gazeta Matematica

36. Find the maximum value of the expression

a3(b + c + d) + b3(c + d + a) + c3(d + a + b) + d3(a + b + c)

where a,b,c,d are real numbers whose sum of squares is 1.

37. [ Walther Janous ] Let x,y,z be positive real numbers. Prove that

x

x +  (x + y)(x + z)+

y

y +  (y + z)(y + x)+

z

z +  (z + x)(z + y)≤ 1.

Crux Mathematicorum

38. Suppose that a1 < a2 < ... < an are real numbers for some integer n ≥ 2.

Prove that

a1a42 + a2a4

3 + ... + ana41 ≥ a2a4

1 + a3a42 + ... + a1a4

n.

Iran, 1999

39. [ Mircea Lascu ] Let a,b,c be positive real numbers. Prove that

b + c

a+

c + a

b+

a + b

c≥ 4

a

b + c+

b

c + a+

c

a + b

.

Page 13: 22573582 This Work Blends Together Classic Inequality Results With Brand New

8/8/2019 22573582 This Work Blends Together Classic Inequality Results With Brand New

http://slidepdf.com/reader/full/22573582-this-work-blends-together-classic-inequality-results-with-brand-new 13/127

Old and New Inequalities 13

40. Let a1, a2, . . . , an > 1 be positive integers. Prove that at least one of the

numbers a1

√a2, a2

√a3, . . . , an−1

√an, an

√a1 is less than or equal to 3

√3.

Adapted after a well-known problem

41. [ Mircea Lascu, Marian Tetiva ] Let x,y,z be positive real numbers which

satisfy the condition

xy + xz + yz + 2xyz = 1.

Prove that the following inequalities hold

a) xyz

1

8

;

b) x + y + z ≥ 3

2;

c)1

x+

1

y+

1

z≥ 4 (x + y + z);

d)1

x+

1

y+

1

z− 4 (x + y + z) ≥ (2z − 1)2

(z (2z + 1)), where z = max {x,y,z}.

42. [ Manlio Marangelli ] Prove that for any positive real numbers x,y,z,

3(x2y + y2z + z2x)(xy2 + yz2 + zx2) ≥ xyz(x + y + z)3.

43. [ Gabriel Dospinescu ] Prove that if  a,b,c are real numbers such that

max{a,b,c} − min{a,b,c} ≤ 1, then

1 + a3 + b3 + c3 + 6abc ≥ 3a2b + 3b2c + 3c2a

44. [ Gabriel Dospinescu ] Prove that for any positive real numbers a,b,c we have

27 +

2 +

a2

bc

2 +

b2

ca

2 +

c2

ab

≥ 6(a + b + c)

1

a+

1

b+

1

c

.

45. Let a0 =1

2and ak+1 = ak +

a2k

n. Prove that 1 − 1

n< an < 1.

TST Singapore

46. [ Calin Popa ] Let a,b,c be positive real numbers, with a,b,c ∈ (0, 1) such

that ab + bc + ca = 1. Prove that

a

1 − a2+

b

1 − b2+

c

1 − c2≥ 3

4

1 − a2

a+

1 − b2

b+

1 − c2

c

.

47. [ Titu Andreescu, Gabriel Dospinescu ] Let x,y,z ≤ 1 and x + y + z = 1.

Prove that1

1 + x2+

1

1 + y2+

1

1 + z2≤ 27

10.

Page 14: 22573582 This Work Blends Together Classic Inequality Results With Brand New

8/8/2019 22573582 This Work Blends Together Classic Inequality Results With Brand New

http://slidepdf.com/reader/full/22573582-this-work-blends-together-classic-inequality-results-with-brand-new 14/127

14 Problems

48. [ Gabriel Dospinescu ] Prove that if √

x +√

y +√

z = 1, then

(1 − x)2(1 − y)2(1 − z)2 ≥ 215xyz(x + y)(y + z)(z + x)

.

49. Let x,y,z be positive real numbers such that xyz = x + y + z + 2. Prove that

(1) xy + yz + zx ≥ 2(x + y + z);

(2)√

x +√

y +√

z ≤ 3

2

√xyz.

50. Prove that if  x,y,z are real numbers such that x2 + y2 + z2 = 2, then

x + y + z ≤ xyz + 2.

IMO Shortlist, 1987

51. [ Titu Andreescu, Gabriel Dospinescu ] Prove that for any x1, x2, . . . , xn ∈(0, 1) and for any permutation σ of the set {1, 2, . . . , n}, we have the inequality

n

i=1

1

1 − xi≥

1 +

ni=1

xi

n

·

n

i=1

1

1 − xi · xσ(i)

.

52. Let x1, x2, . . . , xn be positive real numbers such thatni=1

1

1 + xi= 1. Prove

thatni=1

√xi ≥ (n − 1)

ni=1

1√xi

.

Vojtech Jarnik

53. [ Titu Andreescu ] Let n > 3 and a1, a2, . . . , an be real numbers such that

a1 + a2 + . . . +an≥

n and a21 + a2

2 + . . . +a2n

≥n2. Prove that max

{a1, a2, . . . , an

} ≥2.

USAMO, 1999

54. [ Vasile Cırtoaje ] If  a,b,c,d are positive real numbers, then

a − b

b + c+

b − c

c + d+

c − d

d + a+

d − a

a + b≥ 0.

55. If  x and y are positive real numbers, show that xy + yx > 1.

France, 1996

Page 15: 22573582 This Work Blends Together Classic Inequality Results With Brand New

8/8/2019 22573582 This Work Blends Together Classic Inequality Results With Brand New

http://slidepdf.com/reader/full/22573582-this-work-blends-together-classic-inequality-results-with-brand-new 15/127

Old and New Inequalities 15

56. Prove that if  a,b,c > 0 have product 1, then

(a + b)(b + c)(c + a) ≥ 4(a + b + c − 1).

MOSP, 2001

57. Prove that for any a,b,c > 0,

(a2 + b2 + c2)(a + b − c)(b + c − a)(c + a − b) ≤ abc(ab + bc + ca).

58. [ D.P.Mavlo ] Let a,b,c > 0. Prove that

3 + a + b + c +1

a+

1

b+

1

c+

a

b+

b

c+

c

a≥ 3

(a + 1)(b + 1)(c + 1)

1 + abc.

Kvant, 1988

59. [ Gabriel Dospinescu ] Prove that for any positive real numbers x1, x2, . . . , xn

with product 1 we have the inequality

nn ·ni=1

(xni + 1) ≥

ni=1

xi +

ni=1

1

xi

n

.

60. Let a, b, c, d > 0 such that a + b + c = 1. Prove that

a3 + b3 + c3 + abcd ≥ min

1

4,

1

9+

d

27

.

Kvant, 1993

61. Prove that for any real numbers a,b,c we have the inequality(1 + a2)2(1 + b2)2(a − c)2(b − c)2 ≥ (1 + a2)(1+ b2)(1+ c2)(a − b)2(b − c)2(c − a)2.

AMM

62. [ Titu Andreescu, Mircea Lascu ] Let α,x,y,z be positive real numbers such

that xyz = 1 and α ≥ 1. Prove that

y + z+

z + x+

x + y≥ 3

2.

63. Prove that for any real numbers x1, . . . , xn, y1, . . . , yn such that x21+· · ·+x2

n =

y21 + · · · + y2

n = 1,

(x1y2 − x2y1)2 ≤ 2

1 −

nk=1

xkyk

.

Korea, 2001

Page 16: 22573582 This Work Blends Together Classic Inequality Results With Brand New

8/8/2019 22573582 This Work Blends Together Classic Inequality Results With Brand New

http://slidepdf.com/reader/full/22573582-this-work-blends-together-classic-inequality-results-with-brand-new 16/127

16 Problems

64. [ Laurentiu Panaitopol ] Let a1, a2, . . . , an be pairwise distinct positive inte-

gers. Prove that

a21 + a2

2 + · · · + a2n ≥ 2n + 1

3(a1 + a2 + · · · + an).

TST Romania

65. [ Calin Popa ] Let a,b,c be positive real numbers such that a + b + c = 1.

Prove that

b√

c

a(√3c + √ab) +

c√

a

b(√3a + √bc) +

a√

b

c(√3b + √ca) ≥3√

3

4 .

66. [ Titu Andreescu, Gabriel Dospinescu ] Let a,b,c,d be real numbers such that

(1 + a2)(1 + b2)(1 + c2)(1 + d2) = 16. Prove that

−3 ≤ ab + bc + cd + da + ac + bd − abcd ≤ 5.

67. Prove that

(a2 + 2)(b2 + 2)(c2 + 2)

≥9(ab + bc + ca)

for any positive real numbers a,b,c.

APMO, 2004

68. [ Vasile Cırtoa je ] Prove that if 0 < x ≤ y ≤ z and x + y + z = xyz + 2, then

a) (1 − xy)(1 − yz)(1 − xz) ≥ 0;

b) x2y ≤ 1, x3y2 ≤ 32

27.

69. [ Titu Andreescu ] Let a,b,c be positive real numbers such that a+b+c ≥ abc.

Prove that at least two of the inequalities

2

a +

3

b +

6

c ≥ 6,

2

b +

3

c +

6

a ≥ 6,

2

c +

3

a +

6

b ≥ 6,

are true.

TST 2001, USA

70. [ Gabriel Dospinescu, Marian Tetiva ] Let x, y,z > 0 such that

x + y + z = xyz.

Prove that

(x − 1) (y − 1) (z − 1) ≤ 6√

3 − 10.

Page 17: 22573582 This Work Blends Together Classic Inequality Results With Brand New

8/8/2019 22573582 This Work Blends Together Classic Inequality Results With Brand New

http://slidepdf.com/reader/full/22573582-this-work-blends-together-classic-inequality-results-with-brand-new 17/127

Old and New Inequalities 17

71. [ Marian Tetiva ] Prove that for any positive real numbers a,b,c,a3 − b3

a + b+

b3 − c3

b + c+

c3 − a3

c + a

≤ (a − b)2 + (b − c)2 + (c − a)2

4.

Moldova TST, 2004

72. [ Titu Andreescu ] Let a,b,c be positive real numbers. Prove that

(a5 − a2 + 3)(b5 − b2 + 3)(c5 − c2 + 3) ≥ (a + b + c)3.

USAMO, 2004

73. [ Gabriel Dospinescu ] Let n > 2 and x1, x2, . . . , xn > 0 such thatn

k=1

xk

n

k=1

1

xk

= n2 + 1.

Prove that n

k=1

x2k

·

nk=1

1

x2k

> n2 + 4 +

2

n(n − 1).

74. [ Gabriel Dospinescu, Mircea Lascu, Marian Tetiva ] Prove that for any po-

sitive real numbers a,b,c,

a2 + b2 + c2 + 2abc + 3 ≥ (1 + a)(1 + b)(1 + c).

75. [ Titu Andreescu, Zuming Feng ] Let a,b,c be positive real numbers. Prove

that(2a + b + c)2

2a2 + (b + c)2+

(2b + a + c)2

2b2 + (a + c)2+

(2c + a + b)2

2c2 + (a + b)2≤ 8.

USAMO, 2003

76. Prove that for any positive real numbers x, y and any positive integers m,n,

(n−1)(m−1)(xm+n+ym+n)+ (m+n−1)(xmyn+xnym) ≥ mn(xm+n−1y+ym+n−1x).

Austrian-Polish Competition, 1995

77. Let a,b,c,d,e be positive real numbers such that abcde = 1. Prove that

a + abc

1 + ab + abcd+

b + bcd

1 + bc + bcde+

c + cde

1 + cd + cdea+

d + dea

1 + de + deab+

e + eab

1 + ea + eabc≥ 10

3.

Crux Mathematicorum

Page 18: 22573582 This Work Blends Together Classic Inequality Results With Brand New

8/8/2019 22573582 This Work Blends Together Classic Inequality Results With Brand New

http://slidepdf.com/reader/full/22573582-this-work-blends-together-classic-inequality-results-with-brand-new 18/127

18 Problems

78. [ Titu Andreescu ] Prove that for any a,b,c, ∈

0,π

2

the following inequality

holds

sin a · sin(a − b) · sin(a − c)

sin(b + c)+

sin b · sin(b − c) · sin(b − a)

sin(c + a)+

sin c · sin(c − a) · sin(c − b)

sin(a + b)≥ 0.

TST 2003, USA

79. Prove that if  a,b,c are positive real numbers then,

 a4 + b4 + c4 +

 a2b2 + b2c2 + c2a2 ≥

 a3b + b3c + c3a +

 ab3 + bc3 + ca3.

KMO Summer Program Test, 2001

80. [ Gabriel Dospinescu, Mircea Lascu ] For a given n > 2 find the smallest

constant kn with the property: if  a1, . . . , an > 0 have product 1, then

a1a2

(a21 + a2)(a2

2 + a1)+

a2a3

(a22 + a3)(a2

3 + a2)+ · · · +

ana1

(a2n + a1)(a2

1 + an)≤ kn.

81. [ Vasile Cırtoa je ] For any real numbers a,b,c,x,y,z prove that the inequality

holds

ax + by + cz +  (a2 + b2 + c2)(x2 + y2 + z2) ≥ 2

3(a + b + c)(x + y + z).

Kvant, 1989

82. [ Vasile Cırtoa je ] Prove that the sides a,b,c of a triangle satisfy the inequality

3

a

b+

b

c+

c

a− 1

≥ 2

b

a+

c

b+

a

c

.

83. [ Walther Janous ] Let n > 2 and let x1, x2, . . . , xn > 0 add up to 1. Prove

thatni=1

1 +

1

xi

ni=1

n − xi1 − xi

.

Crux Mathematicorum

84. [ Vasile Cırtoaje, Gheorghe Eckstein ] Consider positive real numbers

x1, x2,...,xn such that x1x2...xn = 1. Prove that

1

n − 1 + x1+

1

n − 1 + x2+ ... +

1

n − 1 + xn≤ 1.

TST 1999, Romania

85. [ Titu Andreescu ] Prove that for any nonnegative real numbers a,b,c such

that a2 + b2 + c2 + abc = 4 we have 0 ≤ ab + bc + ca − abc ≤ 2.

USAMO, 2001

Page 19: 22573582 This Work Blends Together Classic Inequality Results With Brand New

8/8/2019 22573582 This Work Blends Together Classic Inequality Results With Brand New

http://slidepdf.com/reader/full/22573582-this-work-blends-together-classic-inequality-results-with-brand-new 19/127

Old and New Inequalities 19

86. [ Titu Andreescu ] Prove that for any positive real numbers a,b,c the following

inequality holds

a + b + c

3− 3

√abc ≤ max{(

√a −

√b)2, (

√b − √

c)2, (√

c − √a)2}.

TST 2000, USA

87. [ Kiran Kedlaya ] Let a,b,c be positive real numbers. Prove that

a +√

ab + 3√

abc

3 ≤3 a ·

a + b

2 ·

a + b + c

3

.

88. Find the greatest constant k such that for any positive integer n which is not

a square, |(1 +√

n)sin(π√

n)| > k.

Vietnamese IMO Training Camp, 1995

89. [ Dung Tran Nam ] Let x, y,z > 0 such that (x + y + z)3 = 32xyz. Find the

minimum and maximum of x4 + y4 + z4

(x + y + z)4.

Vietnam, 2004

90. [ George Tsintifas ] Prove that for any a, b, c, d > 0,

(a + b)3(b + c)3(c + d)3(d + a)3 ≥ 16a2b2c2d2(a + b + c + d)4.

Crux Mathematicorum

91. [ Titu Andreescu, Gabriel Dospinescu ] Find the maximum value of the ex-

pression(ab)n

1 − ab+

(bc)n

1 − bc+

(ca)n

1 − ca

where a,b,c are nonnegative real numbers which add up to 1 and n is some positive

integer.

92. Let a,b,c be positive real numbers. Prove that

1

a(1 + b)+

1

b(1 + c)+

1

c(1 + a)≥ 3

3√

abc(1 +3√

abc).

93. [ Dung Tran Nam ] Prove that for any real numbers a,b,c such that a2 + b2 +

c2 = 9,

2(a + b + c) − abc ≤ 10.

Vietnam, 2002

Page 20: 22573582 This Work Blends Together Classic Inequality Results With Brand New

8/8/2019 22573582 This Work Blends Together Classic Inequality Results With Brand New

http://slidepdf.com/reader/full/22573582-this-work-blends-together-classic-inequality-results-with-brand-new 20/127

20 Problems

94. [ Vasile Cırtoaje ] Let a,b,c be positive real numbers. Prove thata +

1

b− 1

b +

1

c− 1

+

b +

1

c− 1

c +

1

a− 1

+

c +

1

a− 1

a +

1

b− 1

≥ 3.

95. [ Gabriel Dospinescu ] Let n be an integer greater than 2. Find the greatest

real number mn and the least real number M n such that for any positive real numbers

x1, x2, . . . , xn (with xn = x0, xn+1 = x1),

mn ≤ni=1

xixi−1 + 2(n − 1)xi + xi+1 ≤ M n.

96. [ Vasile Cırtoaje ] If  x,y,z are positive real numbers, then

1

x2 + xy + y2+

1

y2 + yz + z2+

1

z2 + zx + z2≥ 9

(x + y + z)2.

Gazeta Matematica

97. [ Vasile Cırtoaje ] For any a, b, c, d > 0 prove that

2(a3 + 1)(b3 + 1)(c3 + 1)(d3 + 1) ≥ (1 + abcd)(1 + a2)(1 + b2)(1 + c2)(1 + d2).

Gazeta Matematica

98. Prove that for any real numbers a,b,c,

(a + b)4 + (b + c)4 + (c + a)4 ≥ 4

7(a4 + b4 + c4).

Vietnam TST, 1996

99. Prove that if  a,b,c are positive real numbers such that abc = 1, then

1

1 + a + b+

1

1 + b + c+

1

1 + c + a≤ 1

2 + a+

1

2 + b+

1

2 + c.

Bulgaria, 1997

100. [ Dung Tran Nam ] Find the minimum value of the expression1

a+

2

b+

3

cwhere a,b,c are positive real numbers such that 21ab + 2bc + 8ca ≤ 12.

Vietnam, 2001

101. [ Titu Andreescu, Gabriel Dospinescu ] Prove that for any x,y,z,a,b,c > 0

such that xy + yz + zx = 3,

a

b + c(y + z) +

b

c + a(z + x) +

c

a + b(x + y) ≥ 3.

Page 21: 22573582 This Work Blends Together Classic Inequality Results With Brand New

8/8/2019 22573582 This Work Blends Together Classic Inequality Results With Brand New

http://slidepdf.com/reader/full/22573582-this-work-blends-together-classic-inequality-results-with-brand-new 21/127

Old and New Inequalities 21

102. Let a,b,c be positive real numbers. Prove that

(b + c − a)2

(b + c)2 + a2+

(c + a − b)2

(c + a)2 + b2+

(a + b − c)2

(a + b)2 + c2≥ 3

5.

Japan, 1997

103. [ Vasile Cırtoaje, Gabriel Dospinescu ] Prove that if  a1, a2, . . . , an ≥ 0 then

an1 + an2 + · · · + ann − na1a2 . . . an ≥ (n − 1)

a1 + a2 + · · · + an−1

n − 1− an

n

where an is the least among the numbers a1, a2, . . . , an.

104. [ Turkevici ] Prove that for all positive real numbers x,y,z,t,

x4 + y4 + z4 + t4 + 2xyzt ≥ x2y2 + y2z2 + z2t2 + t2x2 + x2z2 + y2t2.

Kvant

105. Prove that for any real numbers a1, a2, . . . , an the following inequality holds

ni=1

ai

2

≤n

i,j=1

ij

i + j − 1aiaj .

106. Prove that if  a1, a2, . . . , an, b1, . . . , bn are real numbers between 1001 and

2002, inclusively, such that a21 + a2

2 + · · · + a2n = b2

1 + b22 + · · · + b2

n, then we have the

inequalitya3

1

b1+

a32

b2+ · · · +

a3n

bn≤ 17

10

a2

1 + a22 + · · · + a2

n

.

TST Singapore

107. [ Titu Andreescu, Gabriel Dospinescu ] Prove that if  a,b,c are positive real

numbers which add up to 1, then

(a2 + b2)(b2 + c2)(c2 + a2) ≥ 8(a2b2 + b2c2 + c2a2)2.

108. [ Vasile Cırtoaje ] If  a,b,c,d are positive real numbers such that abcd = 1,

then1

(1 + a)2+

1

(1 + b)2+

1

(1 + c)2+

1

(1 + d)2≥ 1.

Gazeta Matematica

109. [ Vasile Cırtoa je ] Let a,b,c be positive real numbers. Prove that

a2

b2 + c2+

b2

c2 + a2+

c2

a2 + b2≥ a

b + c+

b

c + a+

c

a + b.

Gazeta Matematica

Page 22: 22573582 This Work Blends Together Classic Inequality Results With Brand New

8/8/2019 22573582 This Work Blends Together Classic Inequality Results With Brand New

http://slidepdf.com/reader/full/22573582-this-work-blends-together-classic-inequality-results-with-brand-new 22/127

22 Problems

110. [ Gabriel Dospinescu ] Let a1, a2, . . . , an be real numbers and let S  be a

non-empty subset of {1, 2, . . . , n}. Prove that

i∈S

ai

2

1≤i≤j≤n

(ai + . . . + aj)2.

TST 2004, Romania

111. [ Dung Tran Nam ] Let x1, x2 . . . , x2004 be real numbers in the interval [−1, 1]

such that x31 + x3

2 + . . . +x32004 = 0. Find the maximal value of the x1 +x2 + · · ·+x2004.

112. [ Gabriel Dospinescu, Calin Popa ] Prove that if  n ≥ 2 and a1, a2, . . . , an

are real numbers with product 1, then

a21 + a2

2 + · · · + a2n − n ≥ 2n

n − 1· n√

n − 1(a1 + a2 + · · · + an − n).

113. [ Vasile Cırtoaje ] If  a,b,c are positive real numbers, then 2a

a + b+

 2b

b + c+

 2c

c + a≤ 3.

Gazeta Matematica

114. Prove the following inequality for positive real numbers x,y,z

(xy + yz + zx)

1

(x + y)2+

1

(y + z)2+

1

(z + x)2

≥ 9

4

Iran, 1996

115. Prove that for any x, y in the interval [0, 1], 1 + x2 +

 1 + y2 +

 (1 − x)2 + (1 − y)2 ≥ (1 +

√5)(1 − xy).

116. [ Suranyi ] Prove that for any positive real numbers a1, a2, . . . , an the fol-

lowing inequality holds

(n−1)(an1 +an2 +· · ·+ann)+na1a2 . . . an ≥ (a1 +a2 +· · ·+an)(an−11 +an−1

2 +· · ·+an−1n ).

Miklos Schweitzer Competition

117. Prove that for any x1, x2, . . . , xn > 0 with product 1,1≤i<j≤n

(xi − xj)2 ≥ni=1

x2i − n.

A generalization of Turkevici’s inequality

Page 23: 22573582 This Work Blends Together Classic Inequality Results With Brand New

8/8/2019 22573582 This Work Blends Together Classic Inequality Results With Brand New

http://slidepdf.com/reader/full/22573582-this-work-blends-together-classic-inequality-results-with-brand-new 23/127

Old and New Inequalities 23

118. [ Gabriel Dospinescu ] Find the minimum value of the expression

ni=1

 a1a2 . . . an

1 − (n − 1)ai

where a1, a2, . . . , an <1

n − 1add up to 1 and n > 2 is an integer.

119. [ Vasile Cırtoaje ] Let a1, a2, . . . , an < 1 be nonnegative real numbers such

that

a =  a21 + a2

2 + . . . + a2n

n ≥

√3

3

.

Prove thata1

1 − a21

+a2

1 − a22

+ . . . +an

1 − a2n

≥ na

1 − a2.

120. [ Vasile Cırtoaje, Mircea Lascu ] Let a,b,c,x,y,z be positive real numbers

such that

(a + b + c)(x + y + z) = (a2 + b2 + c2)(x2 + y2 + z2) = 4.

Prove that

abcxyz <1

36.

121. [ Gabriel Dospinescu ] For a given n > 2, find the minimal value of the

constant kn, such that if  x1, x2, . . . , xn > 0 have product 1, then

1√1 + knx1

+1√

1 + knx2

+ · · · +1√

1 + knxn≤ n − 1.

Mathlinks Contest

122. [ Vasile Cırtoaje, Gabriel Dospinescu ] For a given n > 2, find the maximal

value of the constant kn such that for any x1, x2, . . . , xn > 0 for which x21 + x2

2 + · · · +

x2n = 1 we have the inequality

(1 − x1)(1 − x2) . . . (1 − xn) ≥ knx1x2 . . . xn.

Page 24: 22573582 This Work Blends Together Classic Inequality Results With Brand New

8/8/2019 22573582 This Work Blends Together Classic Inequality Results With Brand New

http://slidepdf.com/reader/full/22573582-this-work-blends-together-classic-inequality-results-with-brand-new 24/127

Page 25: 22573582 This Work Blends Together Classic Inequality Results With Brand New

8/8/2019 22573582 This Work Blends Together Classic Inequality Results With Brand New

http://slidepdf.com/reader/full/22573582-this-work-blends-together-classic-inequality-results-with-brand-new 25/127

CHAPTER 2

Solutions

25

Page 26: 22573582 This Work Blends Together Classic Inequality Results With Brand New

8/8/2019 22573582 This Work Blends Together Classic Inequality Results With Brand New

http://slidepdf.com/reader/full/22573582-this-work-blends-together-classic-inequality-results-with-brand-new 26/127

26 Solutions

1. Prove that the inequality

 a2 + (1 − b)2 +

 b2 + (1 − c)2 +

 c2 + (1 − a)2 ≥ 3

√2

2

holds for arbitrary real numbers a,b,c.

Komal

First solution :

Applying Minkowsky’s Inequality to the left-hand side we have

 a

2

+ (1 − b)2

b2

+ (1 − c)2

c2

+ (1 − a)2

≥  (a + b + c)

2

+ (3 − a − b − c)2

.Denoting a + b + c = x we get

(a + b + c)2 + (3 − a − b − c)2 = 2

x − 3

2

2

+9

2≥ 9

2,

and the conclusion follows.

Second solution :

We have the inequalities

 a2 + (1 − b)2 +

 b2 + (1 − c)2 +

 c2 + (1 − a)2 ≥

≥ |a| + |1 − b|√2

+ |b| + |1 − c|√2

+ |c| + |1 − a|√2

and because |x| + |1 − x| ≥ 1 for all real numbers x the last quantity is at least3√

2

2.

2. [ Dinu Serbanescu ] If  a,b,c ∈ (0, 1) prove that√

abc + 

(1 − a)(1 − b)(1 − c) < 1.

Junior TST 2002, Romania

First solution :

Observe that x12 < x

13 for x

∈(0, 1). Thus√

abc <3√

abc,

and  (1 − a)(1 − b)(1 − c) < 3

 (1 − a)(1 − b)(1 − c).

By the AM-GM Inequality,

√abc <

3√

abc ≤ a + b + c

3,

and

 (1 − a)(1 − b)(1 − c) < 3

 (1 − a)(1 − b)(1 − c) ≤ (1 − a) + (1 − b) + (1 − c)

3.

Page 27: 22573582 This Work Blends Together Classic Inequality Results With Brand New

8/8/2019 22573582 This Work Blends Together Classic Inequality Results With Brand New

http://slidepdf.com/reader/full/22573582-this-work-blends-together-classic-inequality-results-with-brand-new 27/127

Old and New Inequalities 27

Summing up, we obtain

√abc +

 (1 − a)(1 − b)(1 − c) <

a + b + c + 1 − a + 1 − b + 1 − c

3= 1,

as desired.

Second solution :

We have√

abc + 

(1 − a)(1 − b)(1 − c) <√

b · √c +√

1 − b · √1 − c ≤ 1,

by the Cauchy-Schwarz Inequality.

Third solution :

Let a = sin2 x, b = sin2 y, c = sin2 z, where x,y,z ∈

0,π

2

. The inequality

becomes

sin x · sin y · sin z + cos x · cos y · cos z < 1

and it follows from the inequalities

sin x · sin y · sin z + cos x · cos y · cos z < sin x · sin y + cos x · cos y = cos(x − y) ≤ 1

3. [ Mircea Lascu ] Let a,b,c be positive real numbers such that abc = 1. Prove

thatb + c√

a+ c + a√

b+ a + b√

c≥ √a + √b + √c + 3.

Gazeta Matematica

Solution :

From the AM-GM Inequality, we have

b + c√a

+c + a√

b+

a + b√c

≥ 2

 bc

a+

 ca

b+

 ab

c

=

=

 bc

a+  

ca

b

+

 

ca

b+

 ab

c

+

 ab

c+

 bc

a

≥ 2(√

a +√

b +√

c) ≥ √a +

√b +

√c + 3

6√abc =

√a +

√b +

√c + 3.

4. If the equation x4 + ax3 + 2x2 + bx + 1 = 0 has at least one real root, then

a2 + b2 ≥ 8.

Tournament of the Towns, 1993

Page 28: 22573582 This Work Blends Together Classic Inequality Results With Brand New

8/8/2019 22573582 This Work Blends Together Classic Inequality Results With Brand New

http://slidepdf.com/reader/full/22573582-this-work-blends-together-classic-inequality-results-with-brand-new 28/127

28 Solutions

Solution :

Let x be the real root of the equation. Using the Cauchy-Schwarz Inequality

we infer that

a2 + b2 ≥

x4 + 2x2 + 12

x2 + x6≥ 8,

because the last inequality is equivalent to (x2 − 1)4 ≥ 0.

5. Find the maximum value of the expression x3 + y3 + z3 − 3xyz where x2 + y2 +

z2 = 1 and x,y,z are real numbers.

Solution :

Let t = xy + yz + zx. Let us observe that

(x3 + y3 + z3 − 3xyz)2 = (x + y + z)2(1 − xy − yz − zx)2 = (1 + 2t)(1 − t)2.

We also have −1

2≤ t ≤ 1. Thus, we must find the maximum value of the expression

(1+2t)(1 − t)2, where −1

2≤ t ≤ 1. In this case we clearly have (1 + 2t)(t − 1)2 ≤ 1 ⇔

t2(3−2t) ≥ 0 and thus |x3 +y3 +z3−3xyz| ≤ 1. We have equality for x = 1, y = z = 0

and thus the maximum value is 1.

6. Let a,b,c,x,y,z be positive real numbers such that x + y + z = 1. Prove that

ax + by + cz + 2 

(xy + yz + zx)(ab + bc + ca) ≤ a + b + c.

Ukraine, 2001

First solution :

We will use the Cauchy-Schwarz Inequality twice. First, we can write ax+by+

cz ≤ √a2 + b2 + c2 ·

 x2 + y2 + z2 and then we apply again the Cauchy-Schwarz

Inequality to obtain:

ax + by + cz + 2 

(xy + yz + zx)(ab + bc + ca) ≤≤   a2 ·  x2 +  2 ab ·  2xy ≤

≤ 

x2 + 2

xy · 

a2 + 2

ab =

a.

Second solution :

The inequality being homogeneous in a,b,c we can assume that a + b + c = 1. We

apply this time the AM-GM Inequality and we find that

ax+by+cz+2 

(xy + yz + zx)(ab + bc + ca) ≤ ax+by+cz+xy+yz +zx+ab+bc+ca.

Consequently,

xy + yz + zx + ab + bc + ca =1 − x2 − y2 − z2

2+

1 − a2 − b2 − c2

2≤ 1 − ax − by − cz,

Page 29: 22573582 This Work Blends Together Classic Inequality Results With Brand New

8/8/2019 22573582 This Work Blends Together Classic Inequality Results With Brand New

http://slidepdf.com/reader/full/22573582-this-work-blends-together-classic-inequality-results-with-brand-new 29/127

Old and New Inequalities 29

the last one being equivalent to (x − a)2 + (y − b)2 + (z − c)2 ≥ 0.

7. [ Darij Grinberg ] If  a, b, c are three positive real numbers, then

a

(b + c)2+

b

(c + a)2+

c

(a + b)2≥ 9

4 (a + b + c).

First solution :

We rewrite the inequality as

(a + b + c) a

(b + c)2 +

b

(c + a)2 +

c

(a + b)2 ≥

9

4 .

Applying the Cauchy-Schwarz Inequality we get

(a + b + c)

a

(b + c)2 +

b

(c + a)2 +

c

(a + b)2

a

b + c+

b

c + a+

c

a + b

2

.

It remains to prove thata

b + c+

b

c + a+

c

a + b≥ 3

2which is well-known.

Second solution :

Without loss of generality we may assume that a + b + c = 1. Now, considerthe function f  : (0, 1) → (0, ∞), f (x) =

x

(1 − x)2. A short computation of derivatives

shows that f  is convex. Thus, we may apply Jensen’s Inequality and the conclusion

follows.

8. [ Hojoo Lee ] Let a,b,c ≥ 0. Prove that a4 + a2b2 + b4 +

 b4 + b2c2 + c4 +

 c4 + c2a2 + a4 ≥

≥ a 

2a2 + bc + b 

2b2 + ca + c 

2c2 + ab.

Gazeta Matematica

Solution :

We start from (a2−b2)2 ≥ 0. We rewrite it as 4a4 +4a2b2 +4b4 ≥ 3a4 +6a2b2 +3b4.

It follows that√

a4 + a2b2 + b2 ≥√

3

2(a2 + b2).

Using this observation, we find that a4 + a2b2 + b4

2≥ 3

a2

2.

But using the Cauchy-Schwarz Inequality we obtaina 

2a2 + bc2

a2

2a2 + bc ≤ 3

a2

2

and the inequality is proved.

Page 30: 22573582 This Work Blends Together Classic Inequality Results With Brand New

8/8/2019 22573582 This Work Blends Together Classic Inequality Results With Brand New

http://slidepdf.com/reader/full/22573582-this-work-blends-together-classic-inequality-results-with-brand-new 30/127

30 Solutions

9. If  a,b,c are positive real numbers such that abc = 2, then

a3 + b3 + c3 ≥ a√

b + c + b√

c + a + c√

a + b.

When does equality hold?

JBMO 2002 Shortlist

First solution :

Applying the Cauchy-Schwarz Inequality gives

3(a2 + b2 + c2) ≥ 3(a + b + c)2 (1)

and

(a2 + b2 + c2)2 ≤ (a + b + c)(a3 + b3 + c3). (2)

These two inequalities combined yield

a3 + b3 + c3 ≥ (a2 + b2 + c2)(a + b + c)

3

=(a2 + b2 + c2)[(b + c) + (a + c) + (a + b)]

6

≥ (a√

b + c + b√

a + c + c√

a + b)2

6(3)

Using the AM-GM Inequality we obtain

a√

b + c + b√

a + c + c√

a + b ≥ 3 3

 abc

 (a + b)(b + c)(c + a)

≥ 33

 abc

√8abc = 3

3√

8 = 6.

Thus

(a√

b + c + b√

a + c + c√

a + b)2 ≥ 6(a√

b + c + b√

a + c + c√

a + b). (4)

The desired inequality follows from (3) and (4).

Second solution :

We have

a√b + c + b√c + a + c√a + b ≤ 

2(a2 + b2 + c2)(a + b + c).

Using Chebyshev Inequality, we infer that 2(a2 + b2 + c2)(a + b + c) ≤

 6(a3 + b3 + c3)

and so it is enough to prove that a3 + b3 + c3 ≥ 3abc, which is true by the AM-GM

Inequality. We have equality if  a = b = c = 3√

2.

10. [ Ioan Tomescu ] Let x,y,z > 0. Prove that

xyz

(1 + 3x)(x + 8y)(y + 9z)(z + 6)≤ 1

74.

Page 31: 22573582 This Work Blends Together Classic Inequality Results With Brand New

8/8/2019 22573582 This Work Blends Together Classic Inequality Results With Brand New

http://slidepdf.com/reader/full/22573582-this-work-blends-together-classic-inequality-results-with-brand-new 31/127

Old and New Inequalities 31

When do we have equality?

Gazeta Matematica

Solution :

First, we write the inequality in the following form

(1 + 3x)

1 +

8y

x

1 +

9z

y

1 +

6

z

≥ 74.

But this follows immediately from Huygens Inequality. We have equality for

x = 2, y =

3

2 , z = 1.

11. [ Mihai Piticari, Dan Popescu ] Prove that

5(a2 + b2 + c2) ≤ 6(a3 + b3 + c3) + 1,

for all a,b,c > 0 with a + b + c = 1.

Solution :

Because a + b + c = 1, we have a3 + b3 + c3 = 3abc + a2 + b2 + c2 − ab − bc − ca.

The inequality becomes

5(a

2

+ b

2

+ c

2

) ≤ 18abc + 6(a

2

+ b

2

+ c

2

) − 6(ab + bc + ca) + 1 ⇔⇔ 18abc + 1 − 2(ab + bc + ca) + 1 ≥ 6(ab + bc + ca) ⇔⇔ 8(ab + bc + ca) ≤ 2 + 18abc ⇔ 4(ab + bc + ca) ≤ 1 + 9abc ⇔⇔ (1 − 2a)(1 − 2b)(1 − 2c) ≤ abc ⇔⇔ (b + c − a)(c + a − b)(a + b − c) ≤ abc,

which is equivalent to Schur’s Inequality.

12. [ Mircea Lascu ] Let x1, x2, . . . , xn ∈ R, n ≥ 2 and a > 0 such that x1 +

x2 + . . . + xn = a and x21 + x2

2 + . . . + x2n ≤ a2

n

−1

. Prove that xi ∈ 0,2a

n , for all

i ∈ {1, 2, . . . , n}.

Solution :

Using the Cauchy-Schwarz Inequality, we get

(a − x1)2 ≤ (n − 1)

x22 + x2

3 + · · · + x2n

≤ (n − 1)

a2

n − 1− x2

1

.

Thus,

a2 − 2ax1 + x21 ≤ a2 − (n − 1)x2

1 ⇔ x1

x1 − 2a

n

≤ 0

and the conclusion follows.

Page 32: 22573582 This Work Blends Together Classic Inequality Results With Brand New

8/8/2019 22573582 This Work Blends Together Classic Inequality Results With Brand New

http://slidepdf.com/reader/full/22573582-this-work-blends-together-classic-inequality-results-with-brand-new 32/127

32 Solutions

13. [ Adrian Zahariuc ] Prove that for any a,b,c ∈ (1, 2) the following inequality

holdsb√

a

4b√

c − c√

a+

c√

b

4c√

a − a√

b+

a√

c

4a√

b − b√

c≥ 1.

Solution :

The fact that a,b,c ∈ (1, 2) makes all denominators positive. Then

b√

a

4b√c − c√a ≥a

a + b + c ⇔ b(a + b + c) ≥√

a(4b

√c − c

√a) ⇔

⇔ (a + b)(b + c) ≥ 4b√

ac,

the last one coming from a + b ≥ 2√

ab and b + c ≥ 2√

bc. Writing the other two

inequalities and adding them up give the desired result.

14. For positive real numbers a,b,c such that abc ≤ 1, prove that

a

b+

b

c+

c

a≥ a + b + c.

First solution :

If  ab + bc + ca ≤ a + b + c, then the Cauchy-Schwarz Inequality solves the

problem:

a

b+

b

c+

c

a=

a2c + b2a + c2b

abc≥

(a + b + c)2

1

a+

1

b+

1

cabc

≥ a + b + c.

Otherwise, the same inequality gives

a

b+

b

c+

c

a=

a2c + b2a + c2b

abc≥

(ab + bc + ca)2

a + b + cabc

≥ a + b + c

(here we have used the fact that abc ≤ 1).

Second solution :

Replacing a,b,c by ta,tb,tc with t =1

3√

abcpreserves the value of the quantity in

the left-hand side of the inequality and increases the value of the right-hand side and

makes at · bt · ct = abct3 = 1. Hence we may assume without loss of generality that

abc = 1. Then there exist positive real numbers x,y,z such that a =y

x, b =

z

y, c =

x

z.

The Rearrangement Inequality gives

x3 + y3 + z3 ≥ x2y + y2z + z2x.

Page 33: 22573582 This Work Blends Together Classic Inequality Results With Brand New

8/8/2019 22573582 This Work Blends Together Classic Inequality Results With Brand New

http://slidepdf.com/reader/full/22573582-this-work-blends-together-classic-inequality-results-with-brand-new 33/127

Old and New Inequalities 33

Thusa

b+

b

c+

c

a=

x3 + y3 + z3

xyz≥ x2y + y2z + z2x

xyz= a + b + c

as desired.

Third solution :

Using the AM-GM Inequality, we deduce that

2a

b+

b

c≥ 3

3

 a2

bc≥ 3a.

Similarly, 2bc

+ ca

≥ 3b and 2ca

+ ab

≥ 3c. Adding these three inequalities, the conclusion

is immediate.

Forth solution :

Let x =9

 ab4

c2, y =

9

 ca4

b2, z =

9

 bc4

a2. Consequently, a = xy2, b = zx2, c = yz 2,

and also xyz ≤ 1. Thus, using the Rearrangement Inequality, we find that a

b=

x2

yz≥ xyz

x2

yz=

x3 ≥

xy2 =

a.

15. [ Vasile Cırtoaje, Mircea Lascu ] Let a,b,c,x,y,z be positive real numbers

such that a + x ≥ b + y ≥ c + z and a + b + c = x + y + z. Prove that ay + bx ≥ ac + xz.

Solution :

We have

ay+bx−ac−xz = a(y−c)+x(b−z) = a(a+b−x−z)+x(b−z) = a(a−x)+(a+x)(b−z) =

=1

2(a − x)2 +

1

2(a2 − x2) + (a + x)(b − z) =

1

2(a − x)2 +

1

2(a + x)(a − x + 2b − 2z) =

=1

2(a − x)2 +

1

2(a + x)(b − c + y − z) ≥ 0.

The equality occurs when a = x, b = z, c = y and 2x ≥ y + z.

16. [ Vasile Cırtoaje, Mircea Lascu ] Let a,b,c be positive real numbers so that

abc = 1. Prove that

1 +3

a + b + c≥ 6

ab + ac + bc.

Junior TST 2003, Romania

Solution :

We set x =1

a, y =

1

b, z =

1

cand observe that xyz = 1. The inequality is

equivalent to

1 +3

xy + yz + zx≥ 6

x + y + z.

Page 34: 22573582 This Work Blends Together Classic Inequality Results With Brand New

8/8/2019 22573582 This Work Blends Together Classic Inequality Results With Brand New

http://slidepdf.com/reader/full/22573582-this-work-blends-together-classic-inequality-results-with-brand-new 34/127

34 Solutions

From (x + y + z)2 ≥ 3(xy + yz + zx) we get

1 +3

xy + yz + zx≥ 1 +

9

(x + y + z)2,

so it suffices to prove that

1 +9

(x + y + z)2≥ 6

x + y + z.

The last inequality is equivalent to

1 − 3

x + y + z

2

≥ 0 and this ends the proof.

17. Let a,b,c be positive real numbers. Prove that

a3

b2+

b3

c2+

c3

a2≥ a2

b+

b2

c+

c2

a.

JBMO 2002 Shortlist

First solution :

We have

a3

b2≥ a2

b+ a − b ⇔ a3 + b3 ≥ ab(a + b) ⇔ (a − b)2(a + b) ≥ 0,

which is clearly true. Writing the analogous inequalities and adding them up gives

a3

b2 +b3

c2 +c3

a2 ≥a2

b + a − b +b2

c + b − c +c2

a + c − a =a2

b +b2

c +c2

a .

Second solution :

By the Cauchy-Schwarz Inequality we have

(a + b + c)

a3

b2+

b3

c2+

c3

a2

a2

b+

b2

c+

c2

a

2

,

so we only have to prove that

a2

b+

b2

c+

c2

a≥ a + b + c.

But this follows immediately from the Cauchy-Schwarz Inequality.

18. Prove that if  n > 3 and x1, x2, . . . , xn > 0 have product 1, then

1

1 + x1 + x1x2+

1

1 + x2 + x2x3+ · · · +

1

1 + xn + xnx1> 1.

Russia, 2004

Solution :

We use a similar form of the classical substitution x1 =a2

a1, x2 =

a3

a2, . . . , xn =

a1

an.

In this case the inequality becomes

a1

a1 + a2 + a3+

a2

a2 + a3 + a4+ · · · +

anan + a1 + a2

> 1

Page 35: 22573582 This Work Blends Together Classic Inequality Results With Brand New

8/8/2019 22573582 This Work Blends Together Classic Inequality Results With Brand New

http://slidepdf.com/reader/full/22573582-this-work-blends-together-classic-inequality-results-with-brand-new 35/127

Old and New Inequalities 35

and it is clear, because n > 3 and ai + ai+1 + ai+2 < a1 + a2 + · · · + an for all i.

19. [ Marian Tetiva ] Let x,y,z be positive real numbers satisfying the condition

x2 + y2 + z2 + 2xyz = 1.

Prove that

a) xyz ≤ 1

8;

b) x + y + z ≤ 3

2;

c) xy + xz + yz ≤3

4 ≤ x2 + y2 + z2;

d) xy + xz + yz ≤ 1

2+ 2xyz .

Solution :

a) This is very simple. From the AM-GM Inequality, we have

1 = x2 + y2 + z2 + 2xyz ≥ 4 4 

2x3y3z3 ⇒ x3y3z3 ≤ 1

2 · 44⇒ xyz ≤ 1

8.

b) Clearly, we must have x,y,z ∈ (0, 1). If we put s = x+y+z, we get immediately

from the given relation

s2

− 2s + 1 = 2(1 − x) (1 − y) (1 − z) .

Then, again by the AM-GM Inequality (1 − x, 1 − y, 1 − z being positive), we

obtain

s2 − 2s + 1 ≤ 2

1 − x + 1 − y + 1 − z

3

3

= 2

3 − s

3

3

.

After some easy calculations this yields

2s3 + 9s2 − 27 ≤ 0 ⇔ (2s − 3) (s + 3)2 ≤ 0

and the conclusion is plain.

c) These inequalities are simple consequences of a) and b):

xy + xz + yz ≤ (x + y + z)2

3≤ 1

3· 9

4= 3

4

and

x2 + y2 + z2 = 1 − 2xyz ≥ 1 − 2 · 1

8=

3

4.

d) This is more delicate; we first notice that there are always two of the three

numbers, both greater (or both less) than1

2. Because of symmetry, we may assume

that x, y ≤ 1

2, or x, y ≥ 1

2and then

(2x − 1)(2y − 1) ≥ 0 ⇔ x + y − 2xy ≤ 1

2.

Page 36: 22573582 This Work Blends Together Classic Inequality Results With Brand New

8/8/2019 22573582 This Work Blends Together Classic Inequality Results With Brand New

http://slidepdf.com/reader/full/22573582-this-work-blends-together-classic-inequality-results-with-brand-new 36/127

36 Solutions

On the other hand,

1 = x2 + y2 + z2 + 2xyz ≥ 2xy + z2 + 2xyz ⇒⇒ 2xy (1 + z) ≤ 1 − z2 ⇒ 2xy ≤ 1 − z.

Now, we only have to multiply side by side the inequalities from above

x + y − 2xy ≤ 1

2

and

z

≤1

−2xy

to get the desired result:

xz + yz − 2xyz ≤ 1

2− xy ⇔ xy + xz + yz ≤ 1

2+ 2xyz.

It is not important in the proof, but also notice that

x + y − 2xy = xy

1

x+

1

y− 2

> 0,

because1

xand

1

yare both greater than 1.

Remark .

1) One can obtain some other inequalities, using

z + 2xy ≤ 1

and the two likes

y + 2xz ≤ 1, x + 2yz ≤ 1.

For example, multiplying these inequalities by z , y , x respectively and adding the new

inequalities, we get

x2 + y2 + z2 + 6xyz ≤ x + y + z,

or

1 + 4xyz

≤x + y + z.

2) If  ABC  is any triangle, the numbers

x = sinA

2, y = sin

B

2, z = sin

2

satisfy the condition of this problem; conversely, if  x,y, z > 0 verify

x2 + y2 + z2 + 2xyz = 1

then there is a triangle ABC  so that

x = sinA

2, y = sin

B

2, z = sin

2.

According to this, new proofs can be given for such inequalities.

Page 37: 22573582 This Work Blends Together Classic Inequality Results With Brand New

8/8/2019 22573582 This Work Blends Together Classic Inequality Results With Brand New

http://slidepdf.com/reader/full/22573582-this-work-blends-together-classic-inequality-results-with-brand-new 37/127

Old and New Inequalities 37

20. [ Marius Olteanu ] Let x1, x2, x3, x4, x5 ∈ R so that x1 + x2 + x3 +x4 +x5 = 0.

Prove that

| cos x1| + | cos x2| + | cos x3| + | cos x4| + | cos x5| ≥ 1.

Gazeta Matematica

Solution :

It is immediate to prove that

| sin(x + y)| ≤ min{| cos x| + | cos y|, | sin x| + | sin y|}

and | cos(x + y)| ≤ min{| sin x| + | cos y|, | sin y| + | cos x|}.

Thus, we infer that

1 = | cos

5

i=1

xi

| ≤ | cos x1|+| sin

5

i=2

xi

| ≤ | cos x1|+| cos x2|+| cos(x3+x4+x5)|.

But in the same manner we can prove that

| cos(x3 + x4 + x5)| ≤ | cos x3| + | cos x4| + | cos x5|and the conclusion follows.

21. [ Florina Carlan, Marian Tetiva ] Prove that if  x, y,z > 0 satisfy the conditionx + y + z = xyz

then xy + xz + yz ≥ 3 +√

x2 + 1 + 

y2 + 1 +√

z2 + 1.

First solution :

We have

xyz = x + y + z ≥ 2√

xy + z ⇒ z (√

xy)2 − 2√

xy − z ≥ 0.

Because the positive root of the trinomial zt2 − 2t − z is

1 +√

1 + z2

z,

we get from here

√xy ≥ 1 +

√1 + z2

z⇔ z

√xy ≥ 1 +

 1 + z2.

Of course, we have two other similar inequalities. Then,

xy + xz + yz ≥ x√

yz + y√

xz + z√

xy ≥≥ 3 +

 x2 + 1 +

 y2 + 1 +

 z2 + 1,

and we have both a proof of the given inequality, and a little improvement of it.

Page 38: 22573582 This Work Blends Together Classic Inequality Results With Brand New

8/8/2019 22573582 This Work Blends Together Classic Inequality Results With Brand New

http://slidepdf.com/reader/full/22573582-this-work-blends-together-classic-inequality-results-with-brand-new 38/127

38 Solutions

Second solution :

Another improvement is as follows. Start from

1

x2+

1

y2+

1

z2≥ 1

xy+

1

xz+

1

yz= 1 ⇒ x2y2 + x2z2 + y2z2 ≥ x2y2z2,

which is equivalent to

(xy + xz + yz)2 ≥ 2xyz (x + y + z) + x2y2z2 = 3 (x + y + z)2 .

Further on,

(xy + xz + yz − 3)2 = (xy + xz + yz)2 − 6 (xy + xz + yz) + 9 ≥≥ 3 (x + y + z)2 − 6 (xy + xz + yz) + 9 = 3

x2 + y2 + z2

+ 9,

so that

xy + xz + yz ≥ 3 + 

3 (x2 + y2 + z2) + 9.

But  3 (x2 + y2 + z2) + 9 ≥

 x2 + 1 +

 y2 + 1 +

 z2 + 1

is a consequence of the Cauchy-Schwarz Inequality and we have a second improve-

ment and proof for the desired inequality:

xy + xz + yz ≥ 3 + 

3 (x2 + y2 + z2) + 9 ≥≥ 3 +

 x2 + 1 +

 y2 + 1 +

 z2 + 1.

22. [ Laurentiu Panaitopol ] Prove that

1 + x2

1 + y + z2+

1 + y2

1 + z + x2+

1 + z2

1 + x + y2≥ 2,

for any real numbers x, y,z > −1.

JBMO, 2003

Solution :

Let us observe that y ≤ 1 + y2

2and 1 + y + z2 > 0, so

1 + x2

1 + y + z2≥ 1 + x2

1 + z2 +1 + y2

2

and the similar relations. Setting a = 1 + x2, b = 1 + y2, c = 1 + z2, it is sufficient to

prove that

a

2c + b+

b

2a + c+

c

2b + a≥ 1 (1)

Page 39: 22573582 This Work Blends Together Classic Inequality Results With Brand New

8/8/2019 22573582 This Work Blends Together Classic Inequality Results With Brand New

http://slidepdf.com/reader/full/22573582-this-work-blends-together-classic-inequality-results-with-brand-new 39/127

Old and New Inequalities 39

for any a, b, c > 0. Let A = 2c + b, B = 2a + c, C  = 2b + a. Then a =C  + 4B − 2A

9,

b =A + 4C − 2B

9, c =

B + 4A − 2C 

9and the inequality (1) is rewritten as

A+

A

B+

B

C + 4

B

A+

B+

A

≥ 15.

Because A,B, C > 0, we have from the AM-GM Inequality that

A+

A

B+

B

C ≥ 3

3

 A

B· B

C · C 

A= 3

and BA

+ C B

+ AC 

≥ 3 and the conclusion follows.

An alternative solution for (1) is by using the Cauchy-Schwarz Inequality:

a

2c + b+

b

2a + c+

c

2b + a=

a2

2ac + ab+

b2

2ab + cb+

c2

2bc + ac≥ (a + b + c)2

3(ab + bc + ca)≥ 1.

23. Let a,b,c > 0 with a + b + c = 1. Show that

a2 + b

b + c+

b2 + c

c + a+

c2 + a

a + b≥ 2.

Solution :

Using the Cauchy-Schwarz Inequality, we find that

a2 + b

b + c≥

a2 + 1

2

a2(b + c) +

a2 +

ab

.

And so it is enough to prove thata2 + 1

2

a2(b + c) +

a2 +

ab

≥ 2 ⇔ 1 +

a22

≥ 2

a2(b + c) + 2

ab.

The last inequality can be transformed as follows

1 +

a22

≥ 2

a2(b + c) + 2

ab ⇔ 1 +

a22

≥ 2

a2 − 2

a3 + 2

ab ⇔

a22

+ 2

a3 ≥

a2,

and it is true, because

a3 ≥

a2

3(Chebyshev’s Inequality)

and

a2

2

a2

3.

Page 40: 22573582 This Work Blends Together Classic Inequality Results With Brand New

8/8/2019 22573582 This Work Blends Together Classic Inequality Results With Brand New

http://slidepdf.com/reader/full/22573582-this-work-blends-together-classic-inequality-results-with-brand-new 40/127

40 Solutions

24. Let a,b,c ≥ 0 such that a4 + b4 + c4 ≤ 2(a2b2 + b2c2 + c2a2). Prove that

a2 + b2 + c2 ≤ 2(ab + bc + ca).

Kvant, 1988

Solution :

The condition a4 ≤ 2

a2b2

is equivalent to

(a + b + c)(a + b − c)(b + c − a)(c + a − b) ≥ 0.

In any of the cases a = b + c, b = c + a, c = a + b, the inequalitya2 ≤ 2

ab

is clear. So, suppose a + b = c, b + c = a, c + a = b. Because at most one of the

numbers b + c − a, c + a − b, a + b − c is negative and their product is nonnegative,

all of them are positive. Thus, we may assume that

a2 < ab + ac, b2 < bc + ba, c2 < ca + cb

and the conclusion follows.

25. Let n ≥ 2 and x1,...,xn be positive real numbers satisfying

1

x1 + 1998+

1

x2 + 1998+ ... +

1

xn + 1998=

1

1998.

Prove thatn√

x1x2...xnn − 1

≥ 1998.

Vietnam, 1998

Solution :

Let 19981998 + xi

= ai. The problem reduces to proving that for any positive real

numbers a1, a2, . . . , an such that a1 + a2 + · · · + an = 1 we have the inequality

ni=1

1

ai− 1

≥ (n − 1)n.

This inequality can be obtained by multiplying the inequalities

1

ai− 1 =

a1 + · · · + ai−1 + ai+1 + · · · + anai

≥ (n − 1) n−1

 a1 . . . ai−1ai+1 . . . an

an−1i

Page 41: 22573582 This Work Blends Together Classic Inequality Results With Brand New

8/8/2019 22573582 This Work Blends Together Classic Inequality Results With Brand New

http://slidepdf.com/reader/full/22573582-this-work-blends-together-classic-inequality-results-with-brand-new 41/127

Old and New Inequalities 41

26. [ Marian Tetiva ] Consider positive real numbers x,y,z so that

x2 + y2 + z2 = xyz.

Prove the following inequalities

a) xyz ≥ 27;

b) xy + xz + yz ≥ 27;

c) x + y + z ≥ 9;

d) xy + xz + yz ≥ 2 (x + y + z) + 9.

Solution :

a), b), c) Using well-known inequalities, we have

xyz = x2 + y2 + z2 ≥ 3 3 

x2y2z2 ⇒ (xyz)3 ≥ 27 (xyz)2 ,

which yields xyz ≥ 27. Then

xy + xz + yz ≥ 33

 (xyz)

2 ≥ 33√

272 = 27

and

x + y + z ≥ 3 3√

xyz ≥ 33√

27 = 9.

d) We notice that x2

< xyz ⇒ x < yz and the likes. Consequently,

xy < yz · xz ⇒ 1 < z 2 ⇒ z > 1.

Hence all the three numbers must be greater than 1. Set

a = x − 1, b = y − 1, c = z − 1.

We then have a > 0, b > 0, c > 0 and

x = a + 1, y = b + 1, z = c + 1.

Replacing these in the given condition we get

(a + 1)2 + (b + 1)2 + (c + 1)2 = (a + 1) (b + 1) (c + 1)

which is the same as

a2 + b2 + c2 + a + b + c + 2 = abc + ab + ac + bc.

If we put q = ab + ac + bc, we have

q ≤ a2 + b2 + c2, 

3q ≤ a + b + c

and

abc ≤ q

332

=(3q)

32

27.

Page 42: 22573582 This Work Blends Together Classic Inequality Results With Brand New

8/8/2019 22573582 This Work Blends Together Classic Inequality Results With Brand New

http://slidepdf.com/reader/full/22573582-this-work-blends-together-classic-inequality-results-with-brand-new 42/127

42 Solutions

Thus

q + 

3q + 2 ≤ a2 + b2 + c2 + a + b + c + 2 =

= abc + ab + ac + bc ≤√

3q3

27+ q

and setting x =√

3q, we have

x + 2 ≤ x3

27⇔ (x − 6) (x + 3)2 ≥ 0.

Finally,  3q = x ≥ 6 ⇒ q = ab + ac + bc ≥ 12.

Now, recall that a = x − 1, b = y − 1, c = z − 1, so we get

(x − 1) (y − 1) + (x − 1) (z − 1) + (y − 1) (z − 1) ≥ 12 ⇒⇒ xy + xz + yz ≥ 2 (x + y + z) + 9;

and we are done.

One can also prove the stronger inequality

xy + xz + yz ≥ 4 (x + y + z) − 9.

Try it!

27. Let x,y,z be positive real numbers with sum 3. Prove that√

x +√

y +√

z ≥ xy + yz + zx.

Russia, 2002

Solution :

Rewrite the inequality in the form

x2 + 2√

x + y2 + 2√

y + z2 + 2√

z ≥ x2 + y2 + z2 + 2xy + 2yz + 2zx ⇔⇔ x2 + 2

√x + y2 + 2

√y + z2 + 2

√z ≥ 9.

Now, from the AM-GM Inequality, we have

x2 + 2√

x = x2 +√

x +√

x ≥ 33√

x2 · x = 3x,

y2 + 2√

y ≥ 3y, z2 + 2√

z ≥ 3z,

hence

x2 + y2 + z2 + 2√

x +√

y +√

z

≥ 3(x + y + z) ≥ 9.

28. [ D. Olteanu ] Let a,b,c be positive real numbers. Prove that

a + b

b + c· a

2a + b + c+

b + c

c + a· b

2b + c + a+

c + a

a + b· c

2c + a + b≥ 3

4.

Gazeta Matematica

Page 43: 22573582 This Work Blends Together Classic Inequality Results With Brand New

8/8/2019 22573582 This Work Blends Together Classic Inequality Results With Brand New

http://slidepdf.com/reader/full/22573582-this-work-blends-together-classic-inequality-results-with-brand-new 43/127

Old and New Inequalities 43

Solution :

We set x = a + b, y = b + c and z = a + c and after a few computations we obtain

the equivalent form

x

y+

y

z+

z

x+

x

x + y+

y

y + z+

z

z + x≥ 9

2.

But using the Cauchy-Schwarz Inequality,

x

y+

y

z+

z

x+

x

x + y+

y

y + z+

z

z + x≥ (x + y + z)2

xy + yz + zx+

(x + y + z)2

xy + yz + zx + x2 + y2 + z2=

2(x + y + z)4

2(xy + yz + zx)(xy + yz + zx + x2 + y2 + z2) ≥8(x + y + z)4

(xy + yz + zx + (x + y + z)2)2

and the conclusion follows.

29. For any positive real numbers a,b,c show that the following inequality holds

a

b+

b

c+

c

a≥ c + a

c + b+

a + b

a + c+

b + c

b + a.

India, 2002

Solution :

Let us takea

b= x,

b

c= y,

c

a= z. Observe that

a + cb + c

= 1 + xy1 + y

= x + 1 − x1 + y

.

Using similar relations, the problem reduces to proving that if  xyz = 1, then

x − 1

y + 1+

y − 1

z + 1+

z − 1

x + 1≥ 0 ⇔

⇔ (x2 − 1)(z + 1) + (y2 − 1)(x + 1) + (z2 − 1)(y + 1) ≥ 0 ⇔⇔

x2z +

x2 ≥

x + 3.

But this inequality is very easy. Indeed, using the AM-GM Inequality we havex2z ≥ 3 and so it remains to prove that

x2 ≥

x, which follows from the

inequalities

x2 ≥

x2

3≥

x.

30. Let a,b,c be positive real numbers. Prove that

a3

b2 − bc + c2+

b3

c2 − ac + a2+

c3

a2 − ab + b2≥ 3(ab + bc + ca)

a + b + c.

Proposed for the Balkan Mathematical Olympiad

Page 44: 22573582 This Work Blends Together Classic Inequality Results With Brand New

8/8/2019 22573582 This Work Blends Together Classic Inequality Results With Brand New

http://slidepdf.com/reader/full/22573582-this-work-blends-together-classic-inequality-results-with-brand-new 44/127

44 Solutions

First solution :

Since a + b + c ≥ 3(ab + bc + ca)

a + b + c, it suffices to prove that:

a3

b2 − bc + c2+

b3

c2 − ca + a2+

c3

a2 − ab + b2≥ a + b + c.

From the Cauchy-Schwartz Inequality, we get

a3

b2 − bc + c2≥

a2

2

a(b2 − bc + c2)

.

Thus we have to show that

(a2 + b2 + c2)2 ≥ (a + b + c) ·

a(b2 − bc + c2).

This inequality is equivalent to

a4 + b4 + c4 + abc(a + b + c) ≥ ab(a2 + b2) + bc(b2 + c2) + ca(c2 + a2),

which is just Schur’s Inequality.

Remark .

The inequality

a3

b2 − bc + c2+

b3

c2 − ca + a2+

c3

a2 − ab + b2≥ a + b + c

was proposed by Vasile Cartoaje in Gazeta Matematica as a special case (n = 3)of the more general inequality

2an − bn − cn

b2 − bc + c2+

2bn − cn − an

c2 − ca + a2+

2cn − an − bn

a2 − ab + b2≥ 0.

Second solution :

Rewrite our inequality as (b + c)a3

b3 + c3≥ 3(ab + bc + ca)

a + b + c.

But this follows from a more general result:

If  a,b,c,x,y,z > 0 then a(y + z)

b + c≥ 3

xyx

.

But this inequality is an immediate (and weaker) consequence of the result from

problem 101.

Third solution :

Let

A =

a3

b2

−bc + c2

Page 45: 22573582 This Work Blends Together Classic Inequality Results With Brand New

8/8/2019 22573582 This Work Blends Together Classic Inequality Results With Brand New

http://slidepdf.com/reader/full/22573582-this-work-blends-together-classic-inequality-results-with-brand-new 45/127

Old and New Inequalities 45

and

B = b3 + c3

b2 − bc + c2=

(b + c)(b2 − bc + c2)

b2 − bc + c2= 2

a.

So we get

A + B =

a3 1

b2 − bc + c2

=

=1

2

(b + c)(b2 − bc + c2)

1

b2 − bc + c2

≥ 1

2

√b + c

2

,

from the Cauchy-Schwarz Inequality. Hence

A ≥ 12√b + c

2

− 2

a =√a + b · √b + c −

a.

We denote

Aa =√

c + a ·√

b + a − a =

ab 

ab + a2 + a

and the similar relations with Ab and Ac. So A ≥ Aa+Ab+Ac. But because

a2

≥3

ab

we get

Aa

≥ ab

 a2

3 + a2 + a

=3

ab

a2

 

a2

3

+ a2

−a

and also the similar inequalities are true. So we only need to prove that

 

a2

3+ a2 − a

≥ a + b + c ⇔

 1

3+

a

a + b + c

2

≥ 2.

We consider the convex function f (t) =

 1

3+ t2. Using Jensen’s Inequality

we finally deduce that

 13 +

a

a + b + c2

≥ 2.

We have equality if and only if  a = b = c.

31. [ Adrian Zahariuc ] Consider the pairwise distinct integers x1, x2, . . . , xn,

n ≥ 0. Prove that

x21 + x2

2 + · · · + x2n ≥ x1x2 + x2x3 + · · · + xnx1 + 2n − 3.

Page 46: 22573582 This Work Blends Together Classic Inequality Results With Brand New

8/8/2019 22573582 This Work Blends Together Classic Inequality Results With Brand New

http://slidepdf.com/reader/full/22573582-this-work-blends-together-classic-inequality-results-with-brand-new 46/127

46 Solutions

Solution :

The inequality can be rewritten asni=1

(xi − xi+1)2 ≥ 2(2n − 3).

Let xm = min{x1, x2, . . . , xn} and xM  = max{x1, x2, . . . , xn}. Suppose without loss

of generality that m < M . Let

S 1 = (xm − xm+1)2 + · · · + (xM −1 − xM )2

and

S 2 = (xM  − xM +1)2 + · · · + (xn − x1)2 + (x1 − x2)2 + · · · + (xm−1 − xm)2.

The inequality

ki=1

a2i ≥ 1

k

ki=1

ai

2

(which follows from the Cauchy-Schwarz In-

equality) implies that

S 1 ≥ (xM  − xm)2

M − mand

S 2 ≥ (xM  − xm)2

n − (M − m).

So

ni=1

(xi − xi+1)2 = S 1 + S 2 ≥ (xM  − xm)2

1

M − m+

1

n − (M − m)

≥ (n − 1)2 4

n= 4n − 8 +

4

n> 4n − 8.

Butni=1

(xi − xi+1)2 ≡ni=1

xi − xi+1 = 0 mod 2

soni=1

(xi − xi+1)2 ≥ 4n − 6

and the problem is solved.

32. [ Murray Klamkin ] Find the maximum value of the expression x21x2 + x2

2x3 +

· · · + x2n−1xn + x2

nx1 when x1, x2, . . . , xn−1, xn ≥ 0 add up to 1 and n > 2.

Crux Mathematicorum

Solution :

First of all, it is clear that the maximum is at least4

27, because this value is

attained for x1 =1

3, x2 =

2

3, x3 = · · · = xn = 0. Now, we will prove by induction that

x21x2 + x2

2x3 + · · · + x2n−1xn + x2

nx1 ≤ 4

27

Page 47: 22573582 This Work Blends Together Classic Inequality Results With Brand New

8/8/2019 22573582 This Work Blends Together Classic Inequality Results With Brand New

http://slidepdf.com/reader/full/22573582-this-work-blends-together-classic-inequality-results-with-brand-new 47/127

Old and New Inequalities 47

for all x1, x2, . . . , xn−1, xn ≥ 0 which add up to 1. Let us prove first the inductive

step. Suppose the inequality is true for n and we will prove it for n + 1. We can of 

course assume that x2 = min{x1, x2, . . . , xn+1}. But this implies that

x21x2 + x2

2x3 + · · · + x2nx1 ≤ (x1 + x2)2x3 + x2

3x4 + · · · + x2n−1xn + x2

n(x1 + x2).

But from the inductive hypothesis we have

(x1 + x2)2x3 + x23x4 + · · · + x2

n−1xn + x2n(x1 + x2) ≤ 4

27

and the inductive step is proved. Thus, it remains to prove that a

2

b + b

2

c + c

2

a ≤4

27if  a + b + c = 1. We may of course assume that a is the greatest among a,b,c. In this

case the inequality a2b + b2c + c2a ≤

a +c

2

b +c

2

follows immediately from

abc ≥ b2c,a2c

2≥ ac2

2. Because

1 =a +

c

22

+a +

c

22

+ b +c

2≥ 3

3

 b +

c

2

a +

c

2

2

4,

we have proved that a2b + b2c + c2a ≤ 4

27and this shows that the maximum is indeed

4

27.

33. Find the maximum value of the constant c such that for any

x1, x2, . . . , xn, · · · > 0 for which xk+1 ≥ x1 + x2 + · · · + xk for any k, the inequality

√x1 +

√x2 + · · · +

√xn ≤ c

√x1 + x2 + · · · + xn

also holds for any n.

IMO Shortlist, 1986

Solution :

First, let us see what happens if xk+1 and x1 +x2 +· · ·+xk are close for any k. For

example, we can take xk = 2k

, because in this case we have x1+x2+· · ·+xk = xk+1−2.Thus, we find that

c ≥

nk=1

√2k

  nk=1

2k

for any n. Taking the limit, we find that c ≥ 1 +√

2. Now, let us prove that 1 +√

2

works. We will prove the inequality

√x1 +

√x2 + · · · +

√xn ≤ (1 +

√2)

√x1 + x2 + · · · + xn

Page 48: 22573582 This Work Blends Together Classic Inequality Results With Brand New

8/8/2019 22573582 This Work Blends Together Classic Inequality Results With Brand New

http://slidepdf.com/reader/full/22573582-this-work-blends-together-classic-inequality-results-with-brand-new 48/127

48 Solutions

by induction. For n = 1 or n = 2 it is clear. Suppose it is true for n and we will

prove that√

x1 +√

x2 + · · · +√

xn +√

xn+1 ≤ (1 +√

2)√

x1 + x2 + · · · + xn + xn+1.

Of course, it is enough to prove that

√xn+1 ≤ (1 +

√2)

 x1 + x2 + · · · + xn+1 − √

x1 + x2 + · · · + xn

which is equivalent to√

x1 + x2 + · · · + xn + 

x1 + x2 + · · · + xn+1 ≤ (1 +√

2)√

xn+1.

But this one follows because

x1 + x2 + · · · + xn ≤ xn+1.

34. Given are positive real numbers a,b,c and x,y,z, for which a + x = b + y =

c + z = 1. Prove that

(abc + xyz) 1

ay+

1

bz+

1

cx

≥ 3.

Russia, 2002

Solution :

Let us observe that abc + xyz = (1 − b)(1 − c) + ac + ab − a. Thus,

1

−c

a +

c

1 − b − 1 =

abc + xyz

a(1 − b) .

Using these identities we deduce immediately that

3 + (xyz + abc)

1

ay+

1

bz+

1

cx

=

a

1 − c+

b

1 − a+

c

1 − b+

1 − c

a+

1 − a

b+

1 − b

c.

Now, all we have to do is apply the AM-GM Inequality

a

1 − c+

b

1 − a+

c

1 − b+

1 − c

a+

1 − a

b+

1 − b

c≥ 6.

35. [ Viorel Vajaitu, Alexandru Zaharescu ] Let a,b,c be positive real numbers.

Prove thatab

a + b + 2c +

bc

b + c + 2a +

ca

c + a + 2b ≤1

4 (a + b + c).Gazeta Matematica

First solution :

We have the following chain of inequalities ab

a + b + 2c=

ab

a + c + b + c≤

ab

4

1

a + c+

1

b + c

=

a + b + c

4.

Page 49: 22573582 This Work Blends Together Classic Inequality Results With Brand New

8/8/2019 22573582 This Work Blends Together Classic Inequality Results With Brand New

http://slidepdf.com/reader/full/22573582-this-work-blends-together-classic-inequality-results-with-brand-new 49/127

Old and New Inequalities 49

Second solution :

Because the inequality is homogeneous we can consider without loss of generality

that a + b + c = 1 and so the inequality is equivalent to 1

a(a + 1)≤ 1

4abc.

We have1

t(t + 1)=

1

t− 1

t + 1, so the inequality is equivalent to

1

a≤

1

a + 1+

1

4abc.

We will prove now the following intercalation: 1

a≤ 9

4+

1

4abc≤

1

a + 1+

1

4abc.

The inequality in the right follows from the Cauchy-Schwarz Inequality: 1

a + 1

(a + 1)

≥ 9

and the identity

(a + 1) = 4. The inequality in the left can be written as

ab ≤1 + 9abc

4, which is exactly Schur’s Inequality.

36. Find the maximum value of the expression

a3(b + c + d) + b3(c + d + a) + c3(d + a + b) + d3(a + b + c)

where a,b,c,d are real numbers whose sum of squares is 1.

Solution :

The idea is to observe that a3(b+c+d)+ b3(c+d+a)+ c3(d+a+b)+d3(a+b+c) is

equal to

ab(a2 +b2). Now, because the expression ab(a2 +b2) appears when writing

(a − b)4, let us see how the initial expression can be written:ab(a2 + b2) =

a4 + b4 + 6a2b2 − (a − b)4

4=

=

3 a4 + 6 a2b2

− (a

−b)4

4 =3

− (a−

b)4

4 ≤3

4 .

The maximum is attained for a = b = c = d =1

2.

37. [ Walther Janous ] Let x,y,z be positive real numbers. Prove that

x

x + 

(x + y)(x + z)+

y

y + 

(y + z)(y + x)+

z

z + 

(z + x)(z + y)≤ 1.

Crux Mathematicorum

Page 50: 22573582 This Work Blends Together Classic Inequality Results With Brand New

8/8/2019 22573582 This Work Blends Together Classic Inequality Results With Brand New

http://slidepdf.com/reader/full/22573582-this-work-blends-together-classic-inequality-results-with-brand-new 50/127

50 Solutions

First solution :

We have (x + y)(x + z) = xy + (x2 + yz) + xz ≥ xy + 2x√

yz + xz = (√

xy +√

xz)2.

Hence x

x + 

(x + y)(x + z)≤

x

x +√

xy +√

xz.

But x

x +√

xy +√

xz=

√x√

x +√

y +√

z= 1

and this solves the problem.

Second solution :

From Huygens Inequality we have 

(x + y)(x + z) ≥ x +√

yz and using this

inequality for the similar ones we get

x

x + 

(x + y)(x + z)+

y

y + 

(y + z)(y + x)+

z

z + 

(z + x)(z + y)≤

≤ x

2x +√

yz+

y

2y +√

zx+

z

2z +√

xy.

Now, we denote with a =

√yz

x, b =

√zx

y, c =

√xy

zand the inequality becomes

1

2 + a+

1

2 + b+

1

2 + c ≤1.

From the above notations we can see that abc = 1, so the last inequality becomes

after clearing the denominators ab + bc + ca ≥ 3, which follows from the AM-GM

Inequality.

38. Suppose that a1 < a2 < ... < an are real numbers for some integer n ≥ 2.

Prove that

a1a42 + a2a4

3 + ... + ana41 ≥ a2a4

1 + a3a42 + ... + a1a4

n.

Iran, 1999

Solution :

A quick look shows that as soon as we prove the inequality for n = 3, it will be

proved by induction for larger n. Thus, we must prove that for any a < b < c we have

ab(b3 − a3) + bc(c3 − b3) ≥ ca(c3 − a3) ⇔ (c3 − b3)(ac − bc) ≤ (b3 − a3)(ab − ac).

Because a < b < c, the last inequality reduces to a(b2 + ab + a2) ≤ c(c2 + bc + b2).

And this last inequality is equivalent to (c − a)(a2 + b2 + c2 + ab + bc + ca) ≥ 0, which

is clear.

39. [ Mircea Lascu ] Let a,b,c be positive real numbers. Prove that

b + c

a+

c + a

b+

a + b

c≥ 4

a

b + c+

b

c + a+

c

a + b

.

Page 51: 22573582 This Work Blends Together Classic Inequality Results With Brand New

8/8/2019 22573582 This Work Blends Together Classic Inequality Results With Brand New

http://slidepdf.com/reader/full/22573582-this-work-blends-together-classic-inequality-results-with-brand-new 51/127

Old and New Inequalities 51

Solution :

Using the inequality1

x + y≤ 1

4x+

1

4ywe infer that

4a

b + c≤ a

b+

a

c,

4b

a + c≤ b

a+

b

cand

4c

a + b≤ c

a+

c

b.

Adding up these three inequalities, the conclusion follows.

40. Let a1, a2, . . . , an > 1 be positive integers. Prove that at least one of the

numbers a1

√a2, a2

√a3, . . . , an−1

√an, an

√a1 is less than or equal to 3

√3.

Adapted after a well-known problem

Solution :

Suppose we have a1a1

i+1 > 313 for all i. First, we will prove that n

1n ≤ 3

13 for all

natural number n. For n = 1, 2, 3, 4 it is clear. Suppose the inequality is true for n > 3

and let us prove it for n + 1. This follows from the fact that

1 +1

n≤ 1 +

1

4<

3√

3 ⇒ 3n+13 =

3√

3 · 3n

3 ≥ n + 1

n· n = n + 1.

Thus, using this observation, we find that a1ai

i+1 > 313 ≥ a

1ai+1

i+1 ⇒ ai+1 > ai for all i,

which means that a1 < a2 < · · · < an−1 < an < a1, contradiction.

41. [ Mircea Lascu, Marian Tetiva ] Let x,y,z be positive real numbers whichsatisfy the condition

xy + xz + yz + 2xyz = 1.

Prove that the following inequalities hold

a) xyz ≤ 1

8;

b) x + y + z ≥ 3

2;

c)1

x+

1

y+

1

z≥ 4 (x + y + z);

d)1

x+

1

y+

1

z− 4 (x + y + z) ≥ (2z − 1)

2

(z (2z + 1)), where z = max {x,y,z}.

Solution :

a) We put t3 = xyz; according to the AM-GM Inequality we have

1 = xy + xz + yz + 2xyz ≥ 3t2 + 2t3 ⇔ (2t − 1) (t + 1)2 ≤ 0,

therefore 2t − 1 ≤ 0 ⇔ t ≤ 1

2, this meaning that xyz ≤ 1

8.

b) Denote also s = x + y + z; the following inequalities are well-known

(x + y + z)2 ≥ 3 (xy + xz + yz)

and

(x + y + z)3 ≥ 27xyz;

Page 52: 22573582 This Work Blends Together Classic Inequality Results With Brand New

8/8/2019 22573582 This Work Blends Together Classic Inequality Results With Brand New

http://slidepdf.com/reader/full/22573582-this-work-blends-together-classic-inequality-results-with-brand-new 52/127

52 Solutions

then we have 2s3 ≥ 54xyz = 27 − 27 (xy + xz + yz) ≥ 27 − 9s2, i. e.

2s3 + 9s2 − 27 ≥ 0 ⇔ (2s − 3) (s + 3)2 ≥ 0,

where from 2s − 3 ≥ 0 ⇔ s ≥ 3

2.

Or, because p ≤ 1

8, we have

s2 ≥ 3q = 3(1 − 2 p) ≥ 3

1 − 2

8

=

9

4;

if we put q = xy + xz + yz, p = xyz.Now, one can see the following is also true

q = xy + xz + yz ≥ 3

4.

c) The three numbers x,y,z cannot be all less than1

2, because, in this case we

get the contradiction

xy + xz + yz + 2xyz <3

4+ 2 · 1

8= 1;

because of symmetry we may assume then that z

1

2

.

We have 1 = (2z + 1) xy + z (x + y) ≥ (2z + 1) xy + 2z√xy, which can also

be written in the form

(2z + 1)√

xy − 1 √

xy + 1 ≤ 0; and this one yields the

inequality

xy ≤ 1

(2z + 1)2.

We also have 1 = (2z + 1) xy + z (x + y) ≤ (2z + 1)(x + y)2

4+ z (x + y), conse-

quently ((2z + 1) (x + y) − 2) (x + y + 2) ≥ 0, which shows us that

x + y ≥ 2

2z + 1.

The inequality to be proved

1

x+

1

y+

1

z≥ 4 (x + y + z)

can be also rearranged as

(x + y)

1

xy− 4

≥ 4z2 − 1

z=

(2z − 1)(2z + 1)

z.

From the above calculations we infer that

(x + y)

1

xy− 4

≥ 2

2z + 1 (2z + 1)2 − 4

=

2 (2z − 1)(2z + 3)

2z + 1

Page 53: 22573582 This Work Blends Together Classic Inequality Results With Brand New

8/8/2019 22573582 This Work Blends Together Classic Inequality Results With Brand New

http://slidepdf.com/reader/full/22573582-this-work-blends-together-classic-inequality-results-with-brand-new 53/127

Old and New Inequalities 53

(the assumption z ≥ 1

2allows the multiplication of the inequalities side by side), and

this means that the problem would be solved if we proved

2 (2z + 3)

2z + 1≥ 2z + 1

z⇔ 4z2 + 6z ≥ 4z2 + 4z + 1;

but this follows for z ≥ 1

2and we are done.

d) Of course, if  z is the greatest from the numbers x,y,z, then z ≥ 1

2; we saw

that

1x

+ 1y

− 4 (x + y) = (x + y)

1xy

− 4 ≥ 2

2z + 1(2z + 1)

=2 (2z − 1)(2z + 3)

2z + 1= 4z − 1

z+

(2z − 1)2

z (2z + 1),

where from we get our last inequality

1

x+

1

y+

1

z− 4 (x + y + z) ≥ (2z − 1)2

z (2z + 1).

Of course, in the right-hand side z could be replaced by any of the three numbers

which is ≥ 1

2(two such numbers could be, surely there is one).

Remark .It is easy to see that the given condition implies the existence of positive numbers

a,b,c such that x =a

b + c, y =

b

c + a, z =

c

a + b. And now a), b) and c) reduce

immediately to well-known inequalities! Try to prove using this substitution d).

42. [ Manlio Marangelli ] Prove that for any positive real numbers x,y,z,

3(x2y + y2z + z2x)(xy2 + yz2 + zx2) ≥ xyz(x + y + z)3.

Solution :

Using the AM-GM Inequality, we find that

1

3+

y2z

y2z + z2x + x2y+

xy2

yz2 + zx2 + xy2≥ 3y 3

√xyz

3 · (y2z + z2x + x2y) · (yz 2 + zx2 + xy2)

and two other similar relations

1

3+

z2x

y2z + z2x + x2y+

yz2

yz2 + zx2 + xy2≥ 3z 3

√xyz

3 · (y2z + z2x + x2y) · (yz 2 + zx2 + xy2)

1

3+

x2y

y2z + z2x + x2y+

zx2

yz2 + zx2 + xy2≥ 3x 3

√xyz

3 · (y2z + z2x + x2y) · (yz 2 + zx2 + xy2)

Then, adding up the three relations, we find exactly the desired inequality.

Page 54: 22573582 This Work Blends Together Classic Inequality Results With Brand New

8/8/2019 22573582 This Work Blends Together Classic Inequality Results With Brand New

http://slidepdf.com/reader/full/22573582-this-work-blends-together-classic-inequality-results-with-brand-new 54/127

54 Solutions

43. [ Gabriel Dospinescu ] Prove that if  a,b,c are real numbers such that

max{a,b,c} − min{a,b,c} ≤ 1, then

1 + a3 + b3 + c3 + 6abc ≥ 3a2b + 3b2c + 3c2a

Solution :

Clearly, we may assume that a = min{a,b,c} and let us write b = a + x, c = a + y,

where x, y ∈ [0, 1]. It is easy to see that a3 + b3 + c3 −3abc = 3a(x2 −xy + y2) + x3 + y3

and a2b + b2c + c2a − 3abc = a(x2 − xy + y2) + x2y. So, the inequality becomes

1 + x3 + y3 ≥ 3x2y. But this follows from the fact that 1 + x3 + y3 ≥ 3xy ≥ 3x2y,

because 0 ≤ x, y ≤ 1.

44. [ Gabriel Dospinescu ] Prove that for any positive real numbers a,b,c we have

27 +

2 +

a2

bc

2 +

b2

ca

2 +

c2

ab

≥ 6(a + b + c)

1

a+

1

b+

1

c

.

Solution :

By expanding the two sides, the inequality is equivalent to 2abc(a3 + b3 + c3 +

3abc − a2b − a2c − b2a − b2c − c2a − c2b) + (a3b3 + b3c3 + c3a3 + 3a2b2c2 − a3b2c −a3bc2 − ab3c2 − ab2c3 − a2b3c − a2bc3) ≥ 0. But this is true from Schur’s Inequality

applied for a,b,c and ab, bc, ca.

45. Let a0 =1

2and ak+1 = ak +

a2k

n. Prove that 1 − 1

n< an < 1.

TST Singapore

Solution :

We have1

ak+1=

1

ak− 1

ak + nand so

n−1k=0

1

ak− 1

ak+1

=

n−1k=0

1

n + ak< 1 ⇒ 2 − 1

an< 1 ⇒ an < 1.

Now, because the sequence is increasing we also have 2 −1

an =

n−1k=0

1

n + ak >

n

n + 1

and from this inequality and the previous one we conclude immediately that

1 − 1

n< an < 1.

46. [ Calin Popa ] Let a,b,c be positive real numbers, with a,b,c ∈ (0, 1) such

that ab + bc + ca = 1. Prove that

a

1 − a2+

b

1 − b2+

c

1 − c2≥ 3

4

1 − a2

a+

1 − b2

b+

1 − c2

c

.

Page 55: 22573582 This Work Blends Together Classic Inequality Results With Brand New

8/8/2019 22573582 This Work Blends Together Classic Inequality Results With Brand New

http://slidepdf.com/reader/full/22573582-this-work-blends-together-classic-inequality-results-with-brand-new 55/127

Old and New Inequalities 55

Solution :

It is known that in every triangle ABC  the following identity holdstan

A

2tan

B

2= 1 and because tan is bijective on

0,

π

2

we can set a = tan

A

2, b =

tanB

2, c = tan

2. The condition a,b,c ∈ (0, 1) tells us that the triangle ABC  is

acute-angled. With this substitution, the inequality becomes

2tanA

2

1 − tan2 A

2

≥ 3 1 − tan2 A

2

2tanA

2

tan A ≥ 3 1

tan A⇔

⇔ tan A tan B tan C (tan A+tan B+tan C ) ≥ 3(tan A tan B+tan B tan C +tan C tan A) ⇔⇔ (tan A + tan B + tan C )2 ≥ 3(tan A tan B + tan B tan C  + tan C tan A),

clearly true because tan A, tan B, tan C > 0.

47. [ Titu Andreescu, Gabriel Dospinescu ] Let x,y,z ≤ 1 and x + y + z = 1.

Prove that1

1 + x2+

1

1 + y2+

1

1 + z2≤ 27

10.

Solution :

Using the fact that (4 − 3t)(1 − 3t)2 ≥ 0 for any t ≤ 1, we find that

1

1 + x2≤ 27

50(2 − x).

Writing two similar expressions for y and z and adding them up, we find the desired

inequality.

Remark .

Tough it may seem too easy, this problem helps us to prove the following difficult

inequality

(b + c − a)2

a2 + (b + c)2≥ 3

5.

In fact, this problem is equivalent to that difficult one. Try to prove this!

48. [ Gabriel Dospinescu ] Prove that if √

x +√

y +√

z = 1, then

(1 − x)2(1 − y)2(1 − z)2 ≥ 215xyz(x + y)(y + z)(z + x)

.

Solution :

We put a =√

x, b =√

y and c =√

z. Then 1 − x = 1 − a2 = (a + b + c)2 − a2 =

(b + c)(2a + b + c). Now we have to prove that ((a + b)(b + c)(c + a)(2a + b + c)(a +

2b + c)(a + b + 2c))2 ≥ 215a2b2c2(a2 + b2)(b2 + c2)(c2 + a2). But this inequality is true

Page 56: 22573582 This Work Blends Together Classic Inequality Results With Brand New

8/8/2019 22573582 This Work Blends Together Classic Inequality Results With Brand New

http://slidepdf.com/reader/full/22573582-this-work-blends-together-classic-inequality-results-with-brand-new 56/127

56 Solutions

as it follows from the following true inequalities ab(a2 + b2) ≤ (a + b)4

8(this being

equivalent to (a−b)4 ≥ 0) and (2a+b+c)(a+2b+c)(a+b+2c) ≥ 8(b+c)(c+a)(a+b).

49. Let x,y,z be positive real numbers such that xyz = x + y + z + 2. Prove that

(1) xy + yz + zx ≥ 2(x + y + z);

(2)√

x +√

y +√

z ≤ 3

2

√xyz.

Solution :

The initial condition xyz = x + y + z + 2 can be rewritten as follows

1

1 + x+

1

1 + y+

1

1 + z= 1.

Now, let1

1 + x= a,

1

1 + y= b,

1

1 + z= c.

Then

x =1 − a

a=

b + c

a, y =

c + a

b, z =

a + b

c.

(1) We have

xy + yz + zx ≥ 2(x + y + z) ⇔ b + c

a· c + a

b+

c + a

b· a + b

c+

a + b

c· b + c

a≥

≥ 2

b + ca

+ ·c + ab

+ a + bc

⇔ a3 + b3 + c3 + 3abc ≥

≥ ab(a + b) + bc(b + c) + ca(c + a) ⇔

a(a − b)(a − c) ≥ 0,

which is exactly Schur’s Inequality.

(2) Here we have√

x +√

y +√

z ≤ 3

2

√xyz ⇔

⇔ 

a

b + c· b

c + a+

 b

c + a· c

a + b+

 c

a + b· a

b + c≤ 3

2.

This can be proved by adding the inequality

  a

b + c ·b

c + a ≤1

2 a

a + c +b

b + c

,

with the analogous ones.

50. Prove that if  x,y,z are real numbers such that x2 + y2 + z2 = 2, then

x + y + z ≤ xyz + 2.

IMO Shortlist, 1987

Page 57: 22573582 This Work Blends Together Classic Inequality Results With Brand New

8/8/2019 22573582 This Work Blends Together Classic Inequality Results With Brand New

http://slidepdf.com/reader/full/22573582-this-work-blends-together-classic-inequality-results-with-brand-new 57/127

Old and New Inequalities 57

First solution :

If one of  x,y,z is negative, let us say x then

2 + xyz − x − y − z = (2 − y − z) − x(1 − yz) ≥ 0,

because y + z ≤  2(y2 + z2) ≤ 2 and zy ≤ z2 + y2

2≤ 1. So we may assume that

0 < x ≤ y ≤ z. If  z ≤ 1 then

2 + xyz − x − y − z = (1 − z)(1 − xy) + (1 − x)(1 − y) ≥ 0.

Now, if  z > 1 we have

z + (x + y) ≤ 

2(z2 + (x + y)2) = 2 

1 + xy ≤ 2 + xy ≤ 2 + xyz.

This ends the proof.

Second solution :

Using the Cauchy-Schwarz Inequality, we find that

x + y + z − xyz = x(1 − yz) + y + z ≤ 

(x2 + (y + z)2) · (1 + (1 − yz)2).

So, it is enough to prove that this last quantity is at most 2, which is equivalent tothe inequality (2 + 2yz)(2 − 2yz + (yz)2) ≤ 4 ⇔ 2(yz)3 ≤ 2(yz)2, which is clearly

true, because 2 ≥ y2 + z2 ≥ 2yz.

51. [ Titu Andreescu, Gabriel Dospinescu ] Prove that for any x1, x2, . . . , xn ∈(0, 1) and for any permutation σ of the set {1, 2, . . . , n}, we have the inequality

ni=1

1

1 − xi≥

1 +

ni=1

xi

n

·

ni=1

1

1 − xi · xσ(i)

.

Solution :

Using the AM-GM Inequality and the fact that1

x + y≤ 1

4x+

1

4y, we can

write the following chain of inequalities

ni=1

1

1 − xi · yi≤

ni=1

1

1 − x2i + y2

i

2

= 2

ni=1

1

1 − x2i + 1 − y2

i

≤ni=1

1

2(1 − x2i )

+1

2(1 − y2i )

=

ni=1

1

1 − x2i

.

Page 58: 22573582 This Work Blends Together Classic Inequality Results With Brand New

8/8/2019 22573582 This Work Blends Together Classic Inequality Results With Brand New

http://slidepdf.com/reader/full/22573582-this-work-blends-together-classic-inequality-results-with-brand-new 58/127

58 Solutions

So, it is enough to prove the inequality

ni=1

1

1 − xi≥

1 +

ni=1

xi

n

·

ni=1

1

1 − x2i

⇔ni=1

xi

1 − x2i

≥ 1

ni=1

xi

·

ni=1

1

1 − x2i

,

which is Chebyshev Inequality for the systems (x1, x2, . . . , xn) and

1

1 − x21

,1

1 − x22

, . . . ,1

1 − x2n

.

52. Let x1, x2, . . . , xn be positive real numbers such thatni=1

1

1 + xi= 1. Prove

thatni=1

√xi ≥ (n − 1)

ni=1

1√xi

.

Vojtech Jarnik

First solution :Let

1

1 + xi= ai. The inequality becomes

ni=1

 1 − ai

ai≥ (n − 1)

ni=1

 ai

1 − ai⇔

ni=1

1 ai(1 − ai)

≥ nni=1

 ai

1 − ai⇔ n

ni=1

 ai

1 − ai≤

ni=1

ai

ni=1

1 ai(1 − ai)

.

But the last inequality is a consequence of  Chebyshev’s Inequality for the n-tuples

(a1, a2, . . . , an) and

a1(1 − a1), 1 

a2(1 − a2), . . . , 1 

an(1 − an)

.

Solution 2 :

With the same notations, we have to prove that

(n − 1)

ni=1

 ai

a1 + a2 + . . . + ai−1 + ai+1 + . . . + an≤

≤ni=1

 a1 + a2 + . . . + ai−1 + ai+1 + . . . + an

ai.

Page 59: 22573582 This Work Blends Together Classic Inequality Results With Brand New

8/8/2019 22573582 This Work Blends Together Classic Inequality Results With Brand New

http://slidepdf.com/reader/full/22573582-this-work-blends-together-classic-inequality-results-with-brand-new 59/127

Old and New Inequalities 59

But using the Cauchy-Schwarz Inequality and the AM-GM Inequality, we

deduce thatni=1

 a1 + a2 + . . . + ai−1 + ai+1 + . . . + an

ai≥

≥ni=1

√a1 +

√a2 + . . . +

√ai−1 +

√ai+1 + . . . +

√an√

n − 1 · √ai=

=n

i=1

√ai√

n − 1

1

√a

1

+1

√a

2

+ · · · +1

√ai−1

+1

√ai+1

+ · · · +1

√an ≥

≥ni=1

(n − 1)√

n − 1 · √ai√a1 +

√a2 + · · · +

√ai−1 +

√ai+1 + · · · +

√an

≥ni=1

(n − 1)

 ai

a1 + a2 + · · · + ai−1 + ai+1 + · · · + an

and we are done.

53. [ Titu Andreescu ] Let n > 3 and a1, a2, . . . , an be real numbers such that

a1 + a2 + . . . +an ≥ n and a21 + a2

2 + . . . +a2n ≥ n2. Prove that max{a1, a2, . . . , an} ≥ 2.

USAMO, 1999

Solution :

The most natural idea is to suppose that ai < 2 for all i and to substitute

xi = 2 − ai > 0. Then we haveni=1

(2 − xi) ≥ n ⇒ni=1

xi ≤ n and also

n2 ≤ni=1

a2i =

ni=1

(2 − xi)2 = 4n − 4

ni=1

xi +ni=1

x2i

Now, using the fact that xi > 0, we obtainn

i=1

x2i <

n

i=1

xi2

, which combined with

the above inequality yields

n2 < 4n − 4

ni=1

xi +

ni=1

xi

2

< 4n + (n − 4)

ni=1

xi

(we have used the fact thatni=1

xi ≤ n). Thus, we have (n − 4)

ni=1

xi − n

> 0,

which is clearly impossible since n ≥ 4 and

ni=1

xi ≤ n. So, our assumption was wrong

and consequently max{a1, a2, . . . , an} ≥ 2.

Page 60: 22573582 This Work Blends Together Classic Inequality Results With Brand New

8/8/2019 22573582 This Work Blends Together Classic Inequality Results With Brand New

http://slidepdf.com/reader/full/22573582-this-work-blends-together-classic-inequality-results-with-brand-new 60/127

60 Solutions

54. [ Vasile Cırtoaje ] If  a,b,c,d are positive real numbers, then

a − b

b + c+

b − c

c + d+

c − d

d + a+

d − a

a + b≥ 0.

Solution :

We have

a − b

b + c+

b − c

c + d+

c − d

d + a+

d − a

a + b=

a + c

b + c+

b + d

c + d+

c + a

d + a+

d + b

a + b− 4 =

= (a + c)

1b + c + 1d + a

+ (b + d)

1c + d + 1a + b − 4.

Since

1

b + c+

1

d + a≥ 4

(b + c) + (d + a),

1

c + d+

1

a + b≥ 4

(c + d) + (a + b),

we get

a − b

b + c+

b − c

c + d+

c − d

d + a+

d − a

a + b≥ 4(a + c)

(b + c) + (d + a)+

4(b + d)

(c + d) + (a + b)− 4 = 0.

Equality holds for a = c and b = d.

Conjecture (Vasile Cırtoaje)

If  a,b,c,d,e are positive real numbers, thena − b

b + c+

b − c

c + d+

c − d

d + e+

d − e

e + a+

e − a

a + b≥ 0.

55. If  x and y are positive real numbers, show that xy + yx > 1.

France, 1996

Solution :

We will prove that ab ≥ a

a + b − abfor any a, b ∈ (0, 1). Indeed, from Bernoulli

Inequality it follows that a1−b = (1 + a − 1)1−b ≤ 1 + (a − 1)(1 − b) = a + b − ab

and thus the conclusion. Now, it x or y is at least 1, we are done. Otherwise, let

0 < x, y < 1. In this case we apply the above observation and find that xy + yx ≥x

x + y − xy+

y

x + y − xy>

x

x + y+

y

x + y= 1.

56. Prove that if  a,b,c > 0 have product 1, then

(a + b)(b + c)(c + a) ≥ 4(a + b + c − 1).

MOSP, 2001

Page 61: 22573582 This Work Blends Together Classic Inequality Results With Brand New

8/8/2019 22573582 This Work Blends Together Classic Inequality Results With Brand New

http://slidepdf.com/reader/full/22573582-this-work-blends-together-classic-inequality-results-with-brand-new 61/127

Old and New Inequalities 61

First solution :

Using the identity (a + b)(b + c)(c + a) = (a + b + c)(ab + bc + ca) − 1 we reduce

the problem to the following one

ab + bc + ca +3

a + b + c≥ 4.

Now, we can apply the AM-GM Inequality in the following form

ab + bc + ca +3

a + b + c≥ 4 4

 (ab + bc + ca)3

9(a + b + c).

And so it is enough to prove that

(ab + bc + ca)3 ≥ 9(a + b + c).

But this is easy, because we clearly have ab + bc + ca ≥ 3 and (ab + bc + ca)2 ≥3abc(a + b + c) = 3(a + b + c).

Second solution :

We will use the fact that (a + b)(b + c)(c + a) ≥ 8

9(a + b + c)(ab + bc + ca).

So, it is enough to prove that2

9(ab + bc + ca) +

1

a + b + c≥ 1. Using the AM-GM

Inequality, we can write

2

9(ab + bc + ca) +

1

a + b + c≥ 3 3

 (ab + bc + ca)2

81(a + b + c)≥ 1,

because

(ab + bc + ca)2 ≥ 3abc(a + b + c) = 3(a + b + c).

57. Prove that for any a,b,c > 0,

(a2 + b2 + c2)(a + b − c)(b + c − a)(c + a − b) ≤ abc(ab + bc + ca).

Solution :

Clearly, if one of the factors in the left-hand side is negative, we are done. So,

we may assume that a,b,c are the side lenghts of a triangle ABC . With the usual

notations in a triangle, the inequality becomes

(a2+b2+c2)· 16K 2

a + b + c≤ abc(ab+bc+ca) ⇔ (a+b+c)(ab+bc+ca)R2 ≥ abc(a2+b2+c2).

But this follows from the fact that (a + b + c)(ab + bc + ca) ≥ 9abc and

0 ≤ OH 2 = 9R2 − a2 − b2 − c2.

Page 62: 22573582 This Work Blends Together Classic Inequality Results With Brand New

8/8/2019 22573582 This Work Blends Together Classic Inequality Results With Brand New

http://slidepdf.com/reader/full/22573582-this-work-blends-together-classic-inequality-results-with-brand-new 62/127

62 Solutions

58. [ D.P.Mavlo ] Let a,b,c > 0. Prove that

3 + a + b + c +1

a+

1

b+

1

c+

a

b+

b

c+

c

a≥ 3

(a + 1)(b + 1)(c + 1)

1 + abc.

Kvant, 1988

Solution :

The inequality is equivalent to the following one

a +

1

a + a

b ≥ 3 ab + a

abc + 1or

abc

a + 1

a+

a2c +

a

b≥ 2

a +

ab

.

But this follows from the inequalities

a2bc +b

c≥ 2ab,b2ca +

c

a≥ 2bc,c2ab +

a

b≥ 2ca

and

a2c +1

c≥ 2a, b2a +

1

a≥ 2b, c2b +

1

b≥ 2c.

59. [ Gabriel Dospinescu ] Prove that for any positive real numbers x1, x2, . . . , xn

with product 1 we have the inequality

nn ·ni=1

(xni + 1) ≥

ni=1

xi +ni=1

1

xi

n

.

Solution :

Using the AM-GM Inequality, we deduce that

xn11 + xn1

+xn2

1 + xn2+ · · · +

xnn−1

1 + xnn−1

+1

1 + xnn≥ n

xn · n  n

i=1

(1 + xni )

and1

1 + xn1+

1

1 + xn2+ · · · +

1

1 + xnn−1

+xnn

1 + xnn≥ n · xn

n

  ni=1

(1 + xni )

Thus, we have

n

 

ni=1

(1 + xni ) ≥ xn +1

xn.

Page 63: 22573582 This Work Blends Together Classic Inequality Results With Brand New

8/8/2019 22573582 This Work Blends Together Classic Inequality Results With Brand New

http://slidepdf.com/reader/full/22573582-this-work-blends-together-classic-inequality-results-with-brand-new 63/127

Old and New Inequalities 63

Of course, this is true for any other variable, so we can add all these inequalities to

obtain that

n n

  ni=1

(1 + xni ) ≥

ni=1

xi +ni=1

1

xi

which is the desired inequality.

60. Let a, b, c, d > 0 such that a + b + c = 1. Prove that

a3 + b3 + c3 + abcd≥

min 1

4,

1

9+

d

27 .

Kvant, 1993

Solution :

Suppose the inequality is false. Then we have, taking into account that abc ≤ 1

27,

the inequality d

1

27− abc

> a3 + b3 + c3 − 1

9. We may assume that abc <

1

27. Now,

we will reach a contradiction proving that a3 + b3 + c3 + abcd ≥ 1

4. It is sufficient to

prove that

a3 + b3 + c3

1

91

27− abc

abc + a3 + b3 + c3 ≥1

4 .

But this inequality is equivalent to 4

a3 + 15abc ≥ 1. We use now the identitya3 = 3abc+1−3

ab and reduce the problem to proving that

ab ≤ 1 + 9abc

4,

which is Schur’s Inequality.

61. Prove that for any real numbers a,b,c we have the inequality(1 + a2)2(1 + b2)2(a − c)2(b − c)2 ≥ (1 + a2)(1+ b2)(1+ c2)(a − b)2(b − c)2(c − a)2.

AMM

Solution :

The inequality can be also written as (1 + a2)(1 + b2)

(1 + c2)(a − b)2≥ 1 (of course, we may

assume that a,b,c are distinct). Now, adding the inequalities

(1 + a2)(1 + b2)

(1 + c2)(a − b)2+

(1 + b2)(1 + c2)

(1 + a2)(b − c)2≥ 2

1 + b2

|a − b||c − b|(which can be found using the AM-GM Inequality) we deduce that

(1 + a2)(1 + b2)

(1 + c2)(a − b)2≥

1 + b2

|(b − a)(b − c)|

Page 64: 22573582 This Work Blends Together Classic Inequality Results With Brand New

8/8/2019 22573582 This Work Blends Together Classic Inequality Results With Brand New

http://slidepdf.com/reader/full/22573582-this-work-blends-together-classic-inequality-results-with-brand-new 64/127

64 Solutions

and so it is enough to prove that the last quantity is at least 1. But it follows from 1 + b2

|(b − a)(b − c)| ≥ 1 + b2

(b − a)(b − c)

= 1

and the problem is solved.

62. [ Titu Andreescu, Mircea Lascu ] Let α,x,y,z be positive real numbers such

that xyz = 1 and α ≥ 1. Prove that

y + z+

z + x+

x + y≥ 3

2.

First solution :

We may of course assume that x ≥ y ≥ z. Then we have

x

y + z≥ y

z + x≥ z

x + y

and xα−1 ≥ yα−1 ≥ zα−1. Using Chebyshev’s Inequality we infer that xα

y + z≥ 1

xα−1

· x

y + z

.

Now, all we have to do is to observe that this follows from the inequalities xα−1 ≥ 3

(from the AM-GM Inequality) and

xy + z

≥ 32

Second solution :

According to the Cauchy-Schwarz Inequality, we have:

[(x(y +z)+y(z +x)+z(x+y)]]

y + z+

z + x+

x + y

x1+α2 + y

1+α2 + z

1+α2

2.

Thus it remains to show thatx1+α2 + y

1+α2 + z

1+α2

2≥ 3(xy + yz + zx).

Since (x + y + z)2 ≥ 3(xy + yz + zx), it is enough to prove that

x1+α2 + y

1+α2 + z

1+α2 ≥ x + y + z.

From Bernoulli’s Inequality, we get

x1+α2 = [1 + (x − 1)]

1+α2 ≥ 1 +

1 + α

2(x − 1) =

1 − α

2+

1 + α

2x

and, similarly,

y1+α2 ≥ 1 − α

2+

1 + α

2y, z

1+α2 ≥ 1 − α

2+

1 + α

2z.

Thus

x1+α2 + y

1+α2 + z

1+α2 − (x + y + z) ≥ 3(1 − α)

2+

1 + α

2(x + y + z) − (x + y + z) =

Page 65: 22573582 This Work Blends Together Classic Inequality Results With Brand New

8/8/2019 22573582 This Work Blends Together Classic Inequality Results With Brand New

http://slidepdf.com/reader/full/22573582-this-work-blends-together-classic-inequality-results-with-brand-new 65/127

Old and New Inequalities 65

α − 1

2(x + y + z − 3) ≥ α − 1

2(3 3

√xyz − 3) = 0.

Equality holds for x = y = z = 1.

Remark :

Using the substitution β  = α + 1 (β ≥ 2) and x =1

a, y =

1

b, z =

1

c(abc = 1) the

inequality becomes as follows

1

aβ(b + c)+

1

bβ(c + a)+

1

cγ(a + b)≥ 3

2.

For β  = 3, we obtain one of the problems of IMO 1995 (proposed by Russia).

63. Prove that for any real numbers x1, . . . , xn, y1, . . . , yn such that x21+· · ·+x2

n =

y21 + · · · + y2

n = 1,

(x1y2 − x2y1)2 ≤ 2

1 −

nk=1

xkyk

.

Korea, 2001

Solution :

We clearly have the inequality

(x1y2

−x2y1)2

≤ 1≤i<j≤n

(xiyj−

xjyi)2 =

n

i=1

x2i

n

i=1

y2i −

n

i=1

xiyi2

=

=

1 −

ni=1

xiyi

1 +

ni=1

xiyi

.

Because we also have

ni=1

xiyi

≤ 1, we find immediately that

1 −

ni=1

xiyi

1 +

ni=1

xiyi

≤ 2

1 −

ni=1

xiyi

and the problem is solved.

64. [ Laurentiu Panaitopol ] Let a1, a2, . . . , an be pairwise distinct positive inte-

gers. Prove that

a21 + a2

2 + · · · + a2n ≥ 2n + 1

3(a1 + a2 + · · · + an).

TST Romania

Page 66: 22573582 This Work Blends Together Classic Inequality Results With Brand New

8/8/2019 22573582 This Work Blends Together Classic Inequality Results With Brand New

http://slidepdf.com/reader/full/22573582-this-work-blends-together-classic-inequality-results-with-brand-new 66/127

66 Solutions

Solution :

Without loss of generality, we may assume that a1 < a2 < · · · < an and hence

ai ≥ i for all i. Thus, we may take bi = ai − i ≥ 0 and the inequality becomes

ni=1

b2i + 2

ni=1

ibi +n(n + 1)(2n + 1)

6≥ 2n + 1

ni=1

bi +n(n + 1)(2n + 1)

6.

Now, using the fact that ai+1 > ai we infer that b1 ≤ b2 ≤ ·· · ≤ bn and from

Chebyshev’s Inequality we deduce that

2

ni=1

ibi ≥ (n + 1)

ni=1

bi ≥2n + 1

3

ni=1

bi

and the conclusion is immediate. Also, from the above relations we can see imme-

diately that we have equality if and only if  a1, a2, . . . , an is a permutation of the

numbers 1, 2, . . . , n.

65. [ Calin Popa ] Let a,b,c be positive real numbers such that a + b + c = 1.

Prove that

b√

c

a(√

3c +√

ab)+

c√

a

b(√

3a +√

bc)+

a√

b

c(√

3b +√

ca)≥ 3

√3

4.

Solution :

Rewrite the inequality in the form

 

bc

a 3ca

b+ a

≥ 3√

3

4.

With the substitution x =

 bc

a, y =

 ca

b, z =

 ab

cthe condition a + b + c = 1

becomes xy + yz + zx = 1 and the inequality turns into

x√3y + yz

≥ 3√3

4.

But, by applying the Cauchy-Schwarz Inequality we obtain

x2

√3xy + xyz

x2

√3 + 3xyz

≥3

xy

√3 +

1√3

=3√

3

4,

where we used the inequalities

x

2

≥ 3

xy

and xyz ≤ 1

3√

3.

Page 67: 22573582 This Work Blends Together Classic Inequality Results With Brand New

8/8/2019 22573582 This Work Blends Together Classic Inequality Results With Brand New

http://slidepdf.com/reader/full/22573582-this-work-blends-together-classic-inequality-results-with-brand-new 67/127

Old and New Inequalities 67

66. [ Titu Andreescu, Gabriel Dospinescu ] Let a,b,c,d be real numbers such that

(1 + a2)(1 + b2)(1 + c2)(1 + d2) = 16. Prove that

−3 ≤ ab + bc + cd + da + ac + bd − abcd ≤ 5.

Solution :

Let us write the condition in the form 16 =

(i + a) ·

(a− i). Using symmetric

sums, we can write this as follows

16 = 1−

i a− ab + i abc + abcd1 + i a

− ab−

i abc + abcd .

So, we have the identity 16 = (1 −ab + abcd)2 + (

a −

abc)2. This means that

|1 − ab + abcd| ≤ 4 and from here the conclusion follows.

67. Prove that

(a2 + 2)(b2 + 2)(c2 + 2) ≥ 9(ab + bc + ca)

for any positive real numbers a,b,c.

APMO, 2004

First solution :

We will prove even more: (a2

+ 2)(b2

+ 2)(c2

+ 2) ≥ 3(a + b + c)2

. Because(a + b + c)2 ≤ (|a| + |b| + |c|)2, we may assume that a,b,c are nonnegative. We will

use the fact that if  x and y have the same sign then (1 + x)(1 + y) ≥ 1 + x + y. So,

we write the inequality in the forma2 − 1

3+ 1

≥ (a + b + c)2

9

and we have three cases

i) If  a,b,c are at least 1, then

a2 − 1

3+ 1

≥ 1 +

a2 − 1

3≥

a2

9.

ii) If two of the three numbers are at least 1, let them be a and b, then we have

a2 − 1

3+ 1

1 +

a2 − 1

3+

b2 − 1

3

c2 + 2

3

=(a2 + b2 + 1)

9

(12 + 12 + c2)

9≥ (a + b + c)2

9

by the Cauchy-Schwarz Inequality.

iii) If all three numbers are at most 1, then by Bernoulli Inequality we have

a2 − 1

3+ 1

≥ 1 +

a2 − 1

3≥

a2

9

and the proof is complete.

Page 68: 22573582 This Work Blends Together Classic Inequality Results With Brand New

8/8/2019 22573582 This Work Blends Together Classic Inequality Results With Brand New

http://slidepdf.com/reader/full/22573582-this-work-blends-together-classic-inequality-results-with-brand-new 68/127

68 Solutions

Second solution :

Expanding everything, we reduce the problem to proving that

(abc)2 + 2

a2b2 + 4

a2 + 8 ≥ 9

ab.

Because 3

a2 ≥ 3

ab and 2

a2b2 + 6 ≥ 4

ab, we are left with the

inequality (abc)2 +

a2 + 2 ≥ 2

ab. Of course, we can assume that a,b,c are

non-negative and we can write a = x2, b = y2, c = z2. In this case

2

ab −

a2 = (x + y + z)(x + y − z)(y + z − x)(z + x − y).

It is clear that if  x,y,z are not side lengths of a triangle, then the inequalityis trivial. Otherwise, we can take x = u + v, y = v + w, z = w + u and reduce the

inequality to

((u + v)(v + w)(w + u))4 + 2 ≥ 16(u + v + w)uvw.

We have ((u + v)(v + w)(w + u))4

+ 1 + 1 ≥ 3 3 

(u + v)4(v + w)4(u + w)4 and

it remains to prove that the last quantity is at least 16(u + v + w)uvw. This comes

down to

(u + v)4(v + w)4(w + u)4 ≥ 163

33(uvw)3(u + v + w)3.

But this follows from the known inequalities

(u + v)(v + w)(w + u) ≥ 89 (u + v + w)(uv + vw + wu),

(uv + vw + wu)4 ≥ 34(uvw)83 , u + v + w ≥ 3 3

√uvw.

Third solution :

In the same manner as in the Second solution, we reduce the problem to proving

that

(abc)2 + 2 ≥ 2

ab −

a2.

Now, using Schur’s Inequality, we infer that

2 ab

− a2

9abc

a + b + c

and as an immediate consequence of the AM-GM Inequality we have

9abc

a + b + c≤ 3 3

 (abc)2.

This shows that as soon as we prove that

(abc)2 + 2 ≥ 3 3 

(abc)2,

the problem is solved. But the last assertion follows from the AM-GM Inequality.

Page 69: 22573582 This Work Blends Together Classic Inequality Results With Brand New

8/8/2019 22573582 This Work Blends Together Classic Inequality Results With Brand New

http://slidepdf.com/reader/full/22573582-this-work-blends-together-classic-inequality-results-with-brand-new 69/127

Old and New Inequalities 69

68. [ Vasile Cırtoa je ] Prove that if 0 < x ≤ y ≤ z and x + y + z = xyz + 2, then

a) (1 − xy)(1 − yz)(1 − xz) ≥ 0;

b) x2y ≤ 1, x3y2 ≤ 32

27.

Solution :

a) We have

(1 − xy)(1 − yz) = 1 − xy − yz + xy2z = 1 − xy − yz + y(x + y + z − 2) = (y − 1)2 ≥ 0

and similarly

(1 − yz)(1 − zx) = (1 − z)2 ≥ 0, (1 − zx)(1 − xy) = (1 − x)2 ≥ 0.

So the expressions 1 − xy, 1 − yz and 1 − zx have the same sign.

b) We rewrite the relation x+y +z = xyz +2 as (1−x)(1−y)+(1−z)(1−xy) = 0.

If  x > 1 then z ≥ y ≥ x > 1 and so (1 − x)(1 − y) + (1 − z)(1 − xy) > 0, impossible.

So we have x ≤ 1. Next we distinguish two cases 1) xy ≤ 1; 2) xy > 1.

1) xy ≤ 1. We have x2y ≤ x ≤ 1 and x3y2 ≤ x ≤ 1 <32

27.

2) xy > 1. From y ≥ √xy we get y > 1. Next we rewrite the relation x + y + z =

xyz + 2 as x + y − 2 = (xy − 1)z. Because z ≥ y gives x + y − 2 ≥ (xy − 1)y,

(y − 1)(2 − x − xy) ≥ 0 so 2 ≥ x(1 + y). Using the AM-GM Inequality, we have

1 + y ≥ 2√y and 1 + y = 1 + y2

+ y2

≥ 3 3 

1 · y2

· y2

. Thus we have 2 ≥ 2x√y and

2 ≥ 3x3

 y2

4, which means that x2y ≤ 1 and x3y2 ≤ 32

27.

The equality x2y = 1 takes place when x = y = 1 and the equality x3y2 =32

27

takes place when x =2

3, y = z = 2.

69. [ Titu Andreescu ] Let a,b,c be positive real numbers such that a+b+c ≥ abc.

Prove that at least two of the inequalities

2

a+

3

b+

6

c ≥6,

2

b+

3

c+

6

a ≥6,

2

c+

3

a+

6

b ≥6,

are true.

TST 2001, USA

Solution :

The most natural idea is to male the substitution1

a= x,

1

b= y,

1

c= z. Thus, we

have x, y,z > 0 and xy + yz + zx ≥ 1 and we have to prove that at least two of the

inequalities 2x+3y+6z ≥ 6, 2y+3z +6x ≥ 6, 2z +3x+6y ≥ 6 are true. Suppose this is

not the case. Then we may assume that 2x+3y +6z < 6 and 2z +3x+6y < 6. Adding,

we find that 5x+ 9y +8z < 12. But we have x ≥ 1 − yz

y + z. Thus, 12 >

5 − 5yz

y + z+9y +8z

Page 70: 22573582 This Work Blends Together Classic Inequality Results With Brand New

8/8/2019 22573582 This Work Blends Together Classic Inequality Results With Brand New

http://slidepdf.com/reader/full/22573582-this-work-blends-together-classic-inequality-results-with-brand-new 70/127

70 Solutions

which is the same as 12(y + z) > 5 + 9y2 + 8z2 + 12yz ⇔ (2z −1)2 + (3y + 2z −2)2 < 0,

which is clearly impossible. Thus, the conclusion follows.

70. [ Gabriel Dospinescu, Marian Tetiva ] Let x, y,z > 0 such that

x + y + z = xyz.

Prove that

(x − 1) (y − 1) (z − 1) ≤ 6√

3 − 10.

First solution :

Because of  x < xyz ⇒ yz > 1 (and the similar relations xz > 1, xy > 1) at most

one of the three numbers can be less than 1. In any of these cases ( x ≤ 1, y ≥ 1,

z ≥ 1 or the similar ones) the inequality to prove is clear. The only case we still have

to analyse is that when x ≥ 1, y ≥ 1 and z ≥ 1.

In this situation denote

x − 1 = a, y − 1 = b, z − 1 = c.

Then a,b,c are nonnegative real numbers and, because

x = a + 1, y = b + 1, z = c + 1,

they satisfy

a + 1 + b + 1 + c + 1 = (a + 1) (b + 1) (c + 1) ,

which means

abc + ab + ac + bc = 2.

Now let x = 3√

abc; we have

ab + ac + bc ≥ 33√

abacbc = 3x2,

that’s why we get

x3 + 3x2 ≤ 2 ⇔ (x + 1)

x2 + 2x − 2

≤ 0 ⇔

⇔(x + 1) x + 1 +

√3 x + 1

−√

3 ≤0.

For x ≥ 0, this yields3√

abc = x ≤√

3 − 1,

or, equivalently

abc ≤√

3 − 13

,

which is exactly

(x − 1) (y − 1) (z − 1) ≤ 6√

3 − 10.

The proof is complete.

Page 71: 22573582 This Work Blends Together Classic Inequality Results With Brand New

8/8/2019 22573582 This Work Blends Together Classic Inequality Results With Brand New

http://slidepdf.com/reader/full/22573582-this-work-blends-together-classic-inequality-results-with-brand-new 71/127

Old and New Inequalities 71

Second solution :

Like in the first solution (and due to the symmetry) we may suppose that x ≥ 1,

y ≥ 1; we can even assume that x > 1, y > 1 (for x = 1 the inequality is plain). Then

we get xy > 1 and from the given condition we have

z =x + y

xy − 1.

The relation to prove is

(x − 1) (y − 1) (z − 1) ≤ 6√

3 − 10 ⇔

⇔2xyz

−(xy + xz + yz)

≤6√

3−

9,

or, with this expression of  z,

2xyx + y

xy − 1− xy − (x + y)

x + y

xy − 1≤ 6

√3 − 9 ⇔

⇔ (xy − x − y)2 +

6√

3 − 10

xy ≤ 6√

3 − 9,

after some calculations.

Now, we put x = a + 1, y = b + 1 and transform this into

a2b2 +

6√

3 − 10

(a + b + ab) − 2ab ≥ 0.

But

a + b ≥ 2√

ab

and 6√3 − 10 > 0, so it suffices to show that

a2b2 +

6√

3 − 10

2√

ab + ab

− 2ab ≥ 0.

The substitution t =√

ab ≥ 0 reduces this inequality to

t4 +

6√

3 − 12

t2 + 2

6√

3 − 10

t ≥ 0,

or

t3 +

6√

3 − 12

t + 2

6√

3 − 10

≥ 0.

The derivative of the function

f (t) = t3 + 6√

3 − 12 t + 2 6√

3 − 10 , t ≥ 0

is

(t) = 3

t2 −

√3 − 1

2

and has only one positive zero. It is√

3−1 and it’s easy to see that this is a minimum

point for f  in the interval [0, ∞). Consequently

f (t) ≥ f √

3 − 1

= 0,

and we are done.

A final observation: in fact we have

f (t) = t −√

3 + 12

t + 2√

3 − 2 ,

Page 72: 22573582 This Work Blends Together Classic Inequality Results With Brand New

8/8/2019 22573582 This Work Blends Together Classic Inequality Results With Brand New

http://slidepdf.com/reader/full/22573582-this-work-blends-together-classic-inequality-results-with-brand-new 72/127

72 Solutions

which shows that f (t) ≥ 0 for t ≥ 0.

71. [ Marian Tetiva ] Prove that for any positive real numbers a,b,c,a3 − b3

a + b+

b3 − c3

b + c+

c3 − a3

c + a

≤ (a − b)2 + (b − c)2 + (c − a)2

4.

Moldova TST, 2004

First solution :

First of all, we observe that right hand side can be transformed intoa3 − b3

a + b= (a3 − b3)

1

a + b− 1

a + c

+ (b3 − c3)

1

b + c− 1

a + c

=

=(a − b)(c − b)(a − c)

ab

(a + b)(a + c)(b + c)

and so we have to prove the inequality

|(a − b)(b − c)(c − a)|(ab + bc + ca)

(a + b)(b + c)(c + a)≤ 1

2

a2 −

ab

.

It is also easy to prove that (a + b)(b + c)(c + a) ≥ 8

9(a + b + c)(ab + bc + ca) and

so we are left with

2

9 · a · (a − b)2 ≥ (a − b)

.

Using the AM-GM Inequality, we reduce this inequality to the following one

8

27

a3

≥(a − b)

.

This one is easy. Just observe that we can assume that a ≥ b ≥ c and in this case it

becomes

(a − b)(a − c)(b − c) ≤ 8

27(a + b + c)3

and it follows from the AM-GM Inequality.

Second solution (by Marian Tetiva):

It is easy to see that the inequality is not only cyclic, but symmetric. That is whywe may assume that a ≥ b ≥ c > 0. The idea is to use the inequality

x +y

2≥ x2 + xy + y2

x + y≥ y +

x

2,

which is true if  x ≥ y > 0. The proof of this inequality is easy and we won’t insist.

Now, because a ≥ b ≥ c > 0, we have the three inequalities

a +b

2≥ a2 + ab + b2

a + b≥ b +

a

2, b +

c

2≥ b2 + bc + c2

b + c≥ c +

b

2

and of course

a +c

2≥ a2 + ac + c2

a + c≥ c +

a

2.

Page 73: 22573582 This Work Blends Together Classic Inequality Results With Brand New

8/8/2019 22573582 This Work Blends Together Classic Inequality Results With Brand New

http://slidepdf.com/reader/full/22573582-this-work-blends-together-classic-inequality-results-with-brand-new 73/127

Old and New Inequalities 73

That is why we can write a3 − b3

a + b= (a − b)

a2 + ab + b2

a + b+ (b − c)

b2 + bc + c2

b + c− (a − c)

a2 + ac + c2

a + c≥

≥ (a − b)

b +a

2

+ (b − c)

c +

b

2

− (a − c)

a +

c

2

=

= −

(a − b)2

4.

In the same manner we can prove that

a3

−b3

a + b ≤ (a−

b)2

4

and the conclusion follows.

72. [ Titu Andreescu ] Let a,b,c be positive real numbers. Prove that

(a5 − a2 + 3)(b5 − b2 + 3)(c5 − c2 + 3) ≥ (a + b + c)3.

USAMO, 2004

Solution :

We start with the inequality a5 − a2 + 3 ≥ a3 + 2 ⇔ (a2 − 1)(a3 − 1) ≥ 0. Thus,

it remains to show that a3 + 2

≥ a3

.

Using the AM-GM Inequality, one has

a3

a3 + 2+

1

b3 + 2+

1

c3 + 2≥ 3a

3

 (a3 + 2)

.

We write two similar inequalities and then add up all these relations. We will find

that (a3 + 2) ≥

a3

,

which is what we wanted.

73. [ Gabriel Dospinescu ] Let n > 2 and x1

, x2

, . . . , xn

> 0 such thatn

k=1

xk

n

k=1

1

xk

= n2 + 1.

Prove that n

k=1

x2k

·

nk=1

1

x2k

> n2 + 4 +

2

n(n − 1).

Page 74: 22573582 This Work Blends Together Classic Inequality Results With Brand New

8/8/2019 22573582 This Work Blends Together Classic Inequality Results With Brand New

http://slidepdf.com/reader/full/22573582-this-work-blends-together-classic-inequality-results-with-brand-new 74/127

74 Solutions

Solution :

In this problem, a combination between identities and the Cauchy-Schwarz

Inequality is the way to proceed. So, let us start with the expression

1≤i<j≤n

xi

xj+

xj

xi− 2

2

.

We can immediately see that

1≤i<j≤n

xixj

+xjxi

− 22

= 1≤i<j≤n

x2i

x2

j

+x2j

x2

i

− 4 · xixj

− 4 · xjxi

+ 6 =

=

ni=1

x2i

ni=1

1

x2i

− 4

ni=1

xi

ni=1

1

xi

+ 3n2.

Thus we could find from the inequality

1≤i<j≤n

xixj

+xjxi

− 2

2

≥ 0

that n

i=1

x2i

n

i=1

1

x2i

≥ n2 + 4.

Unfortunately, this is not enough. So, let us try to minimize1≤i<j≤n

xi

xj+

xj

xi− 2

2

.

This could be done using the Cauchy-Schwarz Inequality:

1≤i<j≤n

xixj

+xjxi

− 2

2

1≤i<j≤n

xixj

+xjxi

− 2

2

n

2

.

Because

i≤i<j≤n

xixj

+ xjxi

− 2

= 1, we deduce that

ni=1

x2i

ni=1

1

x2

≥ n2 + 4 +

2

n(n − 1),

which is what we wanted to prove. Of course, we should prove that we cannot have

equality. But equality would imply that x1 = x2 = · · · = xn, which contradicts the

assumption n

k=1

xk

n

k=1

1

xk

= n2 + 1.

Page 75: 22573582 This Work Blends Together Classic Inequality Results With Brand New

8/8/2019 22573582 This Work Blends Together Classic Inequality Results With Brand New

http://slidepdf.com/reader/full/22573582-this-work-blends-together-classic-inequality-results-with-brand-new 75/127

Old and New Inequalities 75

74. [ Gabriel Dospinescu, Mircea Lascu, Marian Tetiva ] Prove that for any po-

sitive real numbers a,b,c,

a2 + b2 + c2 + 2abc + 3 ≥ (1 + a)(1 + b)(1 + c).

First solution :

Let f (a,b,c) = a2 + b2 + c2 + abc + 2 − a − b − c − ab − bc − ca. We have to prove

that all values of f  are nonnegative. If  a,b,c > 3, then we clearly have1

a+

1

b+

1

c< 1,

which means that f (a,b,c) > a2 + b2 + c2 + 2

−a

−b

−c > 0. So, we may assume

that a ≤ 3 and let m = b + c2

. Easy computations show that f (a,b,c) − f (a,m,m) =

(3 − a)(b − c)2

4≥ 0 and so it remains to prove that f (a,m,m) ≥ 0, which is the same

as

(a + 1)m2 − 2(a + 1)m + a2 − a + 2 ≥ 0.

This is clearly true, because the discriminant of the quadratic equation is −4(a +

1)(a − 1)2 ≤ 0.

Second solution :

Recall Turkevici’s Inequality

x4 + y4 + z4 + t4 + 2xyzt ≥ x2y2 + y2z2 + z2t2 + t2x2 + x2z2 + y2t2

for all positive real numbers x,y,z,t. Taking t = 1, a = x2, b = y2, x = z2 and using

the fact that 2√

abc ≤ abc + 1, we find the desired inequality.

75. [ Titu Andreescu, Zuming Feng ] Let a,b,c be positive real numbers. Prove

that(2a + b + c)2

2a2 + (b + c)2+

(2b + a + c)2

2b2 + (a + c)2+

(2c + a + b)2

2c2 + (a + b)2≤ 8.

USAMO, 2003

First solution :

Because the inequality is homogeneous, we can assume that a + b + c = 3. Then(2a + b + c)2

2a2 + (b + c)2=

a2 + 6a + 9

3a2 − 6a + 9=

1

3

1 + 2 · 4a + 3

2 + (a − 1)2

≤ 1

3

1 + 2 · 4a + 3

2

=

4a + 4

3.

Thus (2a + b + c)2

2a2 + (b + c)2≤ 1

3

(4a + 4) = 8.

Page 76: 22573582 This Work Blends Together Classic Inequality Results With Brand New

8/8/2019 22573582 This Work Blends Together Classic Inequality Results With Brand New

http://slidepdf.com/reader/full/22573582-this-work-blends-together-classic-inequality-results-with-brand-new 76/127

76 Solutions

Second solution :

Denote x =b + c

a, y =

c + a

b, z =

a + b

c. We have to prove that

(x + 2)2

x2 + 2≤ 8 ⇔

2x + 1

x2 + 2≤ 5

2⇔

(x − 1)2

x2 + 2≥ 1

2.

But, from the Cauchy-Schwarz Inequality, we have (x − 1)2

x2 + 2≥ (x + y + z − 3)2

x2 + y2 + z2 + 6.

It remains to prove that

2(x2 + y2 + z2 + 2xy + 2yz + 2zx − 6x − 6y − 6z + 9) ≥ x2 + y2 + z2 + 6 ⇔⇔ x2 + y2 + z2 + 4(xy + yz + zx) − 12(x + y + z) + 12 ≥ 0.

Now xy + yz + zx ≥ 3 3 

x2y2z2 ≥ 12 (because xyz ≥ 8), so we still have to prove that

(x + y + z)2 + 24 − 12(x + y + z) + 1 2 ≥ 0, which is equivalent to (x + y + z − 6)2 ≥ 0,

clearly true.

76. Prove that for any positive real numbers x, y and any positive integers m,n,

(n−1)(m−1)(xm+n+ym+n)+ (m+n−1)(xmyn+xnym) ≥ mn(xm+n−1y+ym+n−1x).

Austrian-Polish Competition, 1995

Solution :

We transform the inequality as follows:

mn(x − y)(xm+n−1 − ym+n−1) ≥ (m + n − 1)(xm − ym)(xn − yn) ⇔

⇔ xm+n−1 − ym+n−1

(m + n − 1)(x − y)≥ xm − ym

m(x − y)· xn − yn

n(x − y)

(we have assumed that x > y). The last relation can also be written

(x − y)

 xy

tm+n−2dt ≥ xy

tm−1dt · xy

tn−1dt

and this follows from Chebyshev’s Inequality for integrals.

77. Let a,b,c,d,e be positive real numbers such that abcde = 1. Prove that

a + abc

1 + ab + abcd+

b + bcd

1 + bc + bcde+

c + cde

1 + cd + cdea+

d + dea

1 + de + deab+

e + eab

1 + ea + eabc≥ 10

3.

Crux Mathematicorum

Solution :

We consider the standard substitution

a =x

y, b =

y

z, c =

z

t, d =

t

u, e =

u

x

Page 77: 22573582 This Work Blends Together Classic Inequality Results With Brand New

8/8/2019 22573582 This Work Blends Together Classic Inequality Results With Brand New

http://slidepdf.com/reader/full/22573582-this-work-blends-together-classic-inequality-results-with-brand-new 77/127

Old and New Inequalities 77

with x,y, z,t,u > 0. It is clear that

a + abc

1 + ab + abcd=

1

y+

1

t1

x+

1

z+

1

u

.

Writing the other relations as well, and denoting1

x= a1,

1

y= a2,

1

z= a3,

1

t= a4,

1

u=

a5, we have to prove that if  ai > 0, then

a2 + a4

a1 + a3 + a5≥ 1

3.

Using the Cauchy-Schwarz Inequality, we minor the left-hand side with

4S 2

2S 2 − (a2 + a4)2 − (a1 + a4)2 − (a3 + a5)2 − (a2 + a5)2 − (a1 + a3)2,

where S  =5

i=1

ai. By applying the Cauchy-Schwarz Inequality again for the

denominator of the fraction, we obtain the conclusion.

78. [ Titu Andreescu ] Prove that for any a,b,c, ∈

0,π

2

the following inequality

holds

sin a

·sin(a

−b)

·sin(a

−c)

sin(b + c) +

sin b

·sin(b

−c)

·sin(b

−a)

sin(c + a) +

sin c

·sin(c

−a)

·sin(c

−b)

sin(a + b) ≥ 0.

TST 2003, USA

Solution :

Let x = sin a, y = sin b, z = sin c. Then we have x,y, z > 0. It is easy to see that

the following relations are true:

sin a · sin(a − b) · sin(a − c) · sin(a + b) · sin(a + c) = x(x2 − y2)(x2 − z2)

Using similar relations for the other terms, we have to prove that:x(x2 − y2)(x2 − y2) ≥ 0.

With the substitution x = √u, y = √v, z = √w the inequality becomes√u(u −

v)(u − w) ≥ 0. But this follows from Schur’s Inequality.

79. Prove that if  a,b,c are positive real numbers then, a4 + b4 + c4 +

 a2b2 + b2c2 + c2a2 ≥

 a3b + b3c + c3a +

 ab3 + bc3 + ca3.

KMO Summer Program Test, 2001

Solution :

It is clear that it suffices to prove the following inequalities

a4 + a2b2 ≥ a3b + ab3

Page 78: 22573582 This Work Blends Together Classic Inequality Results With Brand New

8/8/2019 22573582 This Work Blends Together Classic Inequality Results With Brand New

http://slidepdf.com/reader/full/22573582-this-work-blends-together-classic-inequality-results-with-brand-new 78/127

78 Solutions

and a4

a2b2

a3b

ab3

.

The first one follows from Schur’s Inequalitya4 + abc

a ≥

a3b +

ab3

and the fact that a2b2 ≥ abc

a.

The second one is a simple consequence of the Cauchy-Schwarz Inequality:

(a3b + b3c + c3a)2 ≤ (a2b2 + b2c2 + c2a2)(a4 + b4 + c4)

(ab3 + bc3 + ca3)2 ≤ (a2b2 + b2c2 + c2a2)(a4 + b4 + c4).

80. [ Gabriel Dospinescu, Mircea Lascu ] For a given n > 2 find the smallest

constant kn with the property: if  a1, . . . , an > 0 have product 1, then

a1a2

(a21 + a2)(a2

2 + a1)+

a2a3

(a22 + a3)(a2

3 + a2)+ · · · +

ana1

(a2n + a1)(a2

1 + an)≤ kn.

Solution :

Let us take first a1 = a2 = · · · = an−1 = x, an =

1

xn−1 . We infer that

kn ≥ 2x2n−1

(xn+1 + 1)(x2n−1 + 1)+

n − 2

(1 + x)2>

n − 2

(1 + x)2

for all x > 0. Clearly, this implies kn ≥ n − 2. Let us prove that n − 2 is a good

constant and the problem will be solved.

First, we will prove that (x2 + y)(y2 + x) ≥ xy(1 + x)(1 + y). Indeed, this is the

same as (x + y)(x − y)2 ≥ 0. So, it suffices to prove that

1

(1 + a1)(1 + a2)+

1

(1 + a2)(1 + a3)+ · · · +

1

(1 + an)(1 + a1)≤ n − 2.

Now, we take a1

=x1

x2, . . . , a

n=

xn

x1and the above inequality becomes

nk=1

1 − xk+1xk+2

(xk + xk+1)(xk+1 + xk+2)

≥ 2,

which can be also written in the following from

nk=1

xkxk + xk+1

+x2k+1

(xk + xk+1)(xk+1 + xk+2)≥ 2.

Clearly,n

k=1

xkxk + xk+1

≥n

k=1

xkx1 + x2 + · · · + xn

= 1.

Page 79: 22573582 This Work Blends Together Classic Inequality Results With Brand New

8/8/2019 22573582 This Work Blends Together Classic Inequality Results With Brand New

http://slidepdf.com/reader/full/22573582-this-work-blends-together-classic-inequality-results-with-brand-new 79/127

Old and New Inequalities 79

So, we have to prove the inequalityn

k=1

x2k+1

(xk + xk+1)(xk+1 + xk+2)≥ 1.

Using the Cauchy-Schwarz Inequality, we infer that

nk=1

x2k+1

(xk + xk+1)(xk+1 + xk+2)≥

n

k+1

xk

2

n

k=1

(xk + xk+1)(xk+1 + xk+2)

and so it suffices to prove thatn

k=1

xk

2

≥n

k=1

x2k + 2

nk=1

xkxk+1 +

nk=1

xkxk+2.

But this one is equivalent to

2

1≤i<j≤n

xixj ≥ 2n

k=1

xkxk+1 +n

k=1

xkxk+2

and it is clear. Thus, kn = n − 2.

81. [ Vasile Cırtoa je ] For any real numbers a,b,c,x,y,z prove that the inequalityholds

ax + by + cz + 

(a2 + b2 + c2)(x2 + y2 + z2) ≥ 2

3(a + b + c)(x + y + z).

Kvant, 1989

Solution :

Let us denote t =

 x2 + y2 + z2

a2 + b2 + c2. Using the substitutions x = tp, y = tq and

z = tr, which imply

a2 + b2 + c2 = p2 + q2 + r2.

The given inequality becomes

ap + bq + cr + a2 + b2 + c2 ≥ 2

3(a + b + c)( p + q + r),

(a + p)2 + (b + q)2 + (c + r)2 ≥ 4

3(a + b + c)( p + q + r).

Since

4(a + b + c)( p + q + r) ≤ [(a + b + c) + ( p + q + r)]2,

it suffices to prove that

(a + p)2 + (b + q)2 + (c + r)2 ≥ 1

3[(a + p) + (b + q) + (c + r)]2.

This inequality is clearly true.

Page 80: 22573582 This Work Blends Together Classic Inequality Results With Brand New

8/8/2019 22573582 This Work Blends Together Classic Inequality Results With Brand New

http://slidepdf.com/reader/full/22573582-this-work-blends-together-classic-inequality-results-with-brand-new 80/127

80 Solutions

82. [ Vasile Cırtoa je ] Prove that the sides a,b,c of a triangle satisfy the inequality

3

a

b+

b

c+

c

a− 1

≥ 2

b

a+

c

b+

a

c

.

First solution :

We may assume that c is the smallest among a,b,c. Then let x = b − a + c

2. After

some computations, the inequality becomes

(3a

−2c)x2+x + c

a

4 (a

−c)2

≥0

⇔(3a

−2c)(2b

−a

−c)2+(4b+2c

−3a)(a

−c)2

≥0

which follows immediately from 3a ≥ 2c, 4b + 2c − 3a = 3(b + c − a) + b − c > 0.

Second solution :

Make the classical substitution a = y + z, b = z + x, c = x + y and clear denomi-

nators. The problem reduces to proving that

x3 + y3 + z3 + 2(x2y + y2z + z2x) ≥ 3(xy2 + yz2 + zx2).

We can of course assume that x is the smallest among x,y,z. Then we can write

y = x + m, z = x + n with nonnegatives m and n. A short computation shows that

the inequality reduces to 2x(m2

−mn + n2) + m3 + n3 + 2m2n

−3n2m

≥0. All we

need to prove is that m3 + n3 + 2m2n ≥ 3n2m ⇔ (n − m)3 − (n − m)m2 + m3 ≥ 0

and this follows immediately from the inequality t3 + 1 ≥ 3, true for t ≥ −1.

83. [ Walther Janous ] Let n > 2 and let x1, x2, . . . , xn > 0 add up to 1. Prove

thatni=1

1 +

1

xi

ni=1

n − xi1 − xi

.

Crux Mathematicorum

First solution :

The most natural idea is to use the fact thatn − xi1 − xi

= 1 +n − 1

x1 + x2 + · · · + xi−1 + xi+1 + · · · + xn.

Thus, we have

ni=1

n − xi1 − xi

ni=1

1 +

1n−1

√x1x2 . . . xi−1xi+1 . . . xn

and we have to prove the inequality

ni=1

1 +

1

xi

ni=1

1 +

1n−1

√x1x2 . . . xi−1xi+1 . . . xn

.

Page 81: 22573582 This Work Blends Together Classic Inequality Results With Brand New

8/8/2019 22573582 This Work Blends Together Classic Inequality Results With Brand New

http://slidepdf.com/reader/full/22573582-this-work-blends-together-classic-inequality-results-with-brand-new 81/127

Old and New Inequalities 81

But this one is not very hard, because it follows immediately by multiplying the

inequalities

j=i

1 +

1

xj

1 + n−1

 j=i

1

xj

n−1

obtained from Huygens Inequality.

Second solution :

We will prove even more, that

ni=1

1 +

1

xi

n2 − 1

n

n

·ni=1

1

1 − xi.

It is clear that this inequality is stronger than the initial one. First, let us prove that

ni=1

1 + xi1 − xi

n + 1

n − 1

n

.

This follows from Jensen’s Inequality for the convex function f (x) = ln(1 + x) −ln(1 − x). So, it suffices to prove that

n + 1

n − 1n

ni=1

xi

·ni=1

(1 − xi)2 ≥

n2 − 1

n

n

.

But a quick look shows that this is exactly the inequality proved in the solution of 

the problem 121.

84. [ Vasile Cırtoaje, Gheorghe Eckstein ] Consider positive real numbers

x1, x2,...,xn such that x1x2...xn = 1. Prove that

1

n − 1 + x1

+1

n − 1 + x2

+ ... +1

n − 1 + xn ≤1.

TST 1999, Romania

First solution :

Suppose the inequality is false for a certain system of  n numbers. Then we can

find a number k > 1 and n numbers which add up to 1, let them be ai, such that1

n − 1 + xi= kai. Then we have

1 =ni=1

1

kai− n + 1

<

ni=1

1

ai− n + 1

.

Page 82: 22573582 This Work Blends Together Classic Inequality Results With Brand New

8/8/2019 22573582 This Work Blends Together Classic Inequality Results With Brand New

http://slidepdf.com/reader/full/22573582-this-work-blends-together-classic-inequality-results-with-brand-new 82/127

82 Solutions

We have used here the fact that ai <1

n − 1. Now, we write 1 − (n − 1)ak = bk and

we find thatn

k=1

bk = 1 and alson

k=1

(1 − bk) < (n − 1)nb1 . . . bn. But this contradicts

the fact that for each j we have

1 − bj = b1 + · · · + bj−1 + bj+1 + · · · + bn ≥ (n − 1) n−1 

b1 . . . bj−1bj+1 . . . bn.

Second solution :

Let us write the inequality in the form

x1

n − 1 + x1+ x2

n − 1 + x2+ . . . + xn

n − 1 + xn≥ 1.

This inequality follows by summing the following inequalities

x1

n − 1 + x1≥ x

1− 1n

1

x1− 1

n

1 + x1− 1

n

2 + . . . + x1− 1

n

n

, . . . ,xn

n − 1 + xn≥ x

1− 1n

n

x1− 1

n

1 + x1− 1

n

2 + . . . + x1− 1

n

n

,

The first from these inequalities is equivalent to

x1− 1

n

1 + x1− 1

n

2 + . . . + x1− 1

nn ≥ (n − 1)x

− 1n

1

and follows from the AM-GM Inequality.

Remark .Replacing the numbers x1, x2, . . . , xn with

1

x1,

1

x2, . . . ,

1

xnrespectively, the in-

equality becomes as follows

1

1 + (n − 1)x1+

1

1 + (n − 1)x2+ . . . +

1

1 + (n − 1)xn≥ 1.

85. [ Titu Andreescu ] Prove that for any nonnegative real numbers a,b,c such

that a2 + b2 + c2 + abc = 4 we have 0 ≤ ab + bc + ca − abc ≤ 2.

USAMO, 2001

First solution (by Richard Stong):

The lower bound is not difficult. Indeed, we have ab + bc + ca ≥ 33√a2b2c2 and

thus it is enough to prove that abc ≤ 33√

a2b2c2, which follows from the fact that

abc ≤ 4. The upper bound instead is hard. Let us first observe that there are two

numbers among the three ones, which are both greater or equal than 1 or smaller

than or equal to 1. Let them be b and c. Then we have

4 ≥ 2bc + a2 + abc ⇒ (2 − a)(2 + a) ≥ bc(2 + a) ⇒ bc ≤ 2 − a.

Thus, ab + bc + ca−abc ≤ ab + 2−a + ac−abc and it is enough to prove the inequality

ab + 2 − a + ac − abc ≤ 2 ⇔ b + c − bc ≤ 1 ⇔ (b − 1)(c − 1) ≥ 0, which is true due to

our choice.

Page 83: 22573582 This Work Blends Together Classic Inequality Results With Brand New

8/8/2019 22573582 This Work Blends Together Classic Inequality Results With Brand New

http://slidepdf.com/reader/full/22573582-this-work-blends-together-classic-inequality-results-with-brand-new 83/127

Old and New Inequalities 83

Second solution :

We won’t prove again the lower part, since this is an easy problem. Let us con-

centrate on the upper bound. Let a ≥ b ≥ c and let a = x + y, b = x − y. The

hypothesis becomes x2(2 + c) + y2(2 − c) = 4 − c2 and we have to prove that

(x2 − y2)(1 − c) ≤ 2(1 − xc). Since y2 = 2 + c − 2 + c

2 − cx2, the problem asks to

prove the inequality4x2 − (4 − c2)

2 − c(1 − c) ≤ 2(1 − cx). Of course, we have c ≤ 1

and 0 ≤ y2 = 2 + c − 2+c2−c

x2 ⇒ x2 ≤ 2 − c ⇒ x ≤ √2 − c (we have used

the fact that a

≥b

⇒y

≥0 and b

≥0

⇒x

≥y

≥0). Now, consider the

function f  : [0, √2 − c] → R, f (x) = 2(1 − cx) − 4x2 − (4 − c2)2 − c

(1 − c). We have

f (x) = −2c − 8x · 1 − c

2 − c≤ 0 and thus f is decreasing and f (x) ≥ f (

√2 − c). So, we

have to prove that

f (√

2 − c) ≥ 0 ⇔ 2(1−c√

2 − c) ≥ (2−c)(1−c) ⇔ 3 ≥ c+2√

2 − c ⇔ (1−√2 − c)2 ≥ 0

clearly true. Thus, the problem is solved.

86. [ Titu Andreescu ] Prove that for any positive real numbers a,b,c the following

inequality holds

a + b + c

3 −3

√abc ≤ max{(√a − √b)2

, (√b − √c)2

, (√c − √a)2

}.

TST 2000, USA

Solution :

A natural idea would be to assume the contrary, which means that

a + b + c

3− 3

√abc ≤ a + b − 2

√ab

a + b + c

3− 3

√abc ≤ b + c − 2

√bc

a + b + c

3− 3

√abc ≤ c + a − 2

√ca.

Adding these inequalities, we find that

a + b + c − 33√

abc > 2(a + b + c −√

ab −√

bc − √ca).

Now, we will prove that a + b + c − 33√

abc ≤ 2(a + b + c − √ab − √

bc − √ca) and the

problem will be solved. Since the above inequality is homogeneous, we may assume

that abc = 1. Then, it becomes 2√

ab + 2√

bc + 2√

ca − a − b − c ≤ 3. Now, using

Schur’s inequality, we find that for any positive reals x,y,z we have:

2xy + 2yz + 2zx − x2 − y2 − z2 ≤ 9xyz

x + y + z≤ 3 3

 x2y2z2

All we have to do is take x =√

a, y =√

b, z =√

c in the above inequality.

Page 84: 22573582 This Work Blends Together Classic Inequality Results With Brand New

8/8/2019 22573582 This Work Blends Together Classic Inequality Results With Brand New

http://slidepdf.com/reader/full/22573582-this-work-blends-together-classic-inequality-results-with-brand-new 84/127

84 Solutions

87. [ Kiran Kedlaya ] Let a,b,c be positive real numbers. Prove that

a +√

ab +3√

abc

3≤ 3

 a · a + b

2· a + b + c

3.

Solution (by Anh Cuong):

We have that

a +√

ab +3√

abc ≤ a +3

 ab

a + b

2+

3√

abc.

Now we will prove that

a +3

 ab

a + b

2+

3√

abc ≤ 3

 a · a + b

2· a + b + c

3.

By the AM-GM Inequality, we have

3

 1 · 2a

a + b· 3a

a + b + c≤

1 +2a

a + b+

3a

a + b + c3

,

3

 1 · 1 · 3b

a + b + c≤

2 +3b

a + b + c3

,

3

 1 · 2b

a + b· 3c

a + b + c≤

1 + 2ba + b

+ 3ca + b + c

3.

Now, just add them up and we have the desired inequality. The equality occurs when

a = b = c.

88. Find the greatest constant k such that for any positive integer n which is not

a square, |(1 +√

n)sin(π√

n)| > k.

Vietnamese IMO Training Camp, 1995

Solution :

We will prove that π2

is the best constant. We must clearly have k <1 +

√i2 + 1

| sin

π√

i2 + 1 | for all positive integers i. Because | sin

π√

i2 + 1 | =

sinπ

i +√

i2 + 1, we deduce that

π

i +√

i2 + 1≥ sin

π

i +√

i2 + 1>

k

1 +√

i2 + 1, from

where it follows that k ≤ π

2. Now, let us prove that this constant is good. Clearly, the

inequality can be written

sin

π · {√n} >π

2(1 +√

n).

We have two cases

Page 85: 22573582 This Work Blends Together Classic Inequality Results With Brand New

8/8/2019 22573582 This Work Blends Together Classic Inequality Results With Brand New

http://slidepdf.com/reader/full/22573582-this-work-blends-together-classic-inequality-results-with-brand-new 85/127

Old and New Inequalities 85

i) The first case is when {√n} ≤ 1

2. Of course,

{√n} ≥ √n − √

n − 1 =1√

n +√

n − 1,

and because sin x ≥ x − x3

6we find that

sin(π{√n}) ≥ sinπ√

n

−1 +

√n

≥ π√

n

−1 +

√n − 1

6 π√

n

−1 +

√n

3

.

Let us prove that the last quantity is at leastπ

2(1 +√

n). This comes down to

2 +√

n − √n − 1

1 +√

n>

π2

3√

n +√

n − 12 ,

or 6(√

n +√

n − 1)2 + 3(√

n +√

n − 1) > π2(1 +√

n) and it is clear.

ii) The second case is when {√n} >1

2. Let x = 1 − {√n} <

1

2and let n =

k2 + p, 1

≤p

≤2k. Because

{

√n

}>

1

2 ⇒p

≥k + 1. Then it is easy to see that

x ≥ 1k + 1 +

√k2 + 2k

and so it suffices to prove that

sinπ

k + 1 +√

k2 + 2k≥ π

2(1 +√

k2 + k).

Using again the inequality sin x ≥ x − x3

6we infer that,

2√

k2 + k − √k2 + 2k − k + 1

1 +√

k2 + k>

π2

3

1 + k +

√k2 + 2k

2 .

But from the Cauchy-Schwarz Inequality we have 2√k2 + k − √k2 + 2k − k ≥ 0.

Because the inequality

1 + k +√

k2 + 2k2

>π2

3

1 +

 k2 + k

holds, this case is

also solved.

89. [ Dung Tran Nam ] Let x, y,z > 0 such that (x + y + z)3 = 32xyz. Find the

minimum and maximum of x4 + y4 + z4

(x + y + z)4.

Vietnam, 2004

First solution (by Tran Nam Dung):

We may of course assume that x + y + z = 4 and xyz = 2. Thus, we have to find the

Page 86: 22573582 This Work Blends Together Classic Inequality Results With Brand New

8/8/2019 22573582 This Work Blends Together Classic Inequality Results With Brand New

http://slidepdf.com/reader/full/22573582-this-work-blends-together-classic-inequality-results-with-brand-new 86/127

86 Solutions

extremal values of x4 + y4 + z4

44. Now, we have

x4 + y4 + z4 = (x2 + y2 + z2)2 − 2

x2y2 =

= (16 − 2

xy)2 − 2(

xy)2 + 4xyz(x + y + z) =

= 2a2 − 64a + 288,

where a = xy +yz +zx. Because y +z = 4−x and yz =2

x, we must have (4−x)2 ≥ 8

x,

which implies that 3 − √5 ≤ x ≤ 2. Due to symmetry, we have x,y,z ∈ [3 − √

5, 2].

This means that (x − 2)(y − 2)(z − 2) ≤ 0 and also

(x − 3 +√

5)(y − 3 +√

5)(z − 3 +√

5) ≥ 0.

Clearing paranthesis, we deduce that

a ∈

5,5√

5 − 1

2

.

But becausex4 + y4 + z4

44=

(a − 16)2 − 112

128, we find that the extremal values of the

expression are383 − 165

√5

256,

9

128, attained for the triples

3 − √

5,1 +

√5

2,

1 +√

5

2

,

respectively (2, 1, 1).

Second solution :

As in the above solution, we must find the extremal values of  x2 + y2 + z2 when

x + y + z = 1,xyz =1

32, because after that the extremal values of the expression

x4 + y4 + z4 can be immediately found. Let us make the substitution x =a

4, y =

b

4, z =

c

2, where abc = 1, a + b + 2c = 4. Then x2 + y2 + z2 =

a2 + b2 + 4c2

16and so we

must find the extremal values of  a2 + b2 + 4c2. Now, a2 + b2 + 4c2 = (4−2c)2 − 2

c+ 4c2

and the problem reduces to finding the maximum and minimum of 4c2

−8c

−1

c

where

there are positive numbers a,b,c such that abc = 1, a + b + 2c = 4. Of course, this

comes down to (4 − 2c)2 ≥ 4

c, or to c ∈

3 − √

5

2, 1

. But this reduces to the study

of the function f (x) = 4x2 − 8x − 1

xdefined for

3 − √

5

2, 1

, which is an easy task.

90. [ George Tsintifas ] Prove that for any a, b, c, d > 0,

(a + b)3(b + c)3(c + d)3(d + a)3 ≥ 16a2b2c2d2(a + b + c + d)4.

Crux Mathematicorum

Page 87: 22573582 This Work Blends Together Classic Inequality Results With Brand New

8/8/2019 22573582 This Work Blends Together Classic Inequality Results With Brand New

http://slidepdf.com/reader/full/22573582-this-work-blends-together-classic-inequality-results-with-brand-new 87/127

Old and New Inequalities 87

Solution :

Let us apply Mac-Laurin Inequality for

x = abc, y = bcd,z = cda, t = dab.

We will find that

abc

4

3

abc · bcd · cda

4=

a2b2c2d2

a

4.

Thus, it is enough to prove the stronger inequality

(a + b)(b + c)(c + d)(d + a) ≥ (a + b + c + d)(abc + bcd + cda + dab).

Now, let us observe that

(a + b)(b + c)(c + d)(d + a) = (ac + bd + ad + bc)(ac + bd + ab + cd) =

= (ac + bd)2 +

a2(bc + bd + cd) ≥ 4abcd +

a2(bc + bd + cd) =

= (a + b + c + d)(abc + bcd + cda + dab).

And so the problem is solved.

91. [ Titu Andreescu, Gabriel Dospinescu ] Find the maximum value of the ex-

pression(ab)n

1 − ab +(bc)n

1 − bc +(ca)n

1 − cawhere a,b,c are nonnegative real numbers which add up to 1 and n is some positive

integer.

Solution :

First, we will treat the case n > 1. We will prove that the maximum value is1

3 · 4n−1. It is clear that ab, bc, ca ≤ 1

4and so

(ab)n

1 − ab+

(bc)n

1 − bc+

(ca)n

1 − ca≤ 4

3((ab)n + (bc)n + (ca)n) .

Thus, we have to prove that (ab)n

+ (bc)n

+ (ca)n

≤1

4n . Let a the maximum amonga,b,c. Then we have

1

4n≥ an(1 − a)n = an(b + c)n ≥ anbn + ancn + nanbn−1c ≥ anbn + bncn + cnan.

So, we have proved that in this case the maximum is at most1

3 · 4n−1. But for

a = b =1

2, c = 0 this value is attained and this shows that the maximum value is

1

3 · 4n−1for n > 1. Now, suppose that n = 1. In this case we have

ab

1

−ab

=

1

1

−ab

− 3.

Page 88: 22573582 This Work Blends Together Classic Inequality Results With Brand New

8/8/2019 22573582 This Work Blends Together Classic Inequality Results With Brand New

http://slidepdf.com/reader/full/22573582-this-work-blends-together-classic-inequality-results-with-brand-new 88/127

88 Solutions

Using the fact that a + b + c = 1, it is not difficult to prove that

1

1 − ab+

1

1 − bc+

1

1 − ca=

3 − 2

ab + abc

1 −

ab + abc − a2b2c2.

We will prove that ab

1 − ab≤ 3

8. With the above observations, this reduces to

ab ≤ 3 − 27(abc)2 + 19abc

11.

But using Schur’s Inequality we infer thatab ≤ 1 + 9abc

4

and so it is enough to show that

9abc + 1

4≤ 3 − 27(abc)2 + 19abc

11⇔ 108(abc)2 + 23abc ≤ 1,

which is true because abc ≤ 1

27.

Hence for n = 1, the maximum value is3

8attained for a = b = c.

92. Let a,b,c be positive real numbers. Prove that

1

a(1 + b)+

1

b(1 + c)+

1

c(1 + a) ≥3

3√abc(1 + 3√abc).

Solution :

The following observation is crucial

(1 + abc)

1

a(1 + b)+

1

b(1 + c)+

1

c(1 + a)

+ 3 =

1 + abc + a + ab

a(1 + b)=

= 1 + a

a(1 + b)+

b(c + 1)

1 + b.

We use now twice the AM-GM Inequality to find that

1 + a

a(1 + b)

+ b(c + 1)

1 + b ≥3

3

√abc

+ 33√

abc.

And so we are left with the inequality

33√

abc+ 3

3√

abc − 3

1 + abc≥ 3

3√

abc(1 +3√

abc),

which is in fact an identity!

93. [ Dung Tran Nam ] Prove that for any real numbers a,b,c such that a2 + b2 +

c2 = 9,

2(a + b + c) − abc ≤ 10.

Vietnam, 2002

Page 89: 22573582 This Work Blends Together Classic Inequality Results With Brand New

8/8/2019 22573582 This Work Blends Together Classic Inequality Results With Brand New

http://slidepdf.com/reader/full/22573582-this-work-blends-together-classic-inequality-results-with-brand-new 89/127

Old and New Inequalities 89

First solution (by Gheorghe Eckstein):

Because max{a,b,c} ≤ 3 and |abc| ≤ 10, it is enough to consider only the cases

when a,b,c ≥ 0 or exactly of the three numbers is negative. First, we will suppose

that a,b,c are nonnegative. If  abc ≥ 1, then we are done, because

2(a + b + c) − abc ≤ 2 

3(a2 + b2 + c2) − 1 < 10.

Otherwise, we may assume that a < 1. In this case we have

2(a + b + c) − abc ≤ 2a + 2 b2 + c2

2 = 2a + 2 18 − 2a2 ≤ 10.

Now, assume that not all three numbers are nonnegative and let c < 0.

Thus, the problem reduces to proving that for any nonnegative x,y,z whose

sum of squares is 9 we have 4(x + y − z) + 2xyz ≤ 20. But we can write this as

(x−2)2+(y−2)2+(z−1)2 ≥ 2xyz − 6z − 2. Because 2xyz−6z−2 ≤ x(y2+z2)−6z−2 =

−z3 + 3z − 2 = −(z − 1)2(z + 2) ≤ 0, the inequality follows.

Second solution :

Of course, we have |a|, |b|, |c| ≤ 3 and |a + b + c|, |abc| ≤ 3√

3. Also, we may

assume of course that a,b,c are non-zero and that a ≤ b ≤ c. If  c < 0 then we have

2(a+b+c)

−abc <

−abc

≤3√

3 < 10. Also, if a

≤b < 0 < c then we have 2(a+b+c) <

2c ≤ 6 < 10 + abc because abc > 0. If  a < 0 < b ≤ c, using the Cauchy-Schwarz

Inequality we find that 2b + 2c− a ≤ 9. Thus, 2(a + b + c) = 2b + 2c− a + 3a ≤ 9 + 3a

and it remains to prove that 3a − 1 ≤ abc. But a < 0 and 2bc ≤ 9 − a2, so that it

remains to show that9a − a3

2≥ 3a − 1 ⇔ (a + 1)2(a − 2) ≤ 0, which follows. So, we

 just have to threat the case 0 < a ≤ b ≤ c. In this case we have 2b + 2c + a ≤ 9 and

2(a + b + c) ≤ 9 + a. So, we need to prove that a ≤ 1 + abc. This is clear if  a < 1 and

if  a > 1 we have b,c > 1 and the inequality is again. Thus, the problem is solved.

94. [ Vasile Cırtoaje ] Let a,b,c be positive real numbers. Prove that

a +

1

b − 1

b +

1

c − 1

+

b +

1

c − 1

c +

1

a − 1

+

c +

1

a − 1

a +

1

b − 1 ≥ 3.

First solution :

With the notations x = a +1

b− 1, y = b +

1

c− 1, z = c +

1

a− 1, the inequality

becomes

xy + yz + zx ≥ 3.

We consider without loss of generality that x = max{x,y,z}. From

(x+1)(y+1)(z+1) = abc+1

abc+a+b+c+

1

a+

1

b+

1

c≥ 2+a+b+c+

1

a+

1

b+

1

c= 5+x+y+z,

Page 90: 22573582 This Work Blends Together Classic Inequality Results With Brand New

8/8/2019 22573582 This Work Blends Together Classic Inequality Results With Brand New

http://slidepdf.com/reader/full/22573582-this-work-blends-together-classic-inequality-results-with-brand-new 90/127

90 Solutions

we get

xyz + xy + yz + zx ≥ 4,

with equality if and only if abc = 1. Because y+z =1

a+b+

(c − 1)2

c> 0 we distinguish

two cases a) x> 0, yz ≤ 0; b)x> 0,y> 0,z> 0.

a) x> 0, yz ≤ 0. We have xyz ≤ 0 and from xyz + xy + yz + zx ≥ 4 we get

xy + yz + zx ≥ 4 > 3.

b) x> 0,y> 0,z> 0. We denote xy + yz + zx = 3d2 with d > 0. From the mean

inequalities we have

xy + yz + zx ≥ 3 3 

x2y2z2,

from which we deduce that xyz ≤ d3. On the basis of this result, from the inequality

xyz + xy + yz + zx ≥ 4, we obtain d3 + 3d2 ≥ 4, (d − 1)(d + 2)2 ≥ 0, d ≥ 1 so

xy + yz + zx ≥ 3. With this the given inequality is proved. We have equality in the

case a = b = c = 1.

Second solution :

Let u = x + 1, v = y + 1, w = z + 1. Then we have

uvw = u + v + w + abc +1

abc ≥u + v + w + 2.

Now, consider the function f (t) = 2t3+t2(u+v+w)−uvw. Because limt→∞

f (t) = ∞ and

f (1) ≤ 0, we can find a real number r ≥ 1 such that f (r) = 0. Consider the numbers

m =u

r, n =

v

r, p =

w

r. They verify mnp = m + n + p + 2 we deduce from problem 49

that mn + np + pm ≥ 2(m + n + p) ⇒ uv + vw + wu ≥ 2r(u + v + w) ≥ 2(u + v + w).

But because u = x + 1, v = y + 1, w = z + 1, this last relation is equivalent to

xy + yz + zx ≥ 3, which is what we wanted.

95. [ Gabriel Dospinescu ] Let n be an integer greater than 2. Find the greatest

real number mn and the least real number M n such that for any positive real numbers

x1, x2, . . . , xn (with xn = x0, xn+1 = x1),

mn ≤ni=1

xixi−1 + 2(n − 1)xi + xi+1

≤ M n.

Solution :

We will prove that mn =1

2(n − 1), M n =

1

2. First, let us see that the inequality

ni=1

xixi−1 + 2(n − 1)xi + xi+1

≥ 1

2(n − 1)

Page 91: 22573582 This Work Blends Together Classic Inequality Results With Brand New

8/8/2019 22573582 This Work Blends Together Classic Inequality Results With Brand New

http://slidepdf.com/reader/full/22573582-this-work-blends-together-classic-inequality-results-with-brand-new 91/127

Old and New Inequalities 91

is trivial, because xi−1 + 2(n − 1)xi + xi+1 ≤ 2(n − 1) ·n

k=1

xk for all i. This shows

that mn ≥ 1

2(n − 1). Taking xi = xi, the expression becomes

1

x + xn−1 + 2(n − 1)+

(n − 2)x

1 + 2(n − 1)x + x2+

xn−1

1 + 2(n − 1)xn−1 + xn−2

and taking the limit when x approaches 0, we find that mn ≤ 1

2(n − 1)and thus

mn =

1

2(n − 1) .

Now, we will prove that M n ≥ 1

2. Of course, it suffices to prove that for any

x1, x2, . . . , xn > 0 we have

ni=1

xi

xi−1 + 2(n − 1)xi + xi+1≤ 1

2.

But it is clear that

ni=1

2xixi−1 + 2(n − 1)xi + xi+1

≤ni=1

2xi2√

xi−1 · xi+1 + 2(n − 1)xi=

=ni=1

1

n − 1 +

√xi−1 · xi+1

xi

.

Taking

√xi−1 · xi+1

xi= ai, we have to prove that if  

ni=1

ai = 1 then

ni=1

1

n − 1 + ai≤ 1. But this has already been proved in the problem 84. Thus,

M n ≥ 1

2and because for x1 = x2 = · · · = xn we have equality, we deduce that

M n =1

2, which solves the problem.

96. [ Vasile Cırtoaje ] If  x,y,z are positive real numbers, then

1

x2 + xy + y2+

1

y2 + yz + z2+

1

z2 + zx + z2≥ 9

(x + y + z)2.

Gazeta Matematica

Solution :

Considering the relation

x2 + xy + y2 = (x + y + z)2 − (xy + yz + zx) − (x + y + z)z,

Page 92: 22573582 This Work Blends Together Classic Inequality Results With Brand New

8/8/2019 22573582 This Work Blends Together Classic Inequality Results With Brand New

http://slidepdf.com/reader/full/22573582-this-work-blends-together-classic-inequality-results-with-brand-new 92/127

92 Solutions

we get(x + y + z)2

x2 + xy + y2=

1

1 − xy + yz + zx

(x + y + z)2− z

x + y + z

,

or(x + y + z)2

x2 + xy + y2=

1

1 − (ab + bc + ca) − c,

where a =x

x + y + z, b =

y

x + y + z, c =

z

x + y + z. The inequality can be rewritten

as1

1 − d − c

+1

1 − d − b

+1

1 − d − a ≥9,

where a,b,c are positive reals with a + b + c = 1 and d = ab + bc + ca. After making

some computations the inequality becomes

9d3 − 6d2 − 3d + 1 + 9abc ≥ 0

or

d(3d − 1)2 + (1 − 4d + 9abc) ≥ 0.

which is Schur’s Inequality.

97. [ Vasile Cırtoaje ] For any a, b, c, d > 0 prove that

2(a3 + 1)(b3 + 1)(c3 + 1)(d3 + 1) ≥ (1 + abcd)(1 + a2)(1 + b2)(1 + c2)(1 + d2).

Gazeta Matematica

Solution :

Using Huygens Inequality(1 + a4) ≥ (1 + abcd)4,

we notice that it is enough to show that that

24

(a3 + 1)4 ≥

(1 + a4)(1 + a2)4.

Of course, it suffices to prove that 2(a3 + 1)4 ≥ (a4 +1)(a2 + 1)4 for any positive

real a. But (a2 +1)4

≤(a+1)2(a3 +1)2 and we are left with the inequality 2(a3 +1)2

≥(a + 1)2(a4 + 1) ⇔ 2(a2 − a + 1)2 ≥ a4 + 1 ⇔ (a − 1)4 ≥ 0, which follows.

98. Prove that for any real numbers a,b,c,

(a + b)4 + (b + c)4 + (c + a)4 ≥ 4

7(a4 + b4 + c4).

Vietnam TST, 1996

Page 93: 22573582 This Work Blends Together Classic Inequality Results With Brand New

8/8/2019 22573582 This Work Blends Together Classic Inequality Results With Brand New

http://slidepdf.com/reader/full/22573582-this-work-blends-together-classic-inequality-results-with-brand-new 93/127

Old and New Inequalities 93

Solution :

Let us make the substitution a + b = 2z, b + c = 2x, c + a = 2y. The inequality

becomes

(y + z − x)4 ≤ 28

x4. Now, we have the following chain of identities

(y + z − x)4 =

x2 + 2yz − 2xy − 2xz

2

= 3

x22

+ 4

x2

(yz − xy − xz)

+ 4

(xy + xz − yz)2 = 3

x2

2− 4

xy

x2

+

+16

x2y2 − 4

xy

2

= 4

x2

2

+ 16

x2y2 −

x

4

≤ 28

x4

because

x2

2 ≤ 3

x4,

x2y2 ≤

x4.

99. Prove that if  a,b,c are positive real numbers such that abc = 1, then

1

1 + a + b+

1

1 + b + c+

1

1 + c + a≤ 1

2 + a+

1

2 + b+

1

2 + c.

Bulgaria, 1997

Solution :

Let x = a + b + c and y = ab + bc + ca. Using brute-force, it is easy to see that

the left hand side isx2 + 4x + y + 3

x2 + 2x + y + xy, while the right hand side is

12 + 4x + y

9 + 4x + 2y. Now,

the inequality becomes

x2 + 4x + y + 3

x2 + 2x + y + xy− 1 ≤ 12 + 4x + y

9 + 4x + 2y− 1 ⇔ 2x + 3 − xy

x2 + 2x + y + xy≤ 3 − y

9 + 4x + 2y.

For the last inequality, we clear denominators. Then using the inequalities x ≥ 3, y ≥3, x2 ≥ 3y, we have

5

3x2y ≥ 5x2,

x2y

3≥ y2, xy2 ≥ 9x, 5xy ≥ 15x,xy ≥ 3y and x2y ≥ 27.

Summing up these inequalities, the desired inequality follows.

100. [ Dung Tran Nam ] Find the minimum value of the expression 1a + 2b + 3cwhere a,b,c are positive real numbers such that 21ab + 2bc + 8ca ≤ 12.

Vietnam, 2001

First solution (by Dung Tran Nam):

Let1

a= x,

2

b= y,

3

c= z. Then it is easy to check that the condition of the problem

becomes 2xyz ≥ 2x + 4y + 7z. And we need to minimize x + y + z. But

z(2xy − 7) ≥ 2x + 4y ⇒

2xy > 7

z ≥ 2x + 4y

2xy

−7

Page 94: 22573582 This Work Blends Together Classic Inequality Results With Brand New

8/8/2019 22573582 This Work Blends Together Classic Inequality Results With Brand New

http://slidepdf.com/reader/full/22573582-this-work-blends-together-classic-inequality-results-with-brand-new 94/127

94 Solutions

Now, we transform the expression so that after one application of the AM-GM

Inequality the numerator 3xy − 7 should vanish x + y + z ≥ x + y +2x + 4y

2xy − 7=

x +11

2x+ y − 7

2x+

2x +14

x2xy − 7

≥ x +11

2x+ 2

 1 +

7

x2. But, it is immediate to prove

that 2

 1 +

7

x2≥

3 +7

x2

and so x + y + z ≥ 3

2+ x +

9

x≥ 15

2. We have equality for

x = 3, y =5

2, z = 2. Therefore, in the initial problem the answer is

15

2, achieved for

a =1

3 , b =4

5 , c =3

2 .

Second solution :

We use the same substitution and reduce the problem to finding the minimum

value of  x + y + z when 2xyz ≥ 2x + 4y + 7z. Applying the weighted the AM-GM

Inequality we find that

x + y + z ≥

5x

2

25

(3y)13

15z

4

415

.

And also 2x + 4y + 7z ≥ 10

15

· 12

13

· 5

715

· x

15

· y

13

· z

715

. This means that (x++y + z)2(2x + 4y + 7z) ≥ 225

2xyz. Because 2xyz ≥ 2x + 4y + 7z, we will have

(x + y + z)2 ≥ 225

4⇒ x + y + z ≥ 15

2

with equality for x = 3, y =5

2, z = 2.

101. [ Titu Andreescu, Gabriel Dospinescu ] Prove that for any x,y,z,a,b,c > 0

such that xy + yz + zx = 3,

ab + c

(y + z) + bc + a

(z + x) + ca + b

(x + y) ≥ 3.

Solution :

We will prove the inequality

a

b + c(y + z) +

b

c + a(z + x) +

c

a + b(x + y) ≥

 3(xy + yz + zx)

for any a,b,c,x,y,z . Because the inequality is homogeneous in x,y,z we can assume

that x + y + z = 1. But then we can apply the Cauchy-Schwarz Inequality so that

Page 95: 22573582 This Work Blends Together Classic Inequality Results With Brand New

8/8/2019 22573582 This Work Blends Together Classic Inequality Results With Brand New

http://slidepdf.com/reader/full/22573582-this-work-blends-together-classic-inequality-results-with-brand-new 95/127

Old and New Inequalities 95

to obtain

a

b + cx +

b

c + ay +

c

a + bz +

 3(xy + yz + zx) ≤

≤ 

a

b + c

2 x2 +

 3

4

 xy +

 3

4

 xy ≤

≤ 

a

b + c

2

+3

2

 x2 + 2

xy =

 a

b + c

2

+3

2.

Thus, we are left with the inequality  a

b + c2

+3

2 ≤ a

b + c . But this one

is equivalent to ab

(c + a)(c + b)≥ 3

4, which is trivial.

Remark .

A stronger inequality is the following:

a

b + c(y + z) +

b

c + a(z + x) +

c

a + b(x + y) ≥

 (x + y)(x + z) − (x + y + z),

which may be obtained by applying the Cauchy-Schwarz Inequality, as follows

a

b + c(y + z) +

b

c + a(z + x) +

c

a + b(x + y) =

(a+b+c)

y + zb + c

+z + xc + a

+x + ya + b

−2(x+y +z) ≥ 1

2(√y + z +

√z + x +√x + y)2−

2(x + y + z) = 

(x + y)(x + z) − (x + y + z).

A good exercise for readers is to show that (x + y)(x + z) ≥ x + y + z +

 3(xy + yz + zx).

102. Let a,b,c be positive real numbers. Prove that

(b + c − a)2

(b + c)2 + a2+

(c + a − b)2

(c + a)2 + b2+

(a + b − c)2

(a + b)2 + c2≥ 3

5.

Japan, 1997

First solution :

Let x =b + c

a, y =

c + a

b, z =

a + b

c. The inequality can be written

(x − 1)2

x2 + 1≥ 3

5.

Using the Cauchy-Schwarz Inequality, we find that

(x − 1)2

x2 + 1≥ (x + y + z − 3)2

x2 + y2 + z2 + 3

Page 96: 22573582 This Work Blends Together Classic Inequality Results With Brand New

8/8/2019 22573582 This Work Blends Together Classic Inequality Results With Brand New

http://slidepdf.com/reader/full/22573582-this-work-blends-together-classic-inequality-results-with-brand-new 96/127

96 Solutions

and so it is enough to prove that

(x + y + z − 3)2

x2 + y2 + z2 + 3≥ 3

5⇔ (

x)2 − 15

x + 3

xy + 18 ≥ 0.

But from Schur’s Inequality, after some computations, we deduce thatxy ≥ 2

x. Thus, we havex2

− 15

x + 3

xy + 18 ≥

x2

− 9

x + 18 ≥ 0,

the last one being clearly true since x ≥ 6.

Second solution :

Of course, we may take a + b + c = 2. The inequality becomes 4(1 − a)2

2 + 2(1 − a)2≥ 3

5⇔

1

1 + (1 − a)2≤ 27

10.

But with the substitution 1 − a = x, 1 − b = y, 1 − c = z, the inequality reduces to

that from problem 47.

103. [ Vasile Cırtoaje, Gabriel Dospinescu ] Prove that if  a1, a2, . . . , an ≥ 0 then

an1 + an2 + · · · + ann − na1a2 . . . an ≥ (n − 1)a1 + a2 + · · · + an−1

n

−1

− ann

where an is the least among the numbers a1, a2, . . . , an.

Solution :

Let ai − an = xi ≥ 0 for i ∈ {1, 2, . . . , n − 1}. Now, let us look at

ni=1

ani − n ·ni=1

ai − (n − 1)

n−1i=1

ai

n − 1− an

n

as a polynomial in a = an. It is in fact

an +n−1i=1

(a + xi)n − na

n−1i=1

(a + xi) − (n − 1)

x1 + x2 + · · · + xn−1

n − 1

n

.

We will prove that the coefficient of  ak is nonnegative for all k ∈ {0, 1, . . . , n − 1},

because clearly the degree of this polynomial is at most n − 1. For k = 0, this follows

from the convexity of the function f (x) = xn

n−1i=1

xni ≥ (n − 1)

n−1i=1

xi

n − 1

n

.

Page 97: 22573582 This Work Blends Together Classic Inequality Results With Brand New

8/8/2019 22573582 This Work Blends Together Classic Inequality Results With Brand New

http://slidepdf.com/reader/full/22573582-this-work-blends-together-classic-inequality-results-with-brand-new 97/127

Old and New Inequalities 97

For k > 0, the coefficient of  ak isn

k

n−1i=1

xn−ki − n

1≤i1<···<in−k≤n−1

xi1xi2 . . . xin−k .

Let us prove that this is nonnegative. From the AM-GM Inequality we have

n

1≤i1<i2<···<in−k≤n−1

xi1xi2 . . . xin−k ≤

n

n−

k 1≤i1<i2<···<in−k≤n−1

xn−ki1+ xn−ki2

+ · · · + xn−kin−k =k

n−

n

kn−1

i=1

xn−ki

which is clearly smaller than

n

k

n−1i=1

xn−ki . This shows that each coefficient of the

polynomial is nonnegative and so this polynomial takes nonnegative values when

restricted to nonnegative numbers.

104. [ Turkevici ] Prove that for all positive real numbers x,y,z,t,

x4 + y4 + z4 + t4 + 2xyzt ≥ x2y2 + y2z2 + z2t2 + t2x2 + x2z2 + y2t2.

Kvant

Solution :Clearly, it is enough to prove the inequality if  xyzt = 1 and so the problem

becomes

If  a,b,c,d have product 1, then a2 + b2 + c2 + d2 + 2 ≥ ab + bc + cd + da + ac + bd.

Let d the minimum among a,b,c,d and let m = 3√

abc. We will prove that

a2 + b2 + c2 + d2 + 2 − (ab + bc + cd + da + ac + bd) ≥ d2 + 3m2 + 2 − (3m2 + 3md),

which is in fact

a2 + b2 + c2 − ab − bc − ca ≥ d

a + b + c − 33√

abc

.

Because d ≤ 3√

abc, proving this first inequality comes down to the inequality

a2 + b2 + c2 − ab − bc − ca ≥ 3√abc

a + b + c − 3 3√abc

.

Take u =a

3√

abc, v =

b3√

abc, w =

c3√

abc. Using problem 74, we find that

u2 + v2 + w2 + 3 ≥ u + v + w + uv + vw + wu

which is exactly a2 + b2 + c2 − ab − bc − ca ≥ 3√

abc

a + b + c − 3 3√

abc

. Thus, it

remains to prove that d2 + 2 ≥ 3md ⇔ d2 + 2 ≥ 33√

d2, which is clear.

Page 98: 22573582 This Work Blends Together Classic Inequality Results With Brand New

8/8/2019 22573582 This Work Blends Together Classic Inequality Results With Brand New

http://slidepdf.com/reader/full/22573582-this-work-blends-together-classic-inequality-results-with-brand-new 98/127

98 Solutions

105. Prove that for any real numbers a1, a2, . . . , an the following inequality holdsni=1

ai

2

≤n

i,j=1

ij

i + j − 1aiaj .

Solution :

Observe that

n

i,j=1

ij

i + j − 1

aiaj =n

i,j=1

iai· jaj  

1

0

ti+j−2dt =

=

 1

0

n

i,j=1

iai · jaj · ti−1+j−1

dt =

=

 1

0

ni=1

iai · ti−1

2

.

Now, using the Cauchy-Schwarz Inequality for integrals, we get

 1

0

n

i=1

iai · ti−1

2

dt ≥ 

1

0

n

i=1

iai · ti−1

dt

2

=

n

i=1

ai

2

,

which ends the proof.

106. Prove that if  a1, a2, . . . , an, b1, . . . , bn are real numbers between 1001 and

2002, inclusively, such that a21 + a2

2 + · · · + a2n = b2

1 + b22 + · · · + b2

n, then we have the

inequality

a31

b1+

a32

b2+ · · · +

a3n

bn≤ 17

10

a2

1 + a22 + · · · + a2

n

.

TST Singapore

Solution :

The key ideas are thataibi

1

2, 2

for any i and that for all x ∈

1

2, 2

we have

the inequality x2 + 1 ≤ 5

2x. Consequently, we have

5

2· ai

bi≥ 1 +

a21

b2i

⇒ 5

2aibi ≥ a2

i + b2i

and also

5

2

ni=1

aibi ≥ni=1

a2i + b2

i

= 2

ni=1

a2i (1)

Page 99: 22573582 This Work Blends Together Classic Inequality Results With Brand New

8/8/2019 22573582 This Work Blends Together Classic Inequality Results With Brand New

http://slidepdf.com/reader/full/22573582-this-work-blends-together-classic-inequality-results-with-brand-new 99/127

Old and New Inequalities 99

Now, the observation thata2i

b2i

=

a3i

biai · bi

and the inequality5

2· ai

bi≥ 1 +

a2i

b2i

allow us to

write5

2a2i ≥ a3

i

bi+ aibi and adding up these inequalities yields

5

2

ni=1

a2i ≥

ni=1

a3i

bi+ aibi

=

ni=1

a3i

bi+

ni=1

aibi (2)

Using (1) and (2) we find that

a31

b1 +

a32

b2 + · · · +

a3n

bn ≤17

10

a21 + a

22 + · · · + a

2n

,

which is the desired inequality.

107. [ Titu Andreescu, Gabriel Dospinescu ] Prove that if  a,b,c are positive real

numbers which add up to 1, then

(a2 + b2)(b2 + c2)(c2 + a2) ≥ 8(a2b2 + b2c2 + c2a2)2.

Solution :

Let x =1

a, y =

1

b, z =

1

c. We find the equivalent form if 

1

x+

1

y+

1

z= 1 then

(x2 + y2)(y2 + z2)(z2 + x2) ≥ 8(x2 + y2 + z2)2.

We will prove the following inequality

(x2 + y2)(y2 + z2)(z2 + x2)

1

x+

1

y+

1

z

2

≥ 8(x2 + y2 + z2)2

for any positive numbers x,y,z.

Write x2 + y2 = 2c, y2 + z2 = 2a, z2 + x2 = 2b. Then the inequality becomes abc

b + c − a≥

a.

Recall Schur’s Inequality

a4 + abc(a + b + c) ≥

a3(b + c) ⇔ abc(a + b + c) ≥ a3(b + c − a).

Now, using Holder’s Inequality, we find that

a3(b + c − a) =

a31√

b + c − a

2≥

a3

1√b + c − a

2.

Combining the two inequalities, we find that abc

b + c − a≥

a

and so the inequality is proved.

Page 100: 22573582 This Work Blends Together Classic Inequality Results With Brand New

8/8/2019 22573582 This Work Blends Together Classic Inequality Results With Brand New

http://slidepdf.com/reader/full/22573582-this-work-blends-together-classic-inequality-results-with-brand-new 100/127

100 Solutions

108. [ Vasile Cırtoaje ] If  a,b,c,d are positive real numbers such that abcd = 1,

then1

(1 + a)2+

1

(1 + b)2+

1

(1 + c)2+

1

(1 + d)2≥ 1.

Gazeta Matematica

Solution :

If follows by summing the inequalities

1

(1 + a)2+

1

(1 + b)2≥ 1

1 + ab,

1

(1 + c)2+

1

(1 + d)2≥ 1

1 + cd.

The first from these inequalities follows from

1

(1 + a)2+

1

(1 + b)2− 1

1 + ab=

ab(a2 + b2) − a2b2 − 2ab + 1

(1 + a)2(1 + b)2(1 + c)2=

=ab(a − b)2 + (ab − 1)2

(1 + a)2(1 + b)2(1 + ab)≥ 0.

Equality holds if  a = b = c = d = 1.

109. [ Vasile Cırtoaje ] Let a,b,c be positive real numbers. Prove that

a2

b2 + c2+ b

2

c2 + a2+ c

2

a2 + b2≥ a

b + c+ b

c + a+ c

a + b.

Gazeta Matematica

Solution :

We have the following identities

a2

b2 + c2− a

b + c=

ab(a − b) + ac(a − c)

(b + c)(b2 + c2)

b2

c2 + a2− b

c + a=

bc(b − c) + ab(b − a)

(c + a)(c2 + a2)

c2

a2 + b2 −c

a + b =

ac(c

−a) + bc(c

−b)

(b + a)(b2 + a2) .Thus, we have a2

b2 + c2−

a

b + c=

ab(a − b)

(b + c)(b2 − c2)− ab(a − b)

(a + c)(a2 + c2)

=

=

a2 + b2 + c2 + ab + bc + ca ·

ab(a − b)2

(b + c)(c + a)(b2 + c2)(c2 + a2)≥ 0.

Page 101: 22573582 This Work Blends Together Classic Inequality Results With Brand New

8/8/2019 22573582 This Work Blends Together Classic Inequality Results With Brand New

http://slidepdf.com/reader/full/22573582-this-work-blends-together-classic-inequality-results-with-brand-new 101/127

Old and New Inequalities 101

110. [ Gabriel Dospinescu ] Let a1, a2, . . . , an be real numbers and let S  be a

non-empty subset of {1, 2, . . . , n}. Prove that

i∈S

ai

2

1≤i≤j≤n

(ai + . . . + aj)2.

TST 2004, Romania

First solution :

Denote sk = a1 +

· · ·+ ak for k = 1, n and also sn+1 = 0. Define now

bi =

1 if  i ∈ S 

0 , otherwise

Using Abel’s summation we find thati∈S

ai = a1b1 + a2b2 + · · · + anbn =

= s1(b1 − b2) + s2(b2 − b3) + · · · + sn−1(bn−1 − bn) + snbn + sn+1(−b1)

Now, put b1 − b2 = x1, b2 − b3 = x2, . . . , bn−1 − bn = xn−1, bn = xn, xn+1 = −b1. So

we have

i∈S

ai =

n+1i=1

xisi

and also xi ∈ {−1, 0, 1}. Clearly,

n+1i=1

xi = 0. On the other hand, using Lagrange

identity we find that

1≤i≤j≤n

(ai + · · · + aj)2 =ni=1

s2i +

1≤i<j≤n

(sj − si)2 =

=

1≤i<j≤n+1

(sj − si)2 = (n + 1)

n+1i=1

s2i −

n+1i=1

si

2

.

So we need to prove that

(n + 1)n+1i=1

s2i ≥

n+1i=1

sixi

2

+

n+1i=1

si

2

.

But it is clear that

n+1i=1

sixi

2

+

n+1i=1

si

2

=n+1i=1

s2i (1 + x2

i )

+ 2

1≤i<j≤n+1

sisj(xixj + 1)

Page 102: 22573582 This Work Blends Together Classic Inequality Results With Brand New

8/8/2019 22573582 This Work Blends Together Classic Inequality Results With Brand New

http://slidepdf.com/reader/full/22573582-this-work-blends-together-classic-inequality-results-with-brand-new 102/127

102 Solutions

Now, using the fact that 2sisj ≤ si2 + s2

j , 1 + xixj ≥ 0, we can write

2

1≤i<jn+1

sisj(xixj + 1) ≤

1≤i<j≤n+1

(s2i + s2

j )(1 + xixj) =

n (s21 + · · · + s2

n+1) + s21x1(x2 + x3 + · · · + xn) + · · · + s2

nxn(x1 +

+ x2 + · · · + xn−1) = −s21x2

1 − · · · − s2nx2

n + n(s21 + · · · + s2

n).

So,

n+1

i=1

sixi2

+ n+1

i=1

si2

n+1

k=1

s2k(x2

k + 1) +n+1

k=1

s2k(n

−x2k) =

= (n + 1)n+1k+1

s2k and we are done.

Second solution (by Andrei Negut ):

First, let us prove a lemma

Lemma

For any a1, a2, . . . , a2k+1 ∈ R we have the inequality

k

i=0

a2i+12

≤ 1≤i≤j≤2k+1

(ai +· · ·

+ aj)2 .

Proof of the lemma

Let us take sk = a1 + · · · + ak. We have

ki=0

a2i+1 = s1 + s3 − s2 + · · · + s2k+1 − s2k

and so the left hand side in the lemma is

2k+1i=1

s2i + 2

0≤i<j≤k

s2i+1s2j+1 + 2

1≤i<j≤k

s2is2j − 2

0≤i≤k1≤j≤k

s2i+1s2j

and the right hand side is just

(2k + 1)2k+1i=1

s2i − 2

1≤i<j≤2k+1

sisj .

Thus, we need to prove that

2k2k+1i=1

s2i ≥ 4

0≤i<j≤k

s2i+1s2j+1 + 4

1≤i<j≤k

s2is2j

and it comes by adding up the inequalities

2s2j+1s2i+1 ≤ s22i+1 + s2

2j+1, 2s2is2j ≤ s22i + s2

2j .

Page 103: 22573582 This Work Blends Together Classic Inequality Results With Brand New

8/8/2019 22573582 This Work Blends Together Classic Inequality Results With Brand New

http://slidepdf.com/reader/full/22573582-this-work-blends-together-classic-inequality-results-with-brand-new 103/127

Old and New Inequalities 103

Now, let us turn back to the solution of the problem. Let us call a succession

of  ai’s a sequence and call a sequence that is missing from S  a gap. We group the

successive sequences from S  and thus S  will look like this

S  = {ai1 , ai1+1, . . . , ai1+k1 , ai2 , ai2+1, . . . , ai2+k2 , . . . , air , . . . , air+kr}where ij + kj < ij+1 − 1. Thus, we write S  as a sequence, followed by a gap, followed

by a sequence, then a gap and so on. Now, take s1 = ai1 +· · ·+ai1+k1 , s2 = ai1+k1+1 +

· · · + ai2−1, . . . , s2r−1 = air + · · · + air+kr

Then,i∈S

ai

2

= (s1 + s3 + · · · + s2r−1)2 ≤

1≤i≤j≤2r−1

(si + · · · + sj)2 ≤

1≤i≤j≤n

(ai + · · · + aj)2

the last inequality being clearly true, because the terms in the left hand side are

among those from the right hand side.

Third solution :

We will prove the inequality using induction. For n = 2 and n = 3 it’s easy.

Suppose the inequality is true for all k < n and let us prove it for n. If 1 /∈ S , then

we just apply the inductive step for the numbers a2, . . . , an, because the right hand

side doesn’t decrease. Now, suppose 1 ∈ S . If 2 ∈ S , then we apply the inductive step

with the numbers a1 + a2, a3, . . . , an. Thus, we may assume that 2 /∈ S . It is easy to

see that (a + b + c)2 + c2 ≥ (a + b)2

2≥ 2ab and thus we have (a1 + a2 + · · · + ak)2 +

(a2 + a3 + · · · + ak−1)2 ≥ 2a1ak (*). Also, the inductive step for a3, . . . , an shows that

i∈S\{1}

2

3≤i≤j≤n

(ai + · · · + aj)2 .

So, it suffices to show that

a21 + 2a1

i∈S\{1}

ai≤

n

i=1

(a1 +

· · ·+ ai)

2 +

n

i=2

(a2 +

· · ·+ ai)

2

But this is clear from the fact that a21 appears in the right hand side and by summing

up the inequalities from (*).

111. [ Dung Tran Nam ] Let x1, x2 . . . , x2004 be real numbers in the interval [−1, 1]

such that x31 + x3

2 + . . . +x32004 = 0. Find the maximal value of the x1 +x2 + · · ·+x2004.

Solution :

Let us take ai = x3i and the function f  : [−1, 1] → R, f (x) = x

13 . We will prove

first the following properties of  f :

Page 104: 22573582 This Work Blends Together Classic Inequality Results With Brand New

8/8/2019 22573582 This Work Blends Together Classic Inequality Results With Brand New

http://slidepdf.com/reader/full/22573582-this-work-blends-together-classic-inequality-results-with-brand-new 104/127

104 Solutions

1. f (x + y + 1) + f (−1) ≥ f (x) + f (y) if −1 < x, y < 0.

2. f  is convex on [−1, 0] and concave on [0, 1].

3. If  x > 0 and y < 0 and x + y < 0 then f (x) + f (y) ≤ f (−1) + f (x + y + 1) and if 

x + y > 0 then f (x) + f (y) ≤ f (x + y). The proofs of these results are easy. Indeed,

for the first one we make the substitution x = −a3, y = −b3 and it comes down to

1 > a3+b3+(1−a−b)3 ⇔ 1 > (a+b)(a2−ab+b2)+1−3(a+b)+3(a+b)2−(a+b)3 ⇔⇔ 3(a + b)(1 − a)(1 − b) > 0, which follows. The second statement is clear and the

third one can be easily deduced in the same manner as 1.

From these arguments we deduce that if (t1, t2, . . . , t2004) = t is the point where

the maximum value of the function g : A = {x ∈ [−1, 1]2004|x1 + · · · + xn = 0} →→ R, g(x1, . . . , x2004) =

2004k=1

f (xk) (this maximum exists because this function is

defined on a compact) is attained then we have that all positive components of  t are

equal to each other and all negative ones are −1. So, suppose we have k components

equal to −1 and 2004−k components equal to a number a. Because t1+t2+· · ·+t2004 =

0 we find that a =k

2004 − kand the value of g in this point is (2004−k)

3

 k

2004 − k−k.

Thus we have to find the maximum value of (2004 − k)3

 k

2004 − k− k, when k is in

the set

{0, 1, . . . , 2004

}. A short analysis with derivatives shows that the maximum is

attained when k = 223 and so the maximum value is 3√223 · 3√17812 − 223.

112. [ Gabriel Dospinescu, Calin Popa ] Prove that if  n ≥ 2 and a1, a2, . . . , an

are real numbers with product 1, then

a21 + a2

2 + · · · + a2n − n ≥ 2n

n − 1· n√

n − 1(a1 + a2 + · · · + an − n).

Solution :

We will prove the inequality by induction. For n = 2 it is trivial. Now, suppose the

inequality is true for n−1 numbers and let us prove it for n. First, it is easy to see that

it is enough to prove it for a1, . . . , an > 0 (otherwise we replace a1, a2, . . . , an with|a1|, |a2|, . . . , |an|, which have product 1. Yet, the right hand side increases). Now,

let an the maximum number among a1, a2, . . . , an and let G the geometric mean of 

a1, a2, . . . , an−1. First, we will prove that

a21 + a2

2 + · · · + a2n − n − 2n

n − 1· n√

n − 1(a1 + a2 + · · · + an − n) ≥

≥ a2n + (n − 1)G2 − n − 2n

n − 1· n√

n − 1(an + (n − 1)G − n)

which is equivalent to

a21 + a2

2 + · · · + a2n−1 − (n − 1) n−1

 a21a2

2 . . . a2n−1 ≥

Page 105: 22573582 This Work Blends Together Classic Inequality Results With Brand New

8/8/2019 22573582 This Work Blends Together Classic Inequality Results With Brand New

http://slidepdf.com/reader/full/22573582-this-work-blends-together-classic-inequality-results-with-brand-new 105/127

Old and New Inequalities 105

≥ 2n

n − 1· n√

n − 1

a1 + a2 + · · · + an−1 − (n − 1) n−1√

a1a2 . . . an−1

.

Because, n−1√

a1a2 . . . an−1 ≤ 1 and a1+a2+· · ·+an−1−(n−1) n−1√

a1a2 . . . an−1 ≥0, it is enough to prove the inequality

a21 + · · · + a2

n−1 − (n − 1)G2 ≥ 2n

n − 1· n√

n − 1 · G · (a1 + · · · + an−1 − (n − 1)G).

Now, we apply the inductive hypothesis for the numbersa1

G, . . . ,

an−1

Gwhich have

product 1 and we infer that

a21 +

· · ·+ a2

n−1

G2 − n + 1 ≥2(n

−1)

n − 2 ·n−1

√n − 2a1 +

· · ·+ an−1

G − n + 1

and so it suffices to prove that

2(n − 1)

n − 2· n−1√n − 2(a1+· · ·+an−1−(n−1)G) ≥ 2n

n − 1· n√n − 1(a1+· · ·+an−1−(n−1)G),

which is the same as 1 +1

n(n − 2)≥

n√

n − 1n−1

√n − 2

. This becomes

1 +

1

n(n − 2)

n(n−1)

≥ (n − 1)n−1

(n − 2)n

and it follows for n > 4 from

1 + 1

n(n − 2)n(n−1)

> 2

and

(n − 1)n−1

(n − 2)n=

1

n − 2·

1 +1

n − 2

·

1 +1

n − 2

n−2

<e

n − 2

1 +

1

n − 2

< 2.

For n = 3 and n = 4 it is easy to check.

Thus, we have proved that

a21 + a2

2 + · · · + a2n − n − 2n

n − 1· n√

n − 1(a1 + a2 + · · · + an − n) ≥

≥ a2n + (n − 1)G2 − n − 2n

n−

1· n√

n − 1(an + (n − 1)G − n)

and it is enough to prove that

x2(n−1) +n − 1

x2− n ≥ 2n

n − 1· n√

n − 1

xn−1 +

n − 1

x− n

for all x ≥ 1 (we took x =1

G). Let us consider the function

f (x) = x2(n−1) +n − 1

x2− n − 2n

n − 1· n√

n − 1

xn−1 +

n − 1

x− n

.

We have

f (x) = 2 · xn − 1

x2·

(n − 1)(xn + 1)

x− n n

√n − 1

≥ 0

Page 106: 22573582 This Work Blends Together Classic Inequality Results With Brand New

8/8/2019 22573582 This Work Blends Together Classic Inequality Results With Brand New

http://slidepdf.com/reader/full/22573582-this-work-blends-together-classic-inequality-results-with-brand-new 106/127

106 Solutions

because

xn−1 +1

x= xn−1 +

1

(n − 1)x+ · · · +

1

(n − 1)x≥ n n

 1

(n − 1)n−1.

Thus, f  is increasing and so f (x) ≥ f (1) = 0. This proves the inequality.

113. [ Vasile Cırtoaje ] If  a,b,c are positive real numbers, then 2a

a + b+

 2b

b + c+

 2c

c + a≤ 3.

Gazeta Matematica

First solution :

With the notations x =

 b

a, y =

 c

b, z =

 a

cthe problem reduces to proving

that xyz = 1 implies  2

1 + x2+

 2

1 + y2+

 2

1 + z2≤ 3.

We presume that x ≤ y ≤ z, which implies xy ≤ 1 and z ≥ 1. We have 2

1 + x2+

 2

1 + y2

2

≤ 2

2

1 + x2+

2

1 + y2

= 4

1 +

1 − x2y2

(1 + x2)(1 + y2)

4

1 +

1 − x2y2

(1 + xy)2

=

8

1 + xy=

8z

z + 1.

so  2

1 + x2+

 2

1 + y2≤ 2

 2z

z + 1and we need to prove that

2

 2z

z + 1+

 2

1 + z2≤ 3.

Because  2

1 + z2≤ 2

1 + z

we only need to prove that

2

 2z

z + 1+

2

1 + z≤ 3.

This inequality is equivalent to

1 + 3z − 2 

2z(1 + z) ≥ 0, (√

2z − √z + 1)2 ≥ 0,

and we are done.

Page 107: 22573582 This Work Blends Together Classic Inequality Results With Brand New

8/8/2019 22573582 This Work Blends Together Classic Inequality Results With Brand New

http://slidepdf.com/reader/full/22573582-this-work-blends-together-classic-inequality-results-with-brand-new 107/127

Old and New Inequalities 107

Second solution :

Clearly, the problem asks to prove that if  xyz = 1 then 2

x + 1≤ 3.

We have two cases. The first and easy one is when xy + yz + zx ≥ x + y + z. In this

case we can apply the Cauchy-Schwarz Inequality to get 2

x + 1≤

 3 2

x + 1.

But 1

x + 1≤ 3

2⇔

2(xy + x + y + 1) ≤ 3(2 + x + y + z + xy + yz + zx) ⇔

⇔ x + y + z ≤ xy + yz + zx

and so in this case the inequality is proved.

The second case is when xy + yz + zx < x + y + z. Thus,

(x − 1)(y − 1)(z − 1) = x + y + z − xy − yz − zx > 0

and so exactly two of the numbers x,y,z are smaller than 1, let them be x and y. So,

we must prove that if  x and y are smaller than 1, then

  2

x + 1 +  2

y + 1 + 2xy

xy + 1 ≤ 3.

Using the Cauchy-Schwarz Inequality, we get 2

x + 1+

 2

y + 1+

 2xy

xy + 1≤ 2

 1

x + 1+

1

y + 1+

 2xy

xy + 1

and so it is enough to prove that this last quantity is at most 3. But this comes down

to

2 ·1

x + 1+

1

y + 1− 1

1 +

 1

x + 1+

1

y + 1

≤1 − 2xy

xy + 1

1 +

 2xy

xy + 1

.

Because we have1

x + 1 +1

y + 1 ≥ 1, the left hand side is at most

1

x + 1+

1

y + 1− 1 =

1 − xy

(x + 1)(y + 1)

and so we are left with the inequality

1 − xy

(x + 1)(y + 1)≤ 1 − xy

(xy + 1)

1 +

 2xy

xy + 1

⇔ xy + 1 + (xy + 1)

 2xy

xy + 1≤

≤ xy + 1 + x + y ⇔ x + y ≥ 

2xy(xy + 1)

which follows from  2xy(xy + 1) ≤ 2√

xy ≤ x + y. The problem is solved.

Page 108: 22573582 This Work Blends Together Classic Inequality Results With Brand New

8/8/2019 22573582 This Work Blends Together Classic Inequality Results With Brand New

http://slidepdf.com/reader/full/22573582-this-work-blends-together-classic-inequality-results-with-brand-new 108/127

108 Solutions

114. Prove the following inequality for positive real numbers x,y,z

(xy + yz + zx)

1

(x + y)2+

1

(y + z)2+

1

(z + x)2

≥ 9

4

Iran, 1996

First solution (by Iurie Boreico):

With the substitution x + y = c, y + z = a, z + x = b, the inequality becomes after

some easy computations

2

ab −1

c2

(a − b)2

≥ 0.

Let a ≥ b ≥ c. If 2c2 ≥ ab, each term in the above expression is positive and

we are done. So, let 2c2 < ab. First, we prove that 2b2 ≥ ac, 2a2 ≥ bc. Suppose that

2b2 < ac. Then (b + c)2 ≤ 2(b2 + c2) < a(b + c) and so b + c < a, false. Clearly, we

can write the inequality like that2

ac− 1

b2

(a − c)2 +

2

bc− 1

a2

(b − c)2 ≥

1

c2− 2

ab

(a − b)2.

We can immediately see that the inequality (a − c)2 ≥ (a − b)2 + (b − c)2 holds

and thus if suffices to prove that

2ac + 2bc − 1a2 − 1b2

(b − c)2 ≥

1b2 + 1c2 − 2ab − 2ac

(a − b)2.

But it is clear that1

b2+

1

c2− 2

ab− 2

ac<

1

b− 1

c

2

and so the right hand side

is at most(a − b)2(b − c)2

b2c2. Also, it is easy to see that

2

ac+

2

bc− 1

a2− 1

b2≥ 1

ac+

1

bc>

(a − b)2

b2c2,

which shows that the left hand side is at least(a − b)2(b − c)2

b2c2and this ends the

solution.

Second solution :

Since the inequality is homogenous, we may assume that xy + yz + zx = 3. Also,

we make the substitution x + y + z = 3a. From (x + y + z)2 ≥ 3(xy + yz + zx), we

get a ≥ 1. Now we write the inequality as follows

1

(3a − z)2+

1

(3a − y)2+

1

(3a − x)2≥ 3

4,

4[(xy + 3az)2 + (yz + 3ax)2 + (zx + 3ay)2] ≥ 3(9a − xyz)2,

4(27a4 − 18a2 + 3 + 4axyz) ≥ (9a − xyz)2,

3(12a2 − 1)(3a2 − 4) + xyz(34a − xyz) ≥ 0, (1)

Page 109: 22573582 This Work Blends Together Classic Inequality Results With Brand New

8/8/2019 22573582 This Work Blends Together Classic Inequality Results With Brand New

http://slidepdf.com/reader/full/22573582-this-work-blends-together-classic-inequality-results-with-brand-new 109/127

Old and New Inequalities 109

12(3a2 − 1)2 + 208a2 ≥ (17a − xyz)2. (2)

We have two cases.

i) Case 3a2 − 4 ≥ 0. Since

34a − xyz =1

9[34(x + y + z)(xy + yz + zx) − 9xyz] > 0,

the inequality (1) is true.

ii) Case 3a2 − 4 < 0. From Schur’s Inequality

(x + y + z)3

−4(x + y + z)(xy + yz + zx) + 9xyz

≥0,

it follows that 3a3 − 4a + xyz ≥ 0. Thus,

12(3a2 − 1)2 + 208a2 − (17a − xyz)2 ≥ 12(3a2 − 1)2 + 208a2 − a2(3a2 + 13)2 =

= 3(4 − 11a2 + 10a4 − 3a6) = 3(1 − a2)2(4 − 3a2)2 ≥ 0.

115. Prove that for any x, y in the interval [0, 1], 1 + x2 +

 1 + y2 +

 (1 − x)2 + (1 − y)2 ≥ (1 +

√5)(1 − xy).

Solution (by Faruk F. Abi-Khuzam and Roy Barbara - ”A sharp in-equality and the iradius conjecture”):

Let the function F  : [0, 1]2 → R, F (x, y) =√

1 + x2 + 

1 + y2 + (1 − x)2 + (1 − y)2 − (1 +

√5)(1 − xy). It is clear that F  is symmetric in x and

y and also the convexity of the function x → √1 + x2 shows that F (x, 0) ≥ 0 for all

x. Now, suppose we fix y and consider F  as a function in x. It’s derivatives are

f (x) = (1 +√

5)y +x√

x2 + 1− 1 − x 

(1 − x)2 + (1 − y)2

and

f (x) =1

 (1 + x

2

)3

+(1 − y)2

 ((1 − x)2 + (1 − y)2)3

.

Thus, f  is convex and its derivative is increasing. Now, let

r =

 1 − 6

√3

(1 +√

5)2, c = 1 +

√5.

The first case we will discuss is y ≥ 1

4. It is easy to see that in this case we have cy >

1 y2 − 2y + 2

(the derivative of the function y → y2c2(y2 − 2y + 2) − 1 is positive)

and so f (0) > 0. Because f  is increasing, we have f (x) > 0 and so f  is increasing

with f (0) = F (0, y) = F (y, 0) ≥ 0. Thus, in this case the inequality is proved.

Page 110: 22573582 This Work Blends Together Classic Inequality Results With Brand New

8/8/2019 22573582 This Work Blends Together Classic Inequality Results With Brand New

http://slidepdf.com/reader/full/22573582-this-work-blends-together-classic-inequality-results-with-brand-new 110/127

110 Solutions

Due to case 1 and to symmetry, it remains to show that the inequality holds in

the cases x ∈

0,1

4

, y ∈ [0, r] and x ∈

r,

1

4

, y ∈

r,

1

4

.

In the first case we deduce immediately that

f (x) ≤ r(1 +√

5) +x√

1 + x2− 1 − x 

1 + (x − 1)2

Thus, we have f 

1

4

< 0, which shows that f  is decreasing on

0,

1

4

. Because from

the first case discussed we have f 1

4 ≥

0, we will have F (x, y) = f (x)

≥0 for all

points (x, y) with x ∈

0,1

4

, y ∈ [0, r].

Now, let us discuss the most important case, when x ∈

r,1

4

, y ∈

r,

1

4

. Let

the points O(0, 0), A(1, 0), B(1, 1), C (0, 1), M (1, y), N (x, 1). The triangle OM N  has

perimeter  1 + x2 +

 1 + y2 +

 (1 − x)2 + (1 − y)2 and area

1 − xy

2.

But it is trivial to show that in any triangle with perimeter P  and area S  we have the

inequality S ≤ P 2

12

√3

. Thus, we find that√

1 + x2+ 1 + y2+ (1 − x)2 + (1 − y)2 ≥ 6√3√1 − xy ≥ (1+√5)(1−xy) due to the fact that xy ≥ r2. The proof is complete.

116. [ Suranyi ] Prove that for any positive real numbers a1, a2, . . . , an the fol-

lowing inequality holds

(n−1)(an1 +an2 +· · ·+ann)+na1a2 . . . an ≥ (a1 +a2 +· · ·+an)(an−11 +an−1

2 +· · ·+an−1n ).

Miklos Schweitzer Competition

Solution :

Again, we will use induction to prove this inequality, but the proof of the inductive

step will be again highly non-trivial. Indeed, suppose the inequality is true for nnumbers and let us prove it for n + 1 numbers.

Due to the symmetry and homogeneity of the inequality, it is enough to prove it

under the conditions a1 ≥ a2 ≥ · · · ≥ an+1 and a1 + a2 + · · · + an = 1. We have to

prove that

n

ni=1

an+1i + nan+1

n+1 + nan+1 ·ni=1

ai + an+1 ·ni=1

ai − (1 + an+1)

ni=1

ani + ann+1

≥ 0.

But from the inductive hypothesis we have

(n − 1)(an1 + an2 + · · · + ann) + na1a2 . . . an ≥ an−11 + an−1

2 + · · · + an−1n

Page 111: 22573582 This Work Blends Together Classic Inequality Results With Brand New

8/8/2019 22573582 This Work Blends Together Classic Inequality Results With Brand New

http://slidepdf.com/reader/full/22573582-this-work-blends-together-classic-inequality-results-with-brand-new 111/127

Old and New Inequalities 111

and so

nan+1

ni=1

ai ≥ an+1

ni=1

an−1i − (n − 1)an+1

ni=1

ani .

Using this last inequality, it remains to prove thatn

ni=1

an+1i −

ni=1

ani

− an+1

n

ni=1

ani −ni=1

an−1i

+

+an+1

ni=1

ai + (n − 1)ann+1 − an−1n+1

≥ 0.

Now, we will break this inequality into

an+1

ni=1

ai + (n − 1)ann+1 − an−1n+1

≥ 0

and n

ni=1

an+1i −

ni=1

ani

− an+1

n

ni=1

ani −ni=1

an−1i

≥ 0.

Let us justify these two inequalities. The first one is pretty obviousni=1

ai + (n − 1)ann+1 − an−1n+1 =

ni=1

(ai − an+1 + an+1) + (n − 1)ann+1 − an−1n+1 ≥

≥ ann+1 + an−1n+1 ·

ni=1

(ai − an+1) + (n − 1)ann+1 − an−1n+1 = 0.

Now, let us prove the second inequality. It can be written as

n

ni=1

an+1i −

ni=1

ani ≥ an+1

n

ni=1

ani −ni=1

an−1i

.

Because n

ni=1

ani −ni=1

an−1i ≥ 0 (using Chebyshev’s Inequality) and an+1 ≤ 1

n, it

is enough to prove that

n

n

i=1

an+1i −

n

i=1

ani ≥ 1

n n

n

i=1

ani −n

i=1

an−1i .

but this one follows, because nan+1i +

1

nan−1i ≥ ani for all i. Thus, the inductive step

is proved.

117. Prove that for any x1, x2, . . . , xn > 0 with product 1,1≤i<j≤n

(xi − xj)2 ≥ni=1

x2i − n.

A generalization of Turkevici’s inequality

Page 112: 22573582 This Work Blends Together Classic Inequality Results With Brand New

8/8/2019 22573582 This Work Blends Together Classic Inequality Results With Brand New

http://slidepdf.com/reader/full/22573582-this-work-blends-together-classic-inequality-results-with-brand-new 112/127

112 Solutions

Solution :

Of course, the inequality can be written in the following form

n ≥ −(n − 1)

n

k=1

x2k

+

n

k=1

xk

2

.

We will prove this inequality by induction. The case n = 2 is trivial. Suppose the

inequality is true for n − 1 and let us prove it for n. Let

f (x1, x2, . . . , xn) = −(n − 1)n

k=1

x2k +

n

k=1

xk2

and let G = n−1√

x2x3 . . . xn, where we have already chosen x1 = min{x1, x2, . . . , xn}.

It is easy to see that the inequality f (x1, x2, . . . , xn) ≤ f (x1, G , G , . . . , G) is equivalent

to

(n − 1)n

k=2

x2k −

n

k=2

xk

2

≥ 2x1

n

k=2

xk − (n − 1)G

.

Clearly, we have x1 ≤ G andn

k=2

xk ≥ (n − 1)G, so it suffices to prove that

(n

−1)

n

k=2

x2k

− n

k=2

xk2

≥2G

n

k=2

xk

−(n

−1)G .

We will prove that this inequality holds for all x2, . . . , xn > 0. Because the inequality

is homogeneous, it is enough to prove it when G = 1. In this case, from the induction

step we already have

(n − 2)n

k=2

x2k + n − 1 ≥

n

k=2

xk

2

and so it suffices to prove that

nk=2

x2k + n − 1 ≥

nk=2

2xk ⇔n

k=2

(xk − 1)2 ≥ 0,

clearly true.Thus, we have proved that f (x1, x2, . . . , xn) ≤ f (x1, G , G , . . . , G). Now, to com-

plete the inductive step, we will prove that f (x1, G , G , . . . , G) ≤ 0. Because clearly

x1 =1

Gn−1, the last assertion reduces to proving that

(n − 1)

(n − 1)G2 +

1

G2(n−1)

+ n ≥

(n − 1)G +

1

Gn−1

2

which comes down ton − 2

G2n−2+ n ≥ 2n − 2

Gn−2

and this one is an immediate consequence of the AM-GM Inequality.

Page 113: 22573582 This Work Blends Together Classic Inequality Results With Brand New

8/8/2019 22573582 This Work Blends Together Classic Inequality Results With Brand New

http://slidepdf.com/reader/full/22573582-this-work-blends-together-classic-inequality-results-with-brand-new 113/127

Old and New Inequalities 113

118. [ Gabriel Dospinescu ] Find the minimum value of the expression

ni=1

 a1a2 . . . an

1 − (n − 1)ai

where a1, a2, . . . , an <1

n − 1add up to 1 and n > 2 is an integer.

Solution :

We will prove that the minimal value is1√

nn−3. Indeed, using Suranyi’s In-

equality, we find that

(n − 1)

ni=1

ani + na1a2 . . . an ≥ni=1

an−1i ⇒ na1a2 . . . an ≥

ni=1

an−1i (1 − (n − 1)ai).

Now, let us observe that

nk=1

an−1k (1 − (n − 1)ak) =

nk=1

3

 an−1k

3

 1

1 − (n − 1)ak

2

and so, by an immediate application of  Holder Inequality, we have

ni=1

an−1i (1 − (n − 1)ai) ≥

nk=1

an−

13

k

3

n

k=1

 1

1 − (n − 1)ak

2.

But for n > 3, we can apply Jensen’s Inequality to deduce that

nk=1

an−13

k

n≥

nk=1

ak

n

n−13

=1

nn−13

.

Thus, combining all these inequalities, we have proved the inequality for n > 3.

For n = 3 it reduces to proving that

 abc

b + c − a≥

a

which was proved in the solution of the problem 107.

119. [ Vasile Cırtoaje ] Let a1, a2, . . . , an < 1 be nonnegative real numbers such

that

a =

 a2

1 + a22 + . . . + a2

n

n≥

√3

3.

Page 114: 22573582 This Work Blends Together Classic Inequality Results With Brand New

8/8/2019 22573582 This Work Blends Together Classic Inequality Results With Brand New

http://slidepdf.com/reader/full/22573582-this-work-blends-together-classic-inequality-results-with-brand-new 114/127

114 Solutions

Prove thata1

1 − a21

+a2

1 − a22

+ . . . +an

1 − a2n

≥ na

1 − a2.

Solution :

We proceed by induction. Clearly, the inequality is trivial for n= 1. Now we

suppose that the inequality is valid for n = k − 1, k ≥ 2 and will prove that

a1

1

−a2

1

+a2

1

−a2

2

+ . . . +ak

1

−a2k

≥ ka

1

−a2

,

for

a =

 a2

1 + a22 + . . . + a2

k

k≥

√3

3.

We assume, without loss of generality, that a1 ≥ a2 ≥ . . . ≥ ak, therefore a ≥ ak.

Using the notation

x =

 a2

1 + a2 + · · · + a2k−1

k − 1=

 ka2 − a2

k

k − 1,

from a ≥ ak it follows that x ≥ a ≥√

3

3. Thus, by the inductive hypothesis, we have

a1

1 − a21

+a2

1 − a22

+ . . . +ak−1

1 − a2k−1

≥(k

−1)x

1 − x2 ,

and it remains to show that

ak1 − a2

k

+(k − 1)x

1 − x2≥ ka

1 − a2.

From

x2 − a2 =ka2 − a2

k

k − 1− a2 =

a2 − a2k

k − 1,

we obtain

(k − 1)(x − a) =(a − ak)(a + ak)

x + aand

ak1 − a2

k

+(k − 1)x

1 − x2− ka

1 − a2=

ak

1 − a2k

− a1 − a2

+ (k − 1)

x

1 − x2− a

1 − a2

=

−(a − ak)(1 + aak)

(1 − a2k)(1 − a2)

+(k − 1)(x − a)(1 + ax)

(1 − x2)(1 − a2)=

a − ak1 − a2

−(1 + aak)

1 − a2k

+(a + ak)(1 + ax)

(1 − x2)(x + a)

=

=(a − ak)(x − ak)[−1 + x2 + a2

k + xak + a(x + ak) + a2 + axak(x + ak) + a2xak]

(1 − a2)(1 − a2k)(1 − x2)(x + a)

.

Since x2 − a2k =

ka2 − a2k

k − 1− a2

k =k(a − ak)(a + ak)

k − 1, it follows that

(a − ak)(x − ak) =k(a − ak)2(a + ak)

(k − 1)(x + ak)≥ 0,

Page 115: 22573582 This Work Blends Together Classic Inequality Results With Brand New

8/8/2019 22573582 This Work Blends Together Classic Inequality Results With Brand New

http://slidepdf.com/reader/full/22573582-this-work-blends-together-classic-inequality-results-with-brand-new 115/127

Old and New Inequalities 115

and hence we have to show that

x2 + a2k + xak + a(x + ak) + a2 + axak(x + ak) + a2xak ≥ 1.

In order to show this inequality, we notice that

x2 + a2k =

ka2 + (k − 2)a2k

k − 1≥ ka2

k − 1

and

x + ak ≥ 

x2 + a2k ≥ a

 k

k − 1.

Consequently, we havex2 + a2

k + xak + a(x + ak) + a2 + axak(x + ak) + a2xak ≥ (x2 + a2k) + a(x + ak) + a2 ≥

k

k − 1+

 k

k − 1+ 1

a2 > 3a2 = 1,

and the proof is complete. Equality holds when a1 = a2 = . . . = an.

Remarks.

1. From the final solution, we can easily see that the inequality is valid for the

larger condition

a ≥ 1

 1 +  nn−1+ n

n−1

.

This is the largest range for a, because for a1 = a2 = · · · = an−1 = x and an = 0

(therefore a = x

 n − 1

n), from the given inequality we get a ≥ 1 

1 + 

nn−1

+ nn−1

.

2. The special case n = 3 and a =

√3

3is a problem from Crux 2003.

120. [ Vasile Cırtoaje, Mircea Lascu ] Let a,b,c,x,y,z be positive real numbers

such that

(a + b + c)(x + y + z) = (a2 + b2 + c2)(x2 + y2 + z2) = 4.

Prove that

abcxyz <1

36.

Solution :

Using the AM-GM Inequality, we have

4(ab+bc+ca)(xy+yz+zx) =

(a + b + c)2 − (a2 + b2 + c2)

(x + y + z)2 − (x2 + y2 + z2)

=

= 20 − (a + b + c)2(x2 + y2 + z2) − (a2 + b2 + c2)(x + y + z)2 ≤≤ 20 − 2 (a + b + c)2(x2 + y2 + z2)(a2 + b2 + c2)(x + y + z)2 = 4,

Page 116: 22573582 This Work Blends Together Classic Inequality Results With Brand New

8/8/2019 22573582 This Work Blends Together Classic Inequality Results With Brand New

http://slidepdf.com/reader/full/22573582-this-work-blends-together-classic-inequality-results-with-brand-new 116/127

116 Solutions

therefore

(ab + bc + ca)(xy + yz + za) ≤ 1. (1)

By multiplying the well-known inequalities

(ab + bc + ca)2 ≥ 3abc(a + b + c), (xy + yz + zx)2 ≥ 3xyz(x + y + z),

it follows that

(ab + bc + ca)2(xy + yz + zx)2 ≥ 9abcxyz(a + b + c)(x + y + z),

or(ab + bc + ca)(xy + yz + zx) ≥ 36abcxyz. (2)

From (1) and (2), we conclude that

1 ≤ (ab + bc + ca)(xy + yz + zx) ≥ 36abcxyz,

therefore 1 ≤ 36abcxyz.

To have 1 = 36abcxyz, the equalities (ab + bc + ca)2 = 3abc(a + b + c) and

(xy + yz + zx)2 = 3xyz(x + y + z) are necessary. But these conditions imply a = b = c

and x = y = z, which contradict the relations (a + b + c)(x + y + z) = (a2 + b2 +

c2)(x2 + y2 + z2) = 4. Thus, it follows that 1 > 36abcxyz.

121. [ Gabriel Dospinescu ] For a given n > 2, find the minimal value of the

constant kn, such that if  x1, x2, . . . , xn > 0 have product 1, then

1√1 + knx1

+1√

1 + knx2

+ · · · +1√

1 + knxn≤ n − 1.

Mathlinks Contest

Solution :

We will prove that kn =2n − 1

(n − 1)2. Taking x1 = x2 = · · · = xn = 1, we find that

kn≥

2n − 1

(n − 1)2

. So, it remains to show that

nk=1

1 1 +

2n − 1

(n − 1)2xk

≤ n − 1

if  x1 · x2 · · · · · xn = 1. Suppose this inequality doesn’t hold for a certain system of  n

numbers with product 1, x1, x2, . . . , xn > 0. Thus, we can find a number M > n − 1

and some numbers ai > 0 which add up to 1, such that

1 1 +

2n − 1

(n − 1)2xk

= Mak.

Page 117: 22573582 This Work Blends Together Classic Inequality Results With Brand New

8/8/2019 22573582 This Work Blends Together Classic Inequality Results With Brand New

http://slidepdf.com/reader/full/22573582-this-work-blends-together-classic-inequality-results-with-brand-new 117/127

Old and New Inequalities 117

Thus, ak <1

n − 1and we have

1 =n

k=1

(n − 1)2

2n − 1

1

M 2a2k

− 1

2n − 1

(n − 1)2

n

<n

k=1

1

(n − 1)2a2k

− 1

.

Now, denote 1 − (n − 1)ak = bk > 0 and observe thatn

k=1

bk = 1. Also, the above

inequality becomes

(n

−1)2n

n

k=1

bk·

n

k=1

(2

−bk) > (2n

−1)n

n

k=1

(1

−bk)

2

.

Because from the AM-GM Inequality we haven

k=1

(2 − bk) ≤

2n − 1

n

n

,

our assumption leads ton

k=1

(1 − bk)2 <(n − 1)2n

nnb1b2 . . . bn.

So, it is enough to prove that for any positive numbers a1, a2, . . . , an the inequalityn

k=1(a1 + a2 + · · ·+ ak−1 + ak+1 + · · ·+ an)

2

≥(n

−1)2n

nn a1a2 . . . an(a1 + a2 + · · ·+ an)

n

holds.

This strong inequality will be proved by induction. For n = 3, it follows from the

fact that (a + b)

2

abc≥

8

a

·

ab2

abc≥ 64

27

a3

.

Suppose the inequality is true for all systems of  n numbers. Let a1, a2, . . . , an+1

be positive real numbers. Because the inequality is symmetric and homogeneous, we

may assume that a1 ≤ a2 ≤ · · · ≤ an+1 and also that a1 + a2 + · · · + an = 1. Applying

the inductive hypothesis we get the inequalityni=1

(1 − ai)2 ≥ (n − 1)2n

nna1a2 . . . an.

To prove the inductive step, we must prove thatni=1

(an+1 + 1 − ai)2 ≥ n2n+2

(n + 1)n+1a1a2 . . . anan+1(1 + an+1)n+1.

Thus, it is enough to prove the stronger inequalityni=1

1 +

an+1

1 − ai

2

≥ n3n+2

(n − 1)2n · (n + 1)n+1an+1(1 + an+1)n+1.

Page 118: 22573582 This Work Blends Together Classic Inequality Results With Brand New

8/8/2019 22573582 This Work Blends Together Classic Inequality Results With Brand New

http://slidepdf.com/reader/full/22573582-this-work-blends-together-classic-inequality-results-with-brand-new 118/127

118 Solutions

Now, using Huygens Inequality and the AM-GM Inequality, we find that

ni=1

1 +

an+1

1 − ai

2

1 +an+1

n

  ni=1

(1 − ai)

2n

1 +nan+1

n − 1

2n

and so we are left with the inequality

1 +

nan+1

n − 12n

≥n3n+2

(n − 1)2n · (n + 1)n+1 an+1(1 + an+1)

n+1

if  an+1 ≥ max{a1, a2, . . . , an} ≥ 1

n. So, we can put

n(1 + an+1)

n + 1= 1 + x, where x is

nonnegative. So, the inequality becomes1 +

x

n(x + 1)

2n

≥ 1 + (n + 1)x

(x + 1)n−1.

Using Bernoulli Inequality, we find immediately that1 +

x

n(x + 1)

2n

≥ 3x + 1

x + 1.

Also, (1 + x)n−1 ≥ 1 + (n − 1)x and so it is enough to prove that

3x + 1x + 1

≥ 1 + (n + 1)x1 + (n − 1)x

which is trivial.

So, we have reached a contradiction assuming the inequality doesn’t hold for a

certain system of  n numbers with product 1, which shows that in fact the inequality

is true for kn =2n − 1

(n − 1)2and that this is the value asked by the problem.

Remark .

For n = 3, we find an inequality stronger than a problem given in China Math-

ematical Olympiad in 2003. Also, the case n = 3 represents a problem proposed by

Vasile Cartoaje in Gazeta Matematica, Seria A.

122. [ Vasile Cırtoaje, Gabriel Dospinescu ] For a given n > 2, find the maximal

value of the constant kn such that for any x1, x2, . . . , xn > 0 for which x21 + x2

2 + · · · +

x2n = 1 we have the inequality

(1 − x1)(1 − x2) . . . (1 − xn) ≥ knx1x2 . . . xn.

Page 119: 22573582 This Work Blends Together Classic Inequality Results With Brand New

8/8/2019 22573582 This Work Blends Together Classic Inequality Results With Brand New

http://slidepdf.com/reader/full/22573582-this-work-blends-together-classic-inequality-results-with-brand-new 119/127

Old and New Inequalities 119

Solution :

We will prove that this constant is (√

n − 1)n

. Indeed, let ai = x2i . We must find

the minimal value of the expressionni=1

(1 − √ai)

ni=1

√ai

when a1+a2+· · ·+an = 1. Let us observe that proving that this minimum is (√

n−1)n

reduces to proving thatni=1

(1 − ai) ≥ (√

n − 1)n ·ni=1

√ai ·

ni=1

(1 +√

ai).

But from the result proved in the solution of the problem 121, we find thatni=1

(1 − ai)2 ≥

(n − 1)2

n

n ni=1

ai.

So, it is enough to prove thatni=1

(1 +√

ai) ≤

1 +1√n

n

.

But this is an easy task, because from the AM-GM Inequality we get

ni=1

(1 +√

ai) ≤

1 +

ni=1

√ai

n

n

1 +1√n

n

,

the last one being a simple consequence of the Cauchy-Schwarz Inequality.

Remark .

The case n = 4 was proposed by Vasile Cartoaje in Gazeta Matematica Annual

Contest, 2001.

Page 120: 22573582 This Work Blends Together Classic Inequality Results With Brand New

8/8/2019 22573582 This Work Blends Together Classic Inequality Results With Brand New

http://slidepdf.com/reader/full/22573582-this-work-blends-together-classic-inequality-results-with-brand-new 120/127

Page 121: 22573582 This Work Blends Together Classic Inequality Results With Brand New

8/8/2019 22573582 This Work Blends Together Classic Inequality Results With Brand New

http://slidepdf.com/reader/full/22573582-this-work-blends-together-classic-inequality-results-with-brand-new 121/127

Glossary

(1) Abel’s Summation Formula

If  a1, a2,...,an, b1, b2,...,bn are real or complex numbers, and

S i = a1 + a2 + ... + ai,i = 1, 2,...,n,

thenni=1

aibi =

n−1i=1

S i(bi − bi+1) + S nbn.

(2) AM-GM (Arithmetic Mean-Geometric Mean) Inequality

If  a1, a2,...,an are nonnegative real numbers, then

1

n

ni=1

ai ≥ (a1a2...an)1n ,

with equality if and only if  a1 = a2 = ... = an. This inequality is a special

case of the Power Mean Inequality.

(3) Arithmetic Mean-Harmonic Mean (AM-HM) Inequality

If  a1, a2,...,an are positive real numbers, then

1n

ni=1

ai ≥ 11n

ni=1

1

ai

with equality if and only if  a1 = a2 = ... = an. This inequality is a special

case of the Power Mean Inequality.

(4) Bernoulli’s Inequality

For any real numbers x > −1 and a > 1 we have (1 + x)a ≥ 1 + ax.

121

Page 122: 22573582 This Work Blends Together Classic Inequality Results With Brand New

8/8/2019 22573582 This Work Blends Together Classic Inequality Results With Brand New

http://slidepdf.com/reader/full/22573582-this-work-blends-together-classic-inequality-results-with-brand-new 122/127

Page 123: 22573582 This Work Blends Together Classic Inequality Results With Brand New

8/8/2019 22573582 This Work Blends Together Classic Inequality Results With Brand New

http://slidepdf.com/reader/full/22573582-this-work-blends-together-classic-inequality-results-with-brand-new 123/127

Old and New Inequalities 123

with R.

Concave up and down functions are also called convex and concave, re-

spectively.

If  f  is concave up on an interval [a, b] and λ1, λ2,...,λn are nonnegative

numbers wit sum equal to 1, then

λ1f (x1) + λ2f (x2) + ... + λnf (xn) ≥ f (λ1x1 + λ2x2 + ... + λnxn)

for any x1, x2,...,xn in the interval [a, b]. If the function is concave down,

the inequality is reversed. This is Jensen’s Inequality.

(11) Cyclic Sum

Let n be a positive integer. Given a function f  of  n variables, define the

cyclic sum of variables (x1, x2,...,xn) ascyc

f (x1, x3,...,xn) = f (x1, x2,...,xn) + f (x2, x3,...,xn, x1)

+... + f (an, a1, a2,...,an−1).

(12) Holder’s InequalityIf  r, s are positive real numbers such that

1

r+

1

s= 1, then for any positive

real numbers a1, a2, . . . , an,

b1, b2, . . . , bn,

ni=1

aibi

n≤

ni=1

ari

n

1r

·

ni=1

bsi

n

1s

.

(13) Huygens Inequality

If  p1, p2, . . . , pn, a1, a2, . . . , an, b1, b2, . . . , bn are positive real numbers with

 p1 + p2 + . . . + pn = 1, then

ni=1

(ai + bi) pi ≥

ni=1

a pii +ni=1

b pii .

(14) Mac Laurin’s Inequality

For any positive real numbers x1, x2, . . . , xn,

S 1 ≥ S 2 ≥ . . . ≥ S n,

Page 124: 22573582 This Work Blends Together Classic Inequality Results With Brand New

8/8/2019 22573582 This Work Blends Together Classic Inequality Results With Brand New

http://slidepdf.com/reader/full/22573582-this-work-blends-together-classic-inequality-results-with-brand-new 124/127

124 Solutions

where

S k = k

 

1≤i1<i2<...<ik≤n

xi1 · xi2 · . . . · xikn

k

.

(15) Minkowski’s Inequality

For any real number r ≥ 1 and any positive real numbers

a1, a2, . . . , an, b1, b2, . . . , bn,

ni=1

(ai + bi)r

1

r

ni=1

ari

1

r

+

ni=1

bri

1

r

.

(16) Power Mean Inequality

For any positive real numbers a1, a2, . . . , an with sum equal to 1, and

any positive real numbers x1, x2, . . . , xn, we define M r = (a1xr1 + a2xr2 +

. . . + anxrn)1r if  r is a non-zero real and M 0 = xa11 xa2

2 . . . xann , M ∞ =

max{x1, x2, . . . , xn}, M −∞ = min{x1, x2, . . . , xn}. Then for any reals s ≤ t

we have M −∞ ≤ M s ≤ M t ≤ M ∞.

(17) Root Mean Square InequalityIf  a1, a2,...,an are nonnegative real numbers, then 

a21 + a2

2 + ... + a2n

n≥ a1 + a2 + ... + an

n,

with equality if and only if  a1 = a2 = ... = an.

(18) Schur’s Inequality

For any positive real numbers x,y,z and any r > 0, xr(x−y)(x−z) + yr(y −z)(y − x) + zr(z − x)(z − y) ≥ 0. The most common case is r = 1, which has

the following equivalent forms:

1) x

3

+ y

3

+ z

3

+ 3xyz ≥ xy(x + y) + yz(y + z) + zx(z + x);2) xyz ≥ (x + y − z)(y + z − z)(z + x − y);

3) if x + y + z = 1 then xy + yz + zx ≤ 1 + 9xyz

4.

(19) Suranyi’s Inequality

For any nonnegative real numbers a1, a2, . . . , an,

(n − 1)

nk=1

ank + n

nk=1

ak ≥

nk=1

ak

·

nk=1

an−1k

.

Page 125: 22573582 This Work Blends Together Classic Inequality Results With Brand New

8/8/2019 22573582 This Work Blends Together Classic Inequality Results With Brand New

http://slidepdf.com/reader/full/22573582-this-work-blends-together-classic-inequality-results-with-brand-new 125/127

Old and New Inequalities 125

(20) Turkevici’s Inequality

For any positive real numbers x,y,z,t,

x4 + y4 + z4 + t4 + 2xyzt ≥ x2y2 + y2z2 + z2t2 + t2x2 + x2z2 + y2t2.

(21) Weighted AM-GM Inequality

For any nonnegative real numbers a1, a2,...,an, if  w1, w2,...,wn are nonneg-

ative real numbers (weights) with sum 1, then

w1a1 + w2a2 + ... + wnan ≥ aw1

1 aw2

2 . . . awnn ,

with equality if and only if  a1 = a2 = ... = an.

Page 126: 22573582 This Work Blends Together Classic Inequality Results With Brand New

8/8/2019 22573582 This Work Blends Together Classic Inequality Results With Brand New

http://slidepdf.com/reader/full/22573582-this-work-blends-together-classic-inequality-results-with-brand-new 126/127

Page 127: 22573582 This Work Blends Together Classic Inequality Results With Brand New

8/8/2019 22573582 This Work Blends Together Classic Inequality Results With Brand New

http://slidepdf.com/reader/full/22573582-this-work-blends-together-classic-inequality-results-with-brand-new 127/127

Further reading

1. Andreescu, T.; Feng, Z., 101 Problems in Algebra from the Training of the USA

IMO Team , Australian Mathematics Trust, 2001.2. Andreescu, T.; Feng, Z., USA and International Mathematical Olympiads 2000,

2001, 2002, 2003 , MAA.

3. Andreescu, T.; Feng, Z., Mathematical Olympiads: Problems and Solutions from 

around the World, 1998-1999, 1999-2000 , MAA.

4. Andreescu, T.; Kedlaya, K., Mathematical Contests 1995-1996, 1996-1997,

1997-1998: Olympiad Problems from around the World, with Solutions , AMC.

5. Andreescu, T.; Enescu, B., Mathematical Olympiad Treasures, Birkhauser, 2003.

6. Andreescu, T.; Gelca, R., Mathematical Olympiad Challenges, Birkhauser, 2000.

7. Andreescu, T.; Andrica, D., 360 Problems for Mathematical Contests, GIL

Publishing House, 2002.8. Becheanu, M., Enescu, B., Inegalitat i elementare. . . si mai put in elementare, GIL

Publishing House, 2002

9. Beckenbach, E., Bellman, R. Inequalities, Springer Verlag, Berlin, 1961

10. Drimbe, M.O., Inegalit˘ at i idei si metode, GIL Publishing House, 2003

A S S S